Крок 3 - Медицина 2018 весна (буклет)

1 / 200
Хвора 50-ти рокiв надiйшла в клiнiку зi скаргами на напади сильного головного болю, запаморочення, вiдчуття пульсацiї в головi, пiтливiсть, м’язову слабкiсть. Пiд час нападу пiдвищується артерiальний тиск до 280/170 мм рт.ст. Напади тривають до 30 хвилин. Останнiм часом почастiшали. За даними УЗД - визначається об’ємне утворення правого наднирника. Який препарат найбiльш показаний хворiй? A 50-year-old patient came to the clinic with complaints of attacks of severe headache, dizziness, pulsation in the head, sweating, muscle weakness. During the attack arterial pressure rises to 280/170 mm Hg. Attacks last up to 30 minutes. Recently, they have become more frequent. According to ultrasound, a volume formation of the right adrenal gland is determined. What drug is most indicated for the patient?

Пентамiн в/в Pentamine IV

Дибазол в/в Dibazole IV

Фуросемiд в/в IV Furosemide

Папаверин в/в Papaverine IV

Фентоламiн в/в IV Phentolamine

2 / 200
У першовагiтної 27-ми рокiв вагiтнiсть 39-40 тижнiв, положення плоду поздовжнє, голiвка притиснута до площини входу в малий таз. Розмiри тазу 23-26-29-18. Перейми болючi, тривалiсть 10 годин, навколоплiднi води вiдiйшли 2 години тому, родiлля неспокiйна. Серцебиття плоду ритмiчне, 160/хв. В ходi зовнiшнього акушерського обстеження встановлено: контракцiйне кiльце розташоване косо, розташоване вище пупа, ознака Вастена позитивна, матка в нижньому сегментi рiзко болюча, круглi зв’язки матки напруженi. Яку патологiю можна припустити? A 27-year-old primigravid woman is 39-40 weeks pregnant, the position of the fetus is longitudinal, the head is pressed against the plane of the entrance to the small pelvis. The dimensions of the pelvis are 23-26-29- 18. Painful contractions, lasting 10 hours, amniotic fluid came out 2 hours ago, labor is restless. Fetal heartbeat is rhythmic, 160/min. During the external obstetric examination, it was found: the contraction ring is located obliquely, located above the navel, Vasten's sign is positive, the uterus is in the lower the segments are sharply painful, the round ligaments of the uterus are tense. What pathology can be assumed?

Розрив матки, що вiдбувся Uterine rupture that occurred

Передчасне вiдшарування плаценти Premature placental abruption

Слабкiсть пологової дiяльностi Weak birth activity

Загроза розриву матки Threat of uterine rupture

Дискоординована пологова дiяльнiсть Discoordinated birth activity

3 / 200
Потерпiлого знайдено непритомним на вулицi пiд час зливи з грозою. На потилицi рвана рана розмiром 4х2 см. Чiтких даних щодо враження електричним струмом або блискавкою, через неможливiсть провести детальний огляд, не виявлено. До якого вiддiлення проводиться госпiталiзацiя? The victim was found unconscious on the street during a downpour with a thunderstorm. On the back of the head there is a lacerated wound measuring 4x2 cm. Clear data on the effect of electric current or lightning, due to the impossibility of conducting a detailed examination , not detected. To which department is hospitalization carried out?

Неврологiчного Neurological

Нейрохiрургiчного Neurosurgical

Хiрургiчного Surgical

Полiтравми Polytrauma

Реанiмацiйного Resuscitator

4 / 200
Хворий 48-ми рокiв скаржиться на задишку, що раптово посилилася, слабкiсть. Перебуваєу кардiологiчному вiддiленнi впродовж 12-ти годин, отримуєгепарин, аспiрин, стрептокiназу. Збуджений, шкiра холодна, волога, цiаноз. Пульс слабкого наповнення, ритмiчний, 135/хв., артерiальний тиск - 60/40 мм рт.ст. У нижнiх вiддiлах легень вологi хрипи. На ЕКГ у вiдведеннях V 3 - V 6 комплекс QRS типу QS, пiдйом сегменту ST вище iзолiнiї, позитивний зубець T. З якого препарату необхiдно починати невiдкладну терапiю? A 48-year-old patient complains of shortness of breath, which suddenly worsened, weakness. He has been in the cardiology department for 12 hours, receiving heparin, aspirin, streptokinase. Excited, skin cold, wet, cyanosis. The pulse is weakly filling, rhythmic, 135/min., blood pressure - 60/40 mm Hg. In the lower parts of the lungs, wet rales. On the ECG, in leads V 3 - V 6, a QRS complex of the QS type, an elevation ST segment above the isoline, positive T wave. Which drug should be used to start emergency therapy?

Фуросемiд Furosemide

Верапамiл Verapamil

Дигоксин Digoxin

Дофамiн Дофамiн

Альбумiн Albumin

5 / 200
У хлопчика 10-ти рокiв через 5 хвилин пiсля iн’єкцiї бiцилiну-5 з’явилися млявiсть, бiль за грудиною i в дiлянцi серця, утруднене дихання, судоми з наступною втратою свiдомостi. Дихання аритмiчне, тони серця глухi, пульс - 130/хв., слабкого наповнення, артерiальний тиск - 80/40 мм рт.ст. З чого треба почати невiдкладну допомогу? 5 minutes after the bicillin-5 injection, a 10-year-old boy developed lethargy, pain behind the sternum and in the region of the heart, difficulty breathing, convulsions with subsequent loss of consciousness. Breathing is arrhythmic, heart sounds are dull, pulse - 130/min., weak filling, blood pressure - 80/40 mm Hg. Where should emergency care be started?

Внутрiшньом’язово ввести антигiстамiннi препарати Intramuscularly inject antihistamines

Обколоти мiсце iн’єкцiї адреналiном Surround the injection site with adrenaline

Внутрiшньовенно ввести серцевi глiкозиди Enter cardiac glycosides intravenously

Внутрiшньовенно ввести глюкокортикоїди Glucocorticoids should be administered intravenously

Почати оксигенотерапiю Start oxygen therapy

6 / 200
Хворий 43-х рокiв доставлений швидкою медичною допомогою у приймальний покiй ЦРЛ зi скаргами на слабкiсть, що виникла раптово, запаморочення, мерехтiння ”мушок” перед очима, одноразовi ряснi рiдкi випорожнення дьогтеподiбної консистенцiї. Об’єктивно: блiдий, пульс - 100/хв., пальпацiя черевної порожнини безболiсна, симптоми подразнення очеревини негативнi. Який метод дiагностики необхiдно провести хворому в першочерговому порядку? A 43-year-old patient was brought by ambulance to the reception room of the Central Hospital with complaints of sudden weakness, dizziness, flickering of 'flies' in front of the eyes, one-time abundant liquid stools of a tar-like consistency. Objectively: pale, pulse - 100/min., palpation of the abdominal cavity is painless, symptoms of peritoneal irritation are negative. What diagnostic method should be performed on the patient as a matter of priority?

Рентгенографiя шлунку i дванадцятипалої кишки X-ray of stomach and duodenum

УЗД органiв черевної порожнини Ultrasound of abdominal organs

Спостереження хiрурга у палатi невiдкладної терапiї Observation of a surgeon in the emergency room

Фiброгастродуоденоскопiя Fibrogastroduodenoscopy

Аналiз калу на приховану кров Stool analysis for occult blood

7 / 200
Хворого доставили у стацiонар з клiнiчною картиною загального переохолодження: сопор, шкiрнi покриви мармуровоцiанотичнi, ректальна температура - 32oC, брадикардiя 48 /хв., артерiальна гiпотензiя - 80/30 мм рт.ст. Який захiд з перерахованих необхiдний в першу чергу? The patient was brought to the hospital with a clinical picture of general hypothermia: sopor, marble cyanotic skin, rectal temperature - 32oC, bradycardia 48/min, arterial hypotension - 80/30 mm Hg. Which measure from the listed is necessary first of all?

Обкласти хворого грiлками з температурою 40oC Cover the patient with heating pads with a temperature of 40oC

Негайне занурення потерпiлого в гарячу ванну (t - 45oC) Immediate immersion of the victim in a hot bath (t - 45oC)

Дати хворому випити алкоголю Give the patient to drink alcohol

Внутрiшньовенна iнфузiя кристалоїдiв нагрiтих до температури 37oC Intravenous infusion of crystalloids heated to a temperature of 37oC

Розтерти шкiрнi покриви спиртом Rub the skin with alcohol

8 / 200
В приймальне вiддiлення доставили дитину 7-ми рокiв. Стан свiдомостi - сопор. На шкiрi тулуба та кiнцiвок спостерiгаються геморагiчнi зiрчастi висипи, що зливаються мiж собою та мають в центрi дiлянки некрозу. Менiнгеальнi ознаки рiзко позитивнi. Дихання часте, поверхневе, артерiальний тиск - 50/0 мм рт.ст., частота серцевих скорочень - 132/хв. Що з перерахованого необхiдно застосувати для корекцiї гемодинамiки в першу чергу? Крок 3 Загальна лiкарська пiдготовка (україномовний варiант) 2018 рiк, весна 2 A 7-year-old child was brought to the reception department. The state of consciousness is sopor. On the skin of the trunk and extremities, hemorrhagic star-like rashes are observed, which merge with each other and have in the center areas of necrosis. Meningeal signs are strongly positive. Breathing is frequent, superficial, blood pressure - 50/0 mm Hg, heart rate - 132/min. Which of the following should be used to correct hemodynamics first of all? Step 3 General medical training (Ukrainian-language version) 2018, spring 2

Кристалоїди + Антибiотик Crystal + Antibiotic

Адреномiметики + Колоїди Adrenomimetics + Colloids

Глюкокортикоїди + Антибiотик Glucocorticoids + Antibiotic

Колоїди + Глюкокортикоїди Colloids + Glucocorticoids

Антигiстамiннi препарати + Адреномiметики Antihistamines + Adrenomimetics

9 / 200
У хворого 32-х рокiв з’явилися загальна слабкiсть, пiтливiсть, вiдчуття печiння та м’язовi посмикування в правiй стопi, яка була травмована за три днi до цього пiд час прогулянки в лiсi. При надходженнi до стацiонару температура тiла - 37,1oC, куточки рота вiдтягнутi догори, очнi щiлини звуженi, дрiбнi зморшки в куточках очей та на лобi. Права стопа набрякла, гiперемована, на пiдошвi колота рана. Тонус м’язiв рук i нiг, за винятком кистей i стоп, тулуба значно пiдвищений. Пiд час огляду з’явилися генералiзованi судоми. Яку невiдкладну допомогу потрiбно надати хворому? A 32-year-old patient developed general weakness, sweating, a burning sensation, and muscle twitching in the right foot, which was injured three days before during a walk in the forest. When entering the hospital, the body temperature is 37.1oC, the corners of the mouth are pulled up, the eye slits are narrowed, small wrinkles in the corners of the eyes and on the forehead. The right foot is swollen, hyperemic, there is a puncture wound on the sole. Muscle tone arms and legs, with the exception of the hands and feet, the trunk is significantly elevated. During the examination, generalized convulsions appeared. What emergency help should be given to the patient?

Первинна хiрургiчна обробка та введення спазмолiтикiв Primary surgical treatment and administration of antispasmodics

Застосування хлоралгiдрату, нейролептаналгезiя та введення лiкувальної сироватки Application of chloral hydrate, neuroleptanalgesia and administration of therapeutic serum

Застосування мiорелаксантiв та переведення на штучну вентиляцiю легень (ШВЛ) Use of muscle relaxants and transfer to artificial lung ventilation (VHL)

Введення пенiцилiну Introduction of penicillin

Введення лiкувальної сироватки та первинна хiрургiчна обробка рани Introduction of therapeutic serum and initial surgical treatment of the wound

10 / 200
У хворої протягом 2-х годин спостерiгаються дуже частi напади, пiд час яких вона перебуваєв одному положеннi, погляд її спрямований в одну точку, а в окремих м’язах обличчя i рук постiйно виникають короткочаснi мимовiльнi посмикування. Свiдомiсть у хворої весь час порушена. Ваша тактика: The patient had very frequent seizures for 2 hours, during which she was in one position, her gaze was directed to one point, and in some muscles there are constant short-term involuntary twitches of the face and hands. The patient's consciousness is disturbed all the time. Your tactics:

Призначення консультацiї лiкаряепiлептолога Physician epileptologist consultation appointment

Госпiталiзацiя в неврологiчне вiддiлення Hospitalization in the neurological department

Госпiталiзацiя в реанiмацiйне вiддiлення Hospitalization in intensive care unit

Призначення амбулаторного лiкування Appointment of outpatient treatment

Госпiталiзацiя в психiатричне вiддiлення Hospitalization in a psychiatric ward

11 / 200
В приймальне вiддiлення доставлений 6ти рiчний хлопчик, у якого через 30 хвилин пiсля введення ампiцилiну внутрiшньом’язово з’явилися блiдiсть, слабкiсть, вiдчуття стискання за грудиною та нестачi повiтря, утруднене дихання. Артерiальний тиск - 70/40 мм рт.ст. Черговий лiкар припускаєанафiлактичний шок. Що треба виконати в першу чергу? A 6-year-old boy was brought to the emergency department, who developed pallor, weakness, chest tightness, and shortness of breath 30 minutes after the intramuscular injection of ampicillin , difficulty breathing. Blood pressure - 70/40 mm Hg. The doctor on duty suggests an anaphylactic shock. What should be done first of all?

Кортикостероїди Corticosteroids

Димедрол Diphenhydramine

Ввести адреналiн Inject Adrenaline

Атропiн Atropine

Серцевi глiкозиди Cardiac glycosides

12 / 200
Через 7 годин пiсля вживання тiстечка дитина 12-ти рокiв захворiла з нудотою, повторним блюванням, болем в животi, температурою тiла, пiдвищеною до 38,5oC, млявiстю. Вiдомо, що в цiй родинi ще двi особи захворiли з аналогiчною клiнiчною картиною. Дитина надiйшла до iнфекцiйного вiддiлення. Якою буде невiдкладна першочергова допомога? 7 hours after eating the cake, a 12-year-old child fell ill with nausea, repeated vomiting, abdominal pain, body temperature increased to 38.5oC, lethargy. It is known that two more people in this family fell ill with a similar clinical picture. The child was admitted to the infectious disease department. What will be the emergency first aid?

Ввести в/м церукал Enter IM Cerucal

Дати смекту для внутрiшнього прийому Give smecta for internal reception

Термiново промити шлунок та кишкiвник Urgently flush the stomach and intestines

Ввести в/м пенiцилiн Give IM penicillin

Ввести в/м левомiцетин Inject IM chloramphenicol

13 / 200
Хворий 50-ти рокiв скаржиться на погiршення зору, головний бiль, загальну слабiсть, вiдчуття нестачi повiтря. Цi явища виникли через 1,5-2 години пiсля роботи на городi з iнсектицидними препаратами. Об’єктивно: цiаноз слизових, фiбрилярнi посмикування окремих м’язiв обличчя. В легенях жорстке дихання, незвучнi сухi свистячi хрипи. Спостерiгаються брадикардiя, зниження артерiального тиску до 90/60 мм рт.ст. З метою антидотної терапiї даному хворому рекомендується призначити: A 50-year-old patient complains of worsening vision, headache, general weakness, feeling of lack of air. These phenomena occurred 1.5-2 hours after working at gardens with insecticide preparations. Objectively: cyanosis of mucous membranes, fibrillar twitching of certain facial muscles. Hard breathing in the lungs, soundless dry whistling wheezing. Bradycardia is observed, a decrease in blood pressure to 90/60 mm Hg. For the purpose of antidote therapy for this the patient is recommended to prescribe:

5% розчину унiтiолу дом’язово 5% unitiol solution intramuscularly

0,1% розчину атропiну пiдшкiрно 0.1% atropine solution subcutaneously

30% розчину етилового спирту всередину 30% ethyl alcohol solution inside

3% розчину нiтриту натрiю довенно 3% sodium nitrite solution daily

10% розчину етилового спирту на 5% розчинi глюкози довенно 10% ethyl alcohol solution on 5% glucose solution daily

14 / 200
Хворий 31-го року госпiталiзований з приводу кетоацидотичної коми. Проведенi невiдкладнi заходи. Однак, незважаючи на покращення лабораторних показникiв ( глiкемiя - 8,6 ммоль/л, на момент надходження - 32,0 ммоль/л, K+ - 3,6 ммоль/л, на момент надходження - 2,8 ммоль/л), хворий перебуваєв станi глибокої коми. Яка найбiльш iмовiрна причина непритомного стану? The 31-year-old patient was hospitalized due to a ketoacidotic coma. Emergency measures were taken. However, despite the improvement of laboratory parameters (glycemia - 8.6 mmol/l, on at the time of admission - 32.0 mmol/l, K+ - 3.6 mmol/l, at the time of admission - 2.8 mmol/l), the patient was in a state of deep coma. What is the most likely cause of unconsciousness?

Набряк мозку Brain edema

Тромбоз мозкових судин Thrombosis of cerebral vessels

Гiпоглiкемiчний стан Hypoglycemic state

Крововилив у головний мозок Hemorrhage in the brain

Гiпокалiємiя Hypokalemia

15 / 200
Хворий 60-ти рокiв скаржиться на бiль в лiвiй половинi грудної клiтки, який пiдсилюється при поворотах тiла. Застосування нiтроглiцерину не дало ефекту. Бiль зменшився пiсля застосування диклофенаку. Аускультативна картина легенiв та серця, ЕКГ без патологiї. Поставте дiагноз: A 60-year-old patient complains of pain in the left half of the chest, which intensifies when turning the body. The use of nitroglycerin had no effect. The pain decreased after the use of diclofenac. Auscultative picture of the lungs and heart, ECG without pathology. Make a diagnosis:

Пневмонiя Pneumonia

Мiжреберна невралгiя Intercostal neuralgia

Плеврит Pleuri

Стенокардiя напруги Tension angina

Iнфаркт мiокарда Myocardial infarction

16 / 200
Хвора 35-ти рокiв, що страждаєна закрепи, скаржиться на сильний, довготривалий бiль пiсля дефекацiї, наявнiсть ”смужки” кровi в калi. Через бiль обмежила вживання їжi. Поставте попереднiй дiагноз: A 35-year-old patient suffering from constipation complains of severe, long-lasting pain after defecation, the presence of a 'strip' of blood in her stool. She limited her food intake because of the pain. Make a preliminary diagnosis:

Гострий парапроктит Acute paraproctitis

Гострий тромбоз гемороїдальних вузлiв Acute thrombosis of hemorrhoidal nodes

Неповна параректальна нориця Incomplete pararectal fistula

Анакуприковий больовий синдром Anacupric pain syndrome

Анальна трiщина Anal fissure

17 / 200
Дiвчинка 12-ти рокiв скаржиться на поКрок 3 Загальна лiкарська пiдготовка ( україномовний варiант) 2018 рiк, весна 3 яву червоного кольору сечi, лихоманку до 38,0oC, висип на кiнцiвках. При оглядi стан важкий, на кiнцiвках, сiдницях симетрично розташований геморагiчний висип. В загальному аналiзi кровi: еритроцити - 3, 2 • 1012/л, тромбоцити - 180 • 109/л. В загальному аналiзi сечi: еритроцити вкривають все поле зору, лейкоцити - одиночнi в препаратi. Поставте дiагноз: A 12-year-old girl complains about Step 3 General medical training (Ukrainian-language version) 2018, spring 3 of red urine, fever up to 38.0oC, rash on the extremities. On examination, the condition is severe, a hemorrhagic rash is symmetrically located on the extremities, buttocks. In the general analysis of blood: erythrocytes - 3.2 • 1012/l, platelets - 180 • 109/l. In the general analysis of urine: erythrocytes cover the entire field of vision , leukocytes - single in the preparation. Make a diagnosis:

Алергiчна реакцiя Allergic reaction

Кiр Enter

Краснуха Krasnukha

Геморагiчний васкулiт Hemorrhagic vasculitis

Iдiопатична тромбоцитопенiчна пурпура Idiopathic thrombocytopenic purpura

18 / 200
Юнак одержав травму пiд час дорожньотранспортної пригоди (ДТП). Свiдомiсть була вiдсутня протягом 2 хвилин. Пiсля цього скарги на головний бiль, запаморочення, нудоту, блювання. Не пам’ятає, що було пiд час дорожньо-транспортної пригоди (ДТП). Брадикардiя. Гiпотензiя. Шкiра блiда. Анiзокорiя. Сухожильнi рефлекси трохи зниженi. Поставте дiагноз: The young man was injured during a traffic accident. He lost consciousness for 2 minutes. After that, he complained of headache, dizziness, nausea, vomiting. I don't remember 'remembers what happened during the traffic accident (traffic accident). Bradycardia. Hypotension. Pale skin. Anisocoria. Tendon reflexes are slightly reduced. Make a diagnosis:

Струс головного мозку Concussion

Закритий перелом черепу Closed skull fracture

Забiй головного мозку Brain stroke

Внутрiшня кровотеча Internal bleeding

Крововилив в головний мозок Hemorrhage in the brain

19 / 200
У 25-рiчного хворого на цукровий дiабет I типу через 1 годину пiсля чергової iн’єкцiї виникли слабкiсть, вiдчуття голоду, пiтливiсть. Через 10 хвилин виникли судоми, втрата свiдомостi. Пiд час огляду: непритомний, тонiчнi та клонiчнi судоми м’язiв. Шкiра волога. Артерiальний тиск - 140/80 мм рт.ст., пульс - 90/хв., ритмiчний. Запаху ацетону у повiтрi, що видихається, немає. Необхiдна перша допомога: A 25-year-old patient with type I diabetes developed weakness, hunger, and sweating 1 hour after the next injection. 10 minutes later, convulsions occurred, loss consciousness. During examination: unconscious, tonic and clonic muscle spasms. Skin moist. Blood pressure - 140/80 mm Hg, pulse - 90/min, rhythmic. There is no smell of acetone in exhaled air. First aid is necessary:

В/в струменево 40% р-н глюкози 60,0 IV jet 40% solution of glucose 60.0

В/в струменево 40% р-н глюкози 500,0 IV jet 40% solution of glucose 500.0

В/в крапельно 20% р-н глюкози 60,0 з 10 ОД iнсулiну IV drip 20% solution of glucose 60.0 with 10 units of insulin

В/в крапельно 5% р-н глюкози 500,0 In/in drip 5% solution of glucose 500.0

В/в струменево 40% р-н глюкози 60,0 з 6 ОД iнсулiну IV jet 40% solution of glucose 60.0 with 6 units of insulin

20 / 200
Дитина 5-ти рокiв захворiла, гостре пiдвищення to до 38-39oC, явища нежитю, посилений головний бiль, супроводжений нудотою i блюванням. Об’єктивно: загальна гiперестезiя, ригiднiсть м’язiв шиї, симптоми Кернiга, Брудзинського. В областi стегон геморагiчний висип. Який з методiв обстеження необхiдно провести для уточнення дiагнозу? A 5-year-old child fell ill, acute temperature rise to 38-39oC, runny nose, severe headache, accompanied by nausea and vomiting. Objectively: general hyperesthesia , stiffness of the neck muscles, symptoms of Kernig, Brudzinsky. Hemorrhagic rash in the thigh area. Which of the examination methods should be performed to clarify the diagnosis?

Променева дiагностика придаткових пазух носа X-ray diagnostics of paranasal sinuses

Комп’ютерна томографiя головного мозку Computed tomography of the brain

Люмбальна пункцiя Lumbar puncture

МРТ головного мозку MRI brain

Загальний аналiз кровi General blood analysis

21 / 200
Хвора 48-ми рокiв оперована з приводу багатовузлового еутиреоїдного зоба IV ступеня. Операцiя на погляд хiрурга пройшла без суттєвих ускладнень. Але у хворої з’явилася захриплiсть голосу, яка не зникла i при подальшому спостереженнi пiд час перебування її в стацiонарi. Яке пiсляоперацiйне ускладнення настало у хворої? A 48-year-old patient was operated on for multinodular euthyroid goiter of the IV degree. In the surgeon's view, the operation went without significant complications. But the patient developed hoarseness, which did not disappear even during further observation during her stay in the hospital. What postoperative complications did the patient experience?

Пошкодження блукаючого нерву Damage of the vagus nerve

Гематома загрудинного простору Retrosternal Hematoma

Пошкодження одного зворотного нерва Damage of one recurrent nerve

Пошкодження трахеї Damage to the trachea

Пошкодження обох зворотних нервiв Damage of both back nerves

22 / 200
У хворого явища гострої дихальної недостатностi. При аускультацiї ослаблене дихання лiворуч. Рентгенологiчно: ексудативний плеврит лiворуч до III ребра. З чого варто розпочати лiкувальнi заходи? The patient has acute respiratory failure. On auscultation, breathing is weakened on the left side. X-ray: exudative pleurisy on the left side up to the 3rd rib. Where should treatment measures be started?

Антибактерiальна + дегiдратацiйна терапiя, за неефективностi - пункцiя i дренування плевральної порожнини Antibacterial + dehydration therapy, if ineffective - puncture and drainage of the pleural cavity

Дренування лiвої плевральної порожнини в V-VI мiжребер’ї по середнiй пахвовiй лiнiї Drainage of the left pleural cavity in the V-VI intercostal space along the middle axillary line

Симптоматична + антибактерiальна терапiя Symptomatic + antibacterial therapy

Пункцiя плевральної порожнини в II мiжребер’ї по середньоключичнiй лiнiї Puncture of the pleural cavity in the II intercostal space along the midclavicular line

Пункцiя плевральної порожнини в VIIVIII мiжребер’ї по заднiй аксилярнiй лiнiї Puncture of the pleural cavity in the VIIVIII intercostal space along the posterior axillary line

23 / 200
У дитини захворювання почалося з пiдвищення температури до 38,0oC та появи висипу. Спочатку висип мав вигляд плям, потiм з’явилися пухирцi. Пiд час огляду на шкiрi обличчя, тулуба, кiнцiвок i на волосянiй частинi голови спостерiгається рясний висип: папули, везикули, кiрочки. Дiагноз: вiтряна вiспа. Протягом якого перiоду дитина має бути iзольована? The child's illness began with an increase in temperature to 38.0oC and the appearance of a rash. At first, the rash had the appearance of spots, then blisters appeared. During the examination of the skin of the face , trunk, extremities, and on the hairy part of the head, a profuse rash is observed: papules, vesicles, crusts. Diagnosis: abraded island. During what period should the child be isolated?

21 день 21 days

До появи останнiх висипiв Before the appearance of the last rashes

До 5-го дня пiсля появи останнiх висипiв Until the 5th day after the appearance of the last rashes

14 днiв 14 days

Iзоляцiї не потребує Isolation is not required

24 / 200
У дитини 9-ти рокiв, хворої на черевний тиф, на 18-ту добу хвороби з’явилися слабкiсть, блiдiсть шкiри та слизових оболонок, холодний пiт, тахiкардiя, артерiальний тиск 80/20 мм рт.ст. Через 12 годин з’явилися чорнi випорожнення. З чим може бути пов’язане погiршення стану? A 9-year-old child with typhoid fever developed weakness, pallor of the skin and mucous membranes, cold sweat, tachycardia on the 18th day of the illness , blood pressure 80/20 mm Hg. After 12 hours, black stools appeared. What could be associated with the deterioration of the condition?

Перфорацiєю кишечнику Intestinal perforation

Гострим апендицитом Acute appendicitis

Кишковою кровотечею Intestinal bleeding

Кишечною суперiнфекцiєю Intestinal superinfection

Перитонiтом Peritoneum

25 / 200
Пацiєнт 64-х рокiв, що перебуває у стацiонарi з приводу гострого iнфаркту мiокарда, раптово знепритомнiв. При оглядi дiагностованi зупинка серцевої дiяльностi i подиху, розпочато легенево-серцеву реанiмацiю. На ЕКГ шлуночкова тахiкардiя. Далi необхiдно: Крок 3 Загальна лiкарська пiдготовка (україномовний варiант) 2018 рiк, весна 4 A 64-year-old patient, who is hospitalized for an acute myocardial infarction, suddenly fainted. Cardiac and respiratory arrest was diagnosed during the examination, cardiopulmonary resuscitation was started . Ventricular tachycardia on the ECG. Next, it is necessary: ​​Step 3 General medical training (Ukrainian-language version) 2018, spring 4

Ввести внутрiшньосерцево адреналiн Inject intracardiac adrenaline

Ввести внутрiшньовенно магнiю сульфат Inject magnesium sulfate intravenously

Дефiбриляцiя Defibrillation

Ввести внутрiшньосерцево атропiн Inject intracardiac atropine

Ввести внутрiшньовенно кальцiю хлорид Inject calcium chloride intravenously

26 / 200
Хвора 42-х рокiв доставлена у сопорозному станi до лiкарнi машиною швидкої допомоги. З анамнезу, що зiбраний зi слiв рiдних, вiдомо, що хвора страждаєна Аддiсонову хворобу. Напередоднi вiдзначалося неодноразове блювання. Пiд час огляду: риси обличчя загостренi, гiперпiгментацiя шкiри, артерiальний тиск - 50/30 мм рт.ст. У додаткових дослiдженнях: цукор кровi - 3,5 ммоль/л, аглюкозурiя, вiдсутнiсть ацетону в сечi, осмолярнiсть кровi - 295 мосмоль/л. Який дiагноз найбiльш iмовiрний? A 42-year-old patient was brought to the hospital in a serious condition by an ambulance. From the history collected from the words of relatives, it is known that the patient suffers from Addison's disease. The day before repeated vomiting was noted. During the examination: facial features are sharpened, hyperpigmentation of the skin, blood pressure - 50/30 mm Hg. In additional studies: blood sugar - 3.5 mmol/l, aglucosuria, absence of acetone in the urine, blood osmolality - 295 mosmol/l. What is the most likely diagnosis?

Аддiсонова криза Addison's Crisis

Гiперкетонемiчна прекома Hyperketonemic precoma

Гiперлактацидемiчна кома Hyperlactacidemic Coma

Гiперацидотична кома Hyperacid Coma

Гiпопiтуїтарна кома Hypopituitary Coma

27 / 200
Хворий на алкоголiзм на третiй день пiсля проведеної резекцiї шлунка вночi став збудженим: робив спроби вставати з лiжка, кричав, що ”по ньому бiгають щурi, повзають змiї; струшував їх iз себе”. Дезорiєнтований в навколишньому середовищi. Артерiальний тиск - 90/60 мм рт.ст. З якого препарату необхiдно розпочати надання невiдкладної допомоги? On the third day after gastric resection, the patient became agitated at night: he tried to get out of bed, shouted that 'rats are running on him, snakes are crawling; he was shaking them from themselves'. Disoriented in the surrounding environment. Blood pressure - 90/60 mm Hg. With which drug is it necessary to start providing emergency care?

Димедрол Diphenhydramine

Дiазепам Дiазепам

Ноотропiл Nootropic

Амiтриптилiн Amitriptyline

Амiназин Aminazine

28 / 200
Хвора 45-ти рокiв скаржиться на появу протягом двох тижнiв випорожнень кров’ю бiльш нiж 12 разiв на добу, болю в животi, суглобах; болючi висипи у порожнинi рота та нижнiх кiнцiвках, набряк колiнних та гомiлковостопних суглобiв, слабкiсть, пiдвищення температури тiла понад 39oC, втрату ваги тiла. Який найбiльш iмовiрний дiагноз? A 45-year-old patient complains of the occurrence of bloody stools more than 12 times a day, pain in the abdomen, joints, painful rashes in the oral cavity for two weeks and lower extremities, swelling of knee and ankle joints, weakness, increase in body temperature over 39oC, loss of body weight. What is the most likely diagnosis?

Ревматоїдний артрит Rheumatoid arthritis

Неспецифiчний виразковий колiт Nonspecific ulcerative colitis

Хвороба Бехчета Behcet's disease

Рак прямої кишки Rectal cancer

Гемороїдальна кровотеча Hemorrhoidal bleeding

29 / 200
Чоловiк 58-ми рокiв, що перебував одну добу в палатi iнтенсивної терапiї з приводу iнфаркту мiокарда, раптово знепритомнiв. Об’єктивно: шкiра блiда, пульс на A 58-year-old man, who spent one day in the intensive care unit due to a myocardial infarction, suddenly fainted. Objectively: the skin is pale, the pulse is low

Введення внутрiшньовенно лiдокаїну Intravenous administration of lidocaine

Введення внутрiшньосерцево адреналiну Introduction of intracardiac adrenaline

Через стравохiдну кардiостимуляцiю 6- Вагуснi проби Via stravohid pacing 6- Vagus samples

Електроiмпульсну терапiю Electropulse therapy

сarotis вiдсутнiй, на ЕКГ - фiбриляцiя шлуночкiв. Яких заходiв буде найбiльш доцiльно вжити? there is no heartbeat, on the ECG - ventricular fibrillation. What measures would be most expedient to take?

30 / 200
Студент 2 днi тому прилетiв з Iндiї. Його госпiталiзовано в тяжкому станi: голос беззвучний, акроцiаноз, загостренi риси обличчя, ”руки прачки”, язик сухий, склери тьмянi. Живiт втягнутий. Генералiзованi судоми м’язiв. Пульс ниткоподiбний, артерiальний тиск не визначається. Яка невiдкладна допомога необхiдна хворому? The student flew in from India 2 days ago. He was hospitalized in a serious condition: voice is silent, acrocyanosis, sharpened facial features, 'washerman's hands', dry tongue, dim sclera Abdomen is drawn in. Generalized muscle spasms. The pulse is thread-like, blood pressure is not determined. What kind of emergency care does the patient need?

Внутрiшньовенна регiдратацiя Intravenous rehydration

Дегiдратацiйна терапiя Dehydration Therapy

Дезiнтоксикацiйна терапiя Detoxification Therapy

Ентеросорбцiя Enterosorption

Оральна регiдратацiя Oral Regidration

31 / 200
Хворий 58-ми рокiв скаржиться на безсоння, неспокiй, кровоточивiсть ясен, збiльшення живота. Хворiє10 рокiв на цироз печiнки. Об’єктивно: хворий збуджений, шкiра жовтушна, множиннi петехiї та судиннi зiрочки, ”печiнковий” запах з рота. Рiвень сечовини кровi - 138 мкмоль/л, загальний бiлок - 54 г/л, альбумiни - 22 г/л, холестерин 2,3 ммоль/л, бiлiрубiн - 48,5 мкмоль/л, протромб. iндекс - 61%, фiбриноген - 1,7 г/л. Чим ускладнилося захворювання? A 58-year-old patient complains of insomnia, restlessness, bleeding gums, abdominal distension. He has been suffering from cirrhosis of the liver for 10 years. Objectively: the patient is agitated, the skin is yellowish, multiple petechiae and vascular stars, 'hepatic' odor from the mouth. Blood urea level - 138 μmol/l, total protein - 54 g/l, albumin - 22 g/l, cholesterol 2.3 mmol/l, bilirubin - 48.5 μmol/l, prothrombin index - 61%, fibrinogen - 1.7 g/l. What complicated the disease?

Кровотечею з варикозно розширених вен стравоходу Bleeding from varicose veins of the esophagus

Гепатоцелюлярною недостатнiстю Hepatocellular insufficiency

Портальною гiпертензiєю Portal hypertension

Астено-невротичним синдромом Astheno-Neurotic Syndrome

Гострою нирковою недостатнiстю Acute kidney failure

32 / 200
Пiд час виконання сiльськогосподарських робiт молодий тракторист отримав вiдкритий перелом стегна в його нижнiй третинi. Вiдзначається деформацiя стегна, патологiчна рухомiсть, крепiтацiя кiсткових уламкiв. З рани фонтануєкров. На якому рiвнi потрiбно застосувати кровоспинний джгут? During agricultural work, a young tractor driver suffered an open hip fracture in the lower third of the thigh. Deformation of the hip, pathological mobility, crepitation of bone fragments is noted. Blood gushes from the wound. On which Do you even need to apply a tourniquet?

Артерiальний джгут в середнiй третинi стегна Arterial tourniquet in the middle third of the thigh

Венозний джгут безпосередньо над раною стегна Venous tourniquet directly above thigh wound

Артерiальний джгут безпосередньо над раною стегна Arterial tourniquet directly above the thigh wound

Артерiальний джгут в верхнiй третинi стегна Arterial tourniquet in the upper third of the thigh

Венозний джгут на верхню третину гомiлки Venous tourniquet on the upper third of the leg

33 / 200
До стацiонару надiйшов хворий на неспецифiчний виразковий колiт зi скаргами на гострий бiль у животi, здуття, криваву дiарею, пiдвищення температури тiла до 39oС На другу добу перебування у стацiонарi на тлi зростання симптомiв iнтоксикацiї частота випорожнень значно зменшилась. На оглядовiй рентгенограмi: збiльшення товстої кишки понад 6 см, перемiжнi острiвцi виразок слизової оболонки. Яке ускладнення зумовило погiршення стану хворого? A patient with nonspecific ulcerative colitis was admitted to the hospital with complaints of acute abdominal pain, bloating, bloody diarrhea, an increase in body temperature to 39oC. On the second day of hospitalization, Against the background of increasing symptoms of intoxication, the frequency of bowel movements decreased significantly. On the X-ray examination: enlargement of the large intestine by more than 6 cm, interspersed islands of ulcers of the mucous membrane. What complication caused the deterioration of the patient's condition?

Стриктура товстої кишки Colon stricture

Тромбоз мезентерiальних артерiй Thrombosis of mesenteric arteries

Токсична дилатацiя товстої кишки Toxic colon dilatation

Рак кишечника Intestinal Cancer

Перфорацiя товстої кишки Perforation of the colon

34 / 200
У дитини з гострою кишковою iнфекцiєю розвинулася клiнiка парезу кишечника Крок 3 Загальна лiкарська пiдготовка (україномовний варiант) 2018 рiк, весна 5 живiт здутий, гази вiдходять погано, кишковi шуми ледве прослуховуються. Введення якого препарату маєпершочергове значення? In a child with an acute intestinal infection, intestinal paresis developed. Step 3 General medical training (Ukrainian version) 2018, spring 5, the stomach is distended, gas is not passed, intestinal noises can barely be heard. Which drug should be administered first?

Цефазолiн Cefazolin

Альбумiн Albumin

Неогемодез Неогемодез

Глюкоза Glucose

Хлорид калiю Potassium Chloride

35 / 200
Через годину пiсля пологiв у породiллi почалася рясна кровотеча, яка склала 1 100 мл та продовжувалася, артерiальний тиск 80/60 мм рт.ст., пульс до 120/хв., центральний венозний тиск - 60 мм вод.ст. В аналiзi кровi: тромбоцитопенiя, анемiя. Дiагноз: геморагiчний шок. Якi показники необхiдно визначити щодо встановлення його ступеня? An hour after giving birth, the woman in labor began to bleed profusely, amounting to 1,100 ml and continuing, blood pressure 80/60 mm Hg, pulse up to 120/ min., central venous pressure - 60 mm Hg. Blood analysis: thrombocytopenia, anemia. Diagnosis: hemorrhagic shock. What indicators should be determined to determine its degree?

Загальний аналiз сечi General urine analysis

Шоковий iндекс Shock Index

Коагулограма Coagulogram

Загальний аналiз кровi General blood analysis

ЕКГ ЕКГ

36 / 200
Рядовий 19-ти рокiв помилково зробив 2 ковтки антифризу, пiсля чого з’явилися запаморочення, слабкiсть, похитування при ходi. Викликав блювання, випив розчин питної соди. У ПМП промили шлунок. Об’єктивно: стан задовiльний, свiдомiсть не порушена, обличчя гiперемоване, iн’єкцiя склер. Пальценосову пробу виконуєневпевнено. У легенях подих везикулярний. Межi серця не змiненi, дiяльнiсть серця ритмiчна, тони збереженi. Частота серцевих скорочень - 90/хв., артерiальний тиск - 100/60 мм рт.ст. Живiт м’який, трохи болючий у епiгастрiї. Печiнка бiля краю реберної дуги. Набрякiв нема. Дiурез - 500 мл/добу. Який обсяг допомоги в ОМедБ? A 19-year-old private mistakenly took 2 sips of antifreeze, after which he felt dizzy, weak, and staggered when walking. He induced vomiting and drank a solution of baking soda. The stomach was washed with PMP. Objectively: the condition is satisfactory, consciousness is not disturbed, the face is hyper-convulsive, scleral injection is performed. The finger-nose test is performed uncertainly. There is vesicular breath in the lungs. The borders of the heart have not changed, the heart's activity is rhythmic, the tones are preserved. Heart rate - 90/ min., blood pressure - 100/60 mm Hg. The abdomen is soft, a little painful in the epigastrium. The liver is near the edge of the costal arch. There is no swelling. Diuresis - 500 ml/day. What is the amount of help in OMedB?

Преднiзолон в/м Prednisone IM

Атропiн, фуросемiд Atropine, furosemide

Етиловий спирт в/в, фуросемiд в/в Ethyl alcohol IV, furosemide IV

Сольове проносне, етаперазин в/м Saline laxative, stageperazine IV

Строфантин в/в Strophanthin IV

37 / 200
Вагiтна у термiнi 34 тижнi доставлена своїм транспортом зi скаргами на сильний головний бiль, погiршення зору, слуху, нудоту, бiль в епiгастрiї. Об’єктивно: анасарка, акроцiаноз, артерiальний тиск - 190/100 мм рт.ст., пульс - 78/хв., ригiднiсть м’язiв потилицi. Яка тактика ведення вагiтної на даному етапi? A 34-week pregnant woman was delivered by her own transport with complaints of a severe headache, deterioration of vision, hearing, nausea, pain in the epigastrium. Objectively: anasarca, acrocyanosis , blood pressure - 190/100 mm Hg, pulse - 78/min, stiffness of the muscles of the back of the head. What are the tactics of managing a pregnant woman at this stage?

Утеротонiчна терапiя Uterotonic therapy

Сечогiннi препарати Diuretics

Гiпотензивна терапiя Antihypertensive therapy

Термiновий кесарський розтин Urgent caesarean section

Протисудомна терапiя Anticonvulsant therapy

38 / 200
Хвора 25-ти рокiв вiдчула себе погано в примiщеннi аптеки. Стоїть, спираючись на пiдвiконня з фiксованим плечовим поясом. Об’єктивно: обличчя одутле, вдих короткий, видих подовжений, частота дихання - 22/хв., частота серцевих скорочень - 100/хв., артерiальний тиск - 100/70 мм рт.ст., над легенями при перкусiї коробковий вiдтiнок перкуторного звуку. При аускультацiї - дихання везикулярне, жорстке, сухi, свистячi хрипи, бiльше на видиху. Cor - акцент II тону над легеневою артерiєю. Живiт без особливостей. Попереднiй дiагноз: A 25-year-old patient felt unwell in the pharmacy. She is standing, leaning on the windowsill with a fixed shoulder belt. Objectively: the face is swollen, the breath is short, the breath is prolonged, respiratory rate - 22/min., heart rate - 100/min., blood pressure - 100/70 mm Hg, over the lungs on percussion, a box tone of percussion sound. On auscultation - breathing is vesicular, hard, dry, whistling wheezes, more on exhalation. Cor - accent of the II tone over the pulmonary artery. Lives without features. Previous diagnosis:

ХОЗЛ ХОЗЛ

Тромбоемболiя легеневої артерiї Thromboembolism of the pulmonary artery

Гострий ларинготрахеїт Acute laryngotracheitis

Бронхiальна астма Bronchial asthma

Трахеобронхiальна дискiнезiя Tracheobronchial dyskinesia

39 / 200
Дiвчинка 10-ти рокiв пiд час шкiльної лiнiйки знепритомнiла. Пiд час огляду шкiра блiда, кiнцiвки холоднi. Тони серця послабленi. Частота серцевих скорочень - 62/хв., артерiальний тиск - 90/50 мм рт.ст. З чого необхiдно розпочати надання допомоги на догоспiтальному етапi? A 10-year-old girl fainted during the school emergency. During the examination, the skin is pale, the extremities are cold. The heart sounds are weakened. The heart rate is 62/min., arterial pressure - 90/50 mm Hg. What is the need to start providing care at the pre-hospital stage?

Звiльнити вiд стискаючого одягу Relieve from tight clothing

Забезпечити доступ свiжого повiтря Ensure access to fresh air

Покласти теплу грiлку до нiг Put a warm heating pad at your feet

Дати теплий чай Give warm tea

Покласти дитину на спину з припiднятими ногами Put the child on his back with his legs up

40 / 200
Лiкарем швидкої медичної допомоги у хворого констатованi ознаки клiнiчної смертi, розпочато реанiмацiйнi заходи. Для введення лiкарських засобiв обрана пункцiя крiкотиреоїдної зв’язки та ендотрахеальне введення. Як змiнюється доза препаратiв для цього шляху введення? The emergency medical doctor diagnosed the patient with signs of clinical death, resuscitation measures were started. For the administration of drugs, a puncture of the cricothyroid ligament and endotracheal administration were chosen. How the dose of drugs changes for this input path?

Збiльшується в 6 разiв Increases by 6 times

Зменшує ться в 2 рази Decreases by 2 times

Збiльшується в 4 рази Increases by 4 times

Збiльшується в 2 рази Increases by 2 times

Залишається незмiнною Remains unchanged

41 / 200
Жiнка 49-ти рокiв на прийомi у сiмейного лiкаря втратила свiдомiсть. Пiд час огляду непритомна, пульс на сонних артерiях не визначається, дихання поверхневе. Розпочато реанiмацiйнi процедури. Як слiд розташувати долонi пiд час проведення зовнiшнього масажу серця? A 49-year-old woman lost consciousness during an appointment with a family doctor. During the examination, she was unconscious, the pulse on the carotid arteries was not detected, her breathing was shallow. Resuscitation procedures were started. How should you place your palms during external heart massage?

На межi верхньої та середньої третин груднини On the border of the upper and middle thirds of the sternum

На межi середньої та нижньої третин груднини On the border of the middle and lower thirds of the sternum

На рукiв’ї груднини On the breastbone sleeve

На мечоподiбному вiдростку On a sword-like teenager

На верхнiй третинi груднини On the upper third of the sternum

42 / 200
У 19-ти рiчної жiнки у III перiодi пологiв проведене ручне вiдокремлення та видалення послiду в зв’язку з iнтимним прирощенням плаценти. Одразу пiсля пологiв з’явився гострий плевральний бiль, занепокоєння, лихоманка, кашель. Шкiрнi покриви блiдi, температура - 38,9oC, частота пульсу - 98/хв., артерiальний тиск - 100/70; 90/60 мм рт.ст., розпочалась маткова кровотеча, проте матка контурується. На ЕКГ - синусова тахiкардiя, гостре легеневе серце [SIQ III; P pulmonale], парцiальний тиск O2 становить 64 мм рт.ст., тиск CO2 - 18 мм рт.ст. Встановiть дiагноз: Крок 3 Загальна лiкарська пiдготовка ( україномовний варiант) 2018 рiк, весна 6 A 19-year-old woman in the 3rd stage of labor underwent manual separation and removal of the litter in connection with intimate growth of the placenta. Immediately after delivery, an acute pleural effusion appeared pain, anxiety, fever, cough, pale skin, temperature - 38.9oC, pulse rate - 98/min, blood pressure - 100/70, 90/60 mm Hg, uterine bleeding has begun, but the uterus is contoured. On the ECG - sinus tachycardia, acute pulmonary heart [SIQ ​​III; P pulmonale], the partial pressure of O2 is 64 mm Hg, the CO2 pressure is 18 mm Hg Make a diagnosis: Step 3 General medical training (Ukrainian version) 2018 year, spring 6

Повiтряна емболiя Air embolism

Iнфаркт мiокарду Myocardial infarction

Синдром Мендельсона Mendelssohn Syndrome

Тромбоемболiя легеневої артерiї Thromboembolism of the pulmonary artery

Емболiя амнiотичною рiдиною Amniotic fluid embolism

43 / 200
Хвора 19-ти рокiв надiйшла на 4-й день хвороби. У пацiєнтки блювання, рiзка слабкiсть, сонливiсть, виражена жовтяниця. Через добу хвора не орiєнтується в часi, свiдомiсть потьмарена, перiодичне психомоторне збудження. Зменшилися розмiри печiнки. Яке ускладнення зумовило таку клiнiчну картину? A 19-year-old patient arrived on the 4th day of illness. The patient has vomiting, severe weakness, drowsiness, pronounced jaundice. A day later, the patient does not orientate in time, consciousness is dimmed, periodic psychomotor agitation. The size of the liver has decreased. What complication caused such a clinical picture?

Гостра надниркова недостатнiсть Acute adrenal insufficiency

Гiповолемiчний шок Hypovolemic Shock

Механiчна жовтяниця Mechanical jaundice

Гостра печiнкова колька Gostra Pechinkova Kolka

Гостра печiнкова недостатнiсть Acute liver failure

44 / 200
До порту прибуваєтеплохiд iз Заїру, за даними командира екiпажу на борту перебувають 4 хворих з пiдозрою на гарячку Ебола. Вкажiть термiн спостереження за особами, що спiлкувалися iз хворими: A heat tanker from Zaire arrives at the port, according to the crew commander, there are 4 patients with suspected Ebola fever on board. Specify the period of observation of the persons who communicated with the patients:

15 дiб 15 дiб

5 дiб 5 дiб

10 дiб 10 дiб

6 дiб 6 дiб

21 доба 21 доба

45 / 200
Хворий iз серцево-судинною патологiєю потрапив в реанiмацiйне вiддiлення з пiдозрою на iнфаркт нирки. У нього пiдвищена температура - 39,5oC, гострий бiль у попереку та гематурiя. Артерiальний тиск - 180/110 мм рт.ст. Зроблено загальний аналiз кровi, сечi, та бiохiмiчне дослiдження кровi. Вкажiть найбiльш характернi ознаки iнфаркту нирки iз перерахованих: A patient with cardiovascular pathology was admitted to the intensive care unit with a suspected kidney infarction. He has an elevated temperature of 39.5oC, acute pain in the lower back, and hematuria. Arterial pressure - 180/110 mm Hg. A general analysis of blood, urine, and a biochemical blood test was performed. Indicate the most characteristic signs of a kidney infarction from the following:

Цилiндрурiя Cylindruria

Лейкоцитурiя Leukocyturia

Протеїнурiя Proteinuria

Бiль i гематурiя Bil i hematuria

Пiдвищена швидкiсть осiдання еритроцитiв Increased erythrocyte sedimentation rate

46 / 200
У 44-рiчного чоловiка пiсля вживання сурогатiв алкоголю виникло багаторазове блювання шлунковим вмiстом та дворазове блювання малозмiненою кров’ю. Пульс 100/хв. Артерiальний тиск - 100/70 мм рт.ст. Гемоглобiн - 110 Г/л. При ендоскопiчному обстеженнi у кардiальному вiддiлi шлунка виявлена лiнiйна трiщина слизової оболонки. Який дiагноз можна встановити на основi клiнiки й ендоскопiчних даних? After consuming alcohol substitutes, a 44-year-old man had repeated vomiting of stomach contents and two vomiting of blood with little change. Pulse 100/min. Blood pressure - 100/70 mm Hg. Hemoglobin - 110 G/l. During the endoscopic examination in the cardiac department of the stomach, a linear crack of the mucous membrane was detected. What diagnosis can be established on the basis of clinical and endoscopic data?

Грижа стравохiдного отвору дiафрагми Hernia of the esophageal orifice of the diaphragm

Ерозивний гастрит, ускладнений кровотечею Erosive gastritis complicated by bleeding

Синдром Меллорi-Вейса Mellori-Weiss Syndrome

Варикозне розширення вен кардiального вiддiлу шлунка, кровотеча Varicose dilatation of the cardiac department of the stomach, bleeding

Гостра виразка шлунка, кровотеча Acute gastric ulcer, bleeding

47 / 200
Пiдлiток 14-ти рокiв перебуваєна диспансерному облiку з приводу артерiальної гiпертензiї. Пiсля емоцiйного стресу у хворого розвинулася гiпертензивна криза. Артерiальний тиск - 170/100 мм рт.ст., частота серцевих скорочень - 50/хв. Який медикаментозний засiб Ви призначите? A 14-year-old teenager is under medical examination for arterial hypertension. After emotional stress, the patient developed a hypertensive crisis. Blood pressure - 170/100 mm Hg. , heart rate - 50/min. What medication do you prescribe?

Дигоксин Digoxin

Кордiамiн Cordiamin

Каптоприл Captopril

Панангiн Panangin

Корвiтол Корвiтол

48 / 200
Чоловiк 20-ти рокiв опинився пiд завалом. Протягом 6 годин гомiлки були пiд фрагментом стiни. Стан потерпiлого важкий. Свiдомiсть вiдсутня, артерiальний тиск - 80/50 мм рт.ст. Яку невiдкладну допомогу треба надати хворому? A 20-year-old man found himself under a rubble. For 6 hours, his lower legs were under a fragment of the wall. The victim's condition is serious. He is unconscious, blood pressure - 80/50 mm Hg .art. What kind of emergency care should be given to the patient?

Дезiнтоксикацiйна терапiя Detoxification Therapy

Накласти джгут на стегна, iммобiлiзувати нижнi кiнцiвки шинами Крамера, ввести знеболювальнi, iнфузiйна терапiя Put a tourniquet on the hips, immobilize the lower extremities with Kramer splints, administer analgesics, infusion therapy

Накласти джгут на стегна, iммобiлiзувати нижнi кiнцiвки шинами Крамера Put a tourniquet on the hips, immobilize the lower extremities with Kramer splints

Знеболення Anesthesia

Iммобiлiзацiя кiнцiвок Immobilization of extremities

49 / 200
Учасник лiквiдацiї аварiї на АЕС направлений у стацiонар з дiагнозом ”променева хвороба”. Протягом 8 днiв перебування в стацiонарi не знайдено нiяких суттєвих вiдхилень вiд норми. Чи можна очiкувати прояви гострого радiацiйного ураження у майбутньому? Participant in liquidation of the accident at the NPP was sent to the hospital with a diagnosis of 'radiation sickness'. No significant deviations from the norm were found during the 8-day stay in the hospital. Can we expect manifestations acute radiation damage in the future?

Не очiкується Not expected

Так, протягом пiвроку Yes, for six months

Так, протягом першого мiсяця Yes, during the first month

Так, протягом двох мiсяцiв Yes, for two months

Так, протягом року Yes, within a year

50 / 200
У потерпiлого внаслiдок тривалого впливу високих концентрацiй чадного газу дiагностовано тяжкий ступiнь отруєння. Назвiть найбiльш ефективну схему проведення кисневої терапiї: The victim was diagnosed with a severe degree of poisoning as a result of prolonged exposure to high concentrations of carbon monoxide. Name the most effective oxygen therapy scheme:

100% кисень протягом 1 години 100% oxygen for 1 hour

60% киснево-повiтряна сумiш протягом 4 годин 60% oxygen-air mixture for 4 hours

Гiпербарична оксигенацiя Hyperbaric oxygenation

40% киснево-повiтряна сумiш протягом 6 годин 40% oxygen-air mixture for 6 hours

80% киснево-повiтряна сумiш протягом 2 годин 80% oxygen-air mixture for 2 hours

51 / 200
У жiнки 32-х рокiв стан пiсля операцiї кесаревого розтину. На третю добу вiдзначено збiльшення в об’ємi лiвої нижньої кiнцiвки, поява болю за ходом глибоких вен гомiлки i гiперемiя шкiри. Температура тiла - 37,8oC. Яких заходiв необхiдно вжити пiсля операцiї для профiлактики глибокого венозного тромбозу? Крок 3 Загальна лiкарська пiдготовка ( україномовний варiант) 2018 рiк, весна 7 The condition of a 32-year-old woman after a caesarean section operation. On the third day, an increase in the volume of the left lower extremity, the appearance of pain along the course of the deep veins of the lower leg and skin hyperemia. Body temperature - 37.8oC. What measures should be taken after surgery to prevent deep venous thrombosis? Step 3 General medical training (Ukrainian version) 2018, spring 7

Лiкування антибiотиками Treatment with antibiotics

Лiкування цитостатиками Treatment with cytostatics

Лiкування утеротонiчними препаратами Treatment with uterotonic drugs

Введення фраксипарину Introduction of Fraxiparin

Озонотерапiя Ozone Therapy

52 / 200
У дитини 12-ти рокiв на тлi неревматичного кардиту з’явилися занепокоєння, акроцiаноз, експiраторна задишка, бiль за грудниною, кiнцiвки холоднi, вологi, артерiальна гiпотонiя. Якi заходи необхiднi пiд час транспортування? A 12-year-old child developed anxiety, acrocyanosis, expiratory shortness of breath, pain behind the sternum, cold, moist extremities, arterial hypotension against the background of non-rheumatic carditis. measures necessary during transportation?

Горизонтальне положення з опущеним головним кiнцем Horizontal position with lowered head end

Положення на лiвому боцi Position on the left side

Ортопное Ортопное

Положення на правому боцi Position on the right side

Горизонтальне положення з пiднятим головним кiнцем Horizontal position with raised head end

53 / 200
У лiкарню доставлена група потерпiлих у результатi бiйки iз застосуванням холодної i вогнепальної зброї. Якi поранення найбiльш несприятливi для прогнозу розвитку iнфекцiйних ускладнень? A group of victims was brought to the hospital as a result of a fight with the use of cold and firearms. Which injuries are the most unfavorable for the prognosis of the development of infectious complications?

Колото-рiзанi Circle-cut

Рiзанi Рiзанi

Вогнепальнi Firearms

Проникнi Infiltrate

Рвано-рiзанi Torn and cut

54 / 200
У гуртожитку виявлено хворого з пiдозрою на чуму. 2 днi назад вiн повернувся з Iндiї. Яких профiлактичних заходiв треба вжити щодо контактних осiб? A patient with suspected plague was found in the dormitory. He returned from India 2 days ago. What preventive measures should be taken for contact persons?

Лабораторне обстеження Laboratory examination

Спостереження на дому 6 днiв Monitoring at home for 6 days

Вакцинацiя Vaccination

Термометрiя Temperature

Екстрена профiлактика тетрациклiном, iзоляцiя Emergency prophylaxis with tetracycline, isolation

55 / 200
Хворий 67-ми рокiв впродовж 15-ти рокiв страждаєна гiпертонiчну хворобу. Пiд час фiзичної роботи раптово з’явився сильний бiль у епiгастрiї, нудота, непритомнiсть, артерiальний тиск - 190/90 мм рт.ст., пульс 110/хв. При пальпацiї в епiгастрiї визначається болюче утворення, що пульсує. Який дiагноз єнайбiльш iмовiрний? A 67-year-old patient has been suffering from hypertension for 15 years. During physical work, severe pain in the epigastrium, nausea, fainting, blood pressure suddenly appeared - 190/90 mm Hg, pulse 110/min. Palpation reveals a painful, pulsating mass in the epigastrium. What is the most likely diagnosis?

Розшарувальна аневризма черевного вiддiлу аорти Laterating aneurysm of the abdominal aorta

Пiдпечiнковий абсцес Subhepatic abscess

Лiмфогранулематоз з ураженням черевних лiмфовузлiв Lymphogranulomatosis with damage to abdominal lymph nodes

Ракова пухлина шлунка Stomach cancer

Iнфаркт мiокарда, гастралгiчна форма Myocardial infarction, gastric form

56 / 200
Хворий надiйшов у лiкарню на третiй день захворювання з t - 39oC, головним болем, остудою. Захворювання почалося з появи папули у пахвиннiй дiлянцi. Наступного дня з’явився пухирець iз темним вмiстом, що потiм розкрився з утворенням болючої ранки з жовтуватим дном й болючим синьочервоним iнфiльтратом в пахвиннiй дiлянцi розмiром 4 х4 см. Яких заходiв необхiдно вжити в першу чергу? The patient came to the hospital on the third day of the disease with t - 39oC, headache, cold. The disease began with the appearance of a papule in the groin area. The next day, a blister appeared with dark contents, which then opened with the formation of a painful wound with a yellowish bottom and a painful blue-red infiltrate in the inguinal area measuring 4 x 4 cm. What measures should be taken in the first place?

Введення протиправцевої сироватки Tetanus serum administration

Первинна хiрургiчна обробка рани Primary surgical treatment of the wound

Введення жарознижувальних препаратiв Introduction of antipyretic drugs

Iзоляцiя хворого з протиепiдемiчними заходами Isolation of the patient with anti-epidemic measures

Накласти пов’язку на рану Put a bandage on the wound

57 / 200
Бригадою швидкої медичної допомоги (ШМД) транспортується потерпiлий з черепномозковою травмою: свiдомiсть вiдсутня, зiницi рiвномiрно звуженi, артерiальний тиск 100/60 мм рт.ст., дихання самостiйне, ефективне. Яке положення слiд надати потерпiлому пiд час транспортування до лiкувального закладу? A victim with a craniocerebral injury is being transported by the ambulance crew: he is unconscious, the pupils are uniformly narrowed, the blood pressure is 100/60 mmHg, he is breathing on his own , effective. What position should be given to the victim during transportation to a medical facility?

Горизонтальне на спинi з припiднятим головним кiнцем на 30o Horizontal on the back with the head end raised by 30o

Горизонтальне на спинi Horizontal on back

На правому боцi з опущеним головним кiнцем On the right side with the main end down

Горизонтальне на спинi з пiднятими ногами на 70o Horizontal on the back with legs raised at 70o

Горизонтальне на спинi з опущеним головним кiнцем Horizontal on the back with the head end down

58 / 200
У хлопчика 2-х рокiв на 5-й день дiареї (E.coli) та субфебрильної лихоманки з’явився поширений петехiальний висип, дiурез 50 мл на добу. Загальний аналiз кровi: гемоглобiн - 70 г/л, КП- 0,85, фрагментованi еритроцити, тромбоцити - 55 Г/л. Креатинiн - 240 мкМ/л. Найiмовiрнiшою причиною ниркової недостатностi є: On the 5th day of diarrhea (E.coli) and low-grade fever, a 2-year-old boy developed a widespread petechial rash, diuresis of 50 ml per day. General blood analysis: hemoglobin - 70 g/l, CP - 0.85, fragmented erythrocytes, platelets - 55 g/l. Creatinine - 240 μM/l. The most likely cause of kidney failure is:

Гломерулонефрит Glomerulonephritis

Гемолiтико-уремiчний синдром Hemolytic uremic syndrome

Геморагiчний васкулiт Hemorrhagic vasculitis

Гiповолемiя Hypovolemia

Iнтерстицiальний нефрит Interstitial Jade

59 / 200
Хвора 57-ми рокiв на пляжi вiдчула рiзкий головний бiль, виникло блювання. Хвора знепритомнiла. Об’єктивно: шкiра рiзко гiперемована. Артерiальний тиск - 200/130 мм рт.ст. Пульс - 98/хв., напружений. Дихання за типом Чейн-Стокса. Симптом ”вiтрила” справа. Права стопа ротована назовнi. Глибокi рефлекси не викликаються. Який симптом необхiдно перевiрити у хворої в непритомному станi для визначення сторони ураженої пiвкулi? A 57-year-old patient felt a sharp headache on the beach, vomiting occurred. The patient fainted. Objectively: the skin was sharply hyperemia. Blood pressure - 200/130 mm Hg. Pulse - 98/min., tense. Respiration according to the Cheyne-Stokes type. The symptom is 'wiped' on the right. The right foot is externally rotated. Deep reflexes are not elicited. What symptom should be checked in an unconscious patient to determine the side of the affected hemisphere ?

Адiадохокiнез Adiadochokinesis

Хиткiсть у позi Ромберга Romberg's Cunning

Ширину зiниць Width of pupils

Орального автоматизму Oral automatism

Холодову пробу Cold Test

60 / 200
Водiй, якого доставлено у приймальне вiддiлення пiсля автодорожньої катастрофи, у рентгенологiчному кабiнетi знепритомнiв. Шкiра обличчя та шиї цiанотична, серцевi тони не вислуховуються, артерiальний тиск не визначається, пiдшкiрнi вени шиї розширенi. На оглядовiй рентгенограмi ОГК легеневi поля не змiненi, серцева тiнь маєкулясту форму. Вкажiть найбiльш iмовiрну причину зупинки кровообiгу: Крок 3 Загальна лiкарська пiдготовка (україномовний варiант) 2018 рiк, весна 8 The driver, who was taken to the receiving department after a road accident, fainted in the x-ray room. The skin of the face and neck is cyanotic, heart sounds are not heard, blood pressure is not determined, subcutaneous the veins of the neck are dilated. On the X-ray of the OGK, the lung fields are unchanged, the cardiac shadow has a spherical shape. Indicate the most likely cause of the stoppage of blood circulation: Step 3 General medical training (Ukrainian-language version) 2018, spring 8

Напружена емфiзема середостiння Tense mediastinal emphysema

Тампонада серця Cardiac tamponade

Гемомедiастинум Hemomediastinum

Тромбоемболiя легеневої артерiї Thromboembolism of the pulmonary artery

Геморагiчний шок Hemorrhagic shock

61 / 200
У пiдлiтка 16-ти рокiв 4 мiсяцi тому дiагностовано туберкульоз легенiв. Пiсля лiкування впродовж 2-х мiсяцiв дитина самостiйно перервала прийом протитуберкульозних препаратiв. У зв’язку з погiршенням стану було повторно призначене лiкування. Через 2 тижнi з’явились втрата рiвноваги, зниження слуху, дзвiн у вухах. Прийом якого препарату зумовив появу таких симптомiв i потребуєвiдмiни? A 16-year-old teenager was diagnosed with pulmonary tuberculosis 4 months ago. After treatment for 2 months, the child independently stopped taking anti-tuberculosis drugs. Due to worsening treatment was prescribed again for the condition. After 2 weeks, loss of balance, hearing loss, and ringing in the ears appeared. Which drug caused the appearance of such symptoms and needs to be discontinued?

Пiразiнамiд Pyrazinamide

Етамбутол Ethambutol

Стрептомiцин Streptomycin

Iзонiазiд Iзонiазiд

Рiфампiцин Рiфампiцин

62 / 200
14- рiчний хлопчик на тлi високої температури тiла раптом починаєповодитися неадекватно, спостерiгається агресивна поведiнка, викрикуєнецензурнi слова. З’являються епiзоди клонiко-тонiчних судом. Яка причина такого стану єнайбiльш iмовiрною? A 14-year-old boy against a background of high body temperature suddenly begins to behave inappropriately, aggressive behavior is observed, shouts obscene words. Episodes of clonic-tonic seizures appear. What is the cause of this condition is the most likely?

Афективно-респiраторнi напади Affective-respiratory attacks

Iстерична реакцiя Hysterical reaction

Фебрильнi судоми Febrile convulsions

Пневмонiя Pneumonia

Енцефалiт Encephalitis

63 / 200
Хворий 40-ка рокiв загальмований, часом агресивний, на питання вiдповiдаєнеадекватно. Вiдомо, що тривалий час зловживаєалкоголем. Шкiра та слизовi оболонки жовтi. Живiт збiльшений у розмiрi, на переднiй черевнiй стiнцi розширенi вени у виглядi голови медузи. Печiнка +4 см, край щiльний, болючий, гладенький. Перкуторно: збiльшення меж селезiнки та притуплення у спадистих мiсцях. Назвiть найбiльш iнформативний метод для дiагностики печiнкової недостатностi: A 40-year-old patient is inhibited, sometimes aggressive, answers questions inadequately. It is known that he has been abusing alcohol for a long time. The skin and mucous membranes are yellow. The abdomen is enlarged, on the front abdominal wall has dilated veins in the form of a jellyfish's head. Liver +4 cm, the edge is dense, painful, smooth. Percussion: enlargement of the spleen borders and dullness in receding areas. Name the most informative method for diagnosing liver failure:

Гастроскопiя Gastroscopy

Сонографiя Sonography

Бiохiмiчнi дослiдження кровi Biochemical blood tests

Комп’ютерна томографiя Computed tomography

Загальний аналiз кровi General blood analysis

64 / 200
Хворому 22-х рокiв з полiтравмою в реанiмацiйнiй залi проводиться серцеволегеневомозкова реанiмацiя. Який критерiй ефективностi закритого масажу серця має найбiльшу прогностичну цiннiсть? A 22-year-old patient with polytrauma is undergoing cardiopulmonary resuscitation in the intensive care unit. Which criterion of the effectiveness of closed heart massage has the greatest prognostic value?

Вiдновлення самостiйного дихання Restoration of independent breathing

Звуження зiниць Pupillary constriction

Наявнiсть пульсу на променевiй артерiї Presence of a pulse on the radial artery

Наявнiсть пульсацiї на соннiй артерiї Presence of pulsation on the carotid artery

Аускультативно вислуховуються тони серця Heart tones are auscultated

65 / 200
У хворого 32-х рокiв пiсля перебування у лiсi та контакту з мурахами з’явилися набряк суглобiв рук, блювання, бiль у шлунку, остуда, утруднене дихання, загальна слабкiсть. При оглядi хворого вiдзначено набряк обличчя, шкiрнi покриви гiперемованi. Над легенями вислуховуються жорстке дихання, сухi свистячi хрипи. Тахiкардiя. Артерiальний тиск - 70/40 мм рт.ст. Якi заходи невiдкладної допомоги необхiдно провести хворому? A 32-year-old patient developed swelling of the joints of the hands, vomiting, stomach pain, chills, difficulty breathing, general weakness. During the examination of the patient, swelling of the face is noted, the skin is hypertrophied. Harsh breathing, dry whistling rales are heard above the lungs. Tachycardia. Blood pressure - 70/40 mm Hg. What emergency measures should the patient take?

Призначення сорбентiв Application of sorbents

Введення адреналiну Injection of adrenaline

Введення антигiстамiнних препаратiв Introduction of antihistamines

Введення серцевих глiкозидiв Introduction of cardiac glycosides

Введення кордiамiну Cordiamine input

66 / 200
Солдат 20-ти рокiв на полi бою отримав ураження у виглядi термiчного опiку обох кистей. Об’єктивно: загальний стан задовiльний частота дихальних рухiв - 16/хв., дихання везикулярне. Частота серцевих скорочень - 78/хв. Дiяльнiсть серця ритмiчна. На тильнiй поверхнi долонь пухирi з блiдорожевим вмiстом. З чого необхiдно розпочати невiдкладну допомогу? A 20-year-old soldier suffered a thermal burn on both hands on the battlefield. Objectively: the general condition is satisfactory, the respiratory rate is 16/min. breathing is vesicular. Heart rate - 78/min. Heart activity is rhythmic. On the back surface of the palms, there are blisters with pale pink contents. What is the need to start emergency care?

Промити перекисом водню Rinse with hydrogen peroxide

Промити холодною водою Rinse with cold water

Ввести антигiстамiннi препарати Enter antihistamines

Ввести знеболювальнi препарати Enter painkillers

Накласти асептичну пов’язку Apply an aseptic bandage

67 / 200
Хворий 40-ка рокiв доставлений швидкою медичною допомогою, свiдомiсть потьмарена, блiдий, вкритий холодним потом. Частота дихальних рухiв - 28/хв., артерiальний тиск 80 /50 мм рт.ст., тони серця приглушенi, живiт при пальпацiї болiсний в епiгастрiї, в анамнезi блювання кавовою гущею. Якi першочерговi заходи невiдкладної допомоги? A 40-year-old patient was delivered by ambulance, his consciousness is dimmed, he is pale, covered in cold sweat. The frequency of respiratory movements is 28/min., blood pressure 80/50 mm Hg, the heart sounds are muffled, the abdomen is painful on palpation in the epigastrium, there is a history of vomiting with coffee grounds. What are the primary emergency measures?

Введення довенно кристалоїдних розчинiв Introduction of permanent crystalloid solutions

Введення довенно гепарину Introduction of intravenous heparin

Введення довенно розчину глюкози Introduction of intravenous glucose solution

Введення довенно атропiну Introduction of intravenous atropine

Введення довенно адреналiну Injection of intravenous adrenaline

68 / 200
Жiнка 30-ти рокiв швидкою допомогою доставлена у непритомному станi з мiсця автоаварiї. Об’єктивно: свiдомiсть вiдсутня, на больовi подразники або спонтанно з’являється тонiчне напруження м’язiв з випростуванням рук i нiг та закиданням голови назад, двостороннi стопнi патологiчнi рефлекси. Дихання Чейн-Стокса, пульс - 120/хв., артерiальний тиск - 140/90 мм рт.ст., температура - 37,9o A 30-year-old woman was brought by ambulance in an unconscious state from the scene of a car accident. Objectively: consciousness is absent, tonic tension appears spontaneously on painful stimuli or 'joints with arms and legs straightened and head thrown back, bilateral foot pathological reflexes. Cheyne-Stokes breathing, pulse - 120/min., blood pressure - 140/90 mm Hg, temperature - 37.9o

Назвiть найбiльш iмовiрний дiагноз: Name the most likely diagnosis:

Забiй головного мозку важкого ступеня Severe brain injury

Внутрiшньочерепна гематома F. Менiнгiт Intracranial hematoma F. Meningitis

Гостре порушення мозкового кровообiгу Acute violation of cerebral circulation

Аневризма судин головного мозку Aneurysm of cerebral vessels

69 / 200
У чоловiка 26-ти рокiв в нижнiй третинi лiвого стегна єрана 7х3 см неправильної форми з рваними краями та помiрною кровотечею. Близько 20-ти хвилин тому хворого вкусив бродячий пес. Якою буде перша Крок 3 Загальна лiкарська пiдготовка (україномовний варiант) 2018 рiк, весна 9 допомога? A 26-year-old man has a 7x3 cm irregularly shaped wound with ragged edges and moderate bleeding in the lower third of his left thigh. About 20 minutes ago, the patient was bitten by a stray dog What will be the first Step 3 General medical training (Ukrainian-language version) 2018, spring 9 help?

Накладання стискальної пов’язки. Рану не зашивати. Госпiталiзацiя в хiрургiчний стацiонар для проведення антирабiчної профiлактики Applying a compression bandage. The wound should not be sutured. Hospitalization in a surgical hospital for anti-rabies prophylaxis

ПХО. Рану зашити. Госпiталiзацiя в хiрургiчний стацiонар для проведення антирабiчної профiлактики PHO. Suture the wound. Hospitalization in a surgical hospital for anti-rabies prophylaxis

ПХО. Рану зашити. Амбулаторне спостереження хiрурга та рабiолога PHO. Sew up the wound. Outpatient observation by a surgeon and a radiologist

- -

ПХО. Рану не зашивати. Амбулаторне спостереження хiрурга та рабiолога PHO. The wound should not be sutured. Outpatient observation by a surgeon and a radiologist

70 / 200
Хвора 72-х рокiв скаржиться на запаморочення з нудотою i блюванням. В ходi неврологiчного дослiдження виявленi горизонтальний нiстагм влiво, лiвобiчна гемiатаксiя, зниження тонусу лiвих кiнцiвок. Яка локалiзацiя патологiчного процесу? A 72-year-old patient complains of dizziness with nausea and vomiting. During the neurological examination, horizontal nystagmus to the left, left-sided hemiataxia, decreased tone of the left extremities were found. What is the localization of the pathological process?

Мозочок Cerebellum

Потилична частка Occipital lobe

Скронева частка Skroneva

Стовбур головного мозку Brain stem

Лобова частка Frontal lobe

71 / 200
У хворого 33-х рокiв пiсля введення анальгiну з’явилась слабкiсть, шум у вухах, почуття страху, серцебиття. На шкiрi обличчя - уртикарнi висипання. Шкiра вкрита холодним, липким потом. Артерiальний тиск - 50/20 мм рт.ст. Пульс слабкий, ниткоподiбний, його не можливо пiдрахувати. Через 15 хвилин з’явилися здуття живота, тиск у епiгастральнiй областi, нудота, блювання. Який найбiльш iмовiрний дiагноз у даного хворого? A 33-year-old patient developed weakness, ringing in the ears, a feeling of fear, palpitations after the injection of analgin. He has urticarial rashes on the face. The skin is covered with cold , sticky sweat. Blood pressure - 50/20 mm Hg. The pulse is weak, thread-like, it cannot be counted. After 15 minutes, abdominal distension, pressure in the epigastric region, nausea, vomiting appeared. What is the most likely diagnosis for this patient ?

Гострий панкреатит Hostry Pancreatitis

Анафiлактичний шок Anaphylactic shock

Гостра кишкова непрохiднiсть Acute intestinal obstruction

Геморагiчний шок Hemorrhagic shock

Прободна виразка шлунка Perforated gastric ulcer

72 / 200
Хвора 40-ка рокiв, iнженер, маєскарги на пароксизми гострого болю в лiвiй половинi обличчя, тривалiстю 1-2 хвилини. Частота нападiв до 10 разiв на добу. Провокуються напади болю вмиванням, вживанням їжi, розмовами, дотиком до лiвої щоки (”куркова зона”). Хворобливiсть при пальпацiї супраорбiтальної точки злiва. Сформулюйте клiнiчний дiагноз: A 40-year-old patient, an engineer, complains of paroxysms of sharp pain in the left side of the face, lasting 1-2 minutes. The frequency of attacks is up to 10 times a day. They are provoked pain attacks when washing, eating, talking, touching the left cheek ('chicken zone'). Soreness when palpating the supraorbital point on the left. Formulate a clinical diagnosis:

Невралгiя трiйчастого нерва Triparticular Neuralgia

Невралгiя великого потиличного нерву Neuralgia of the large occipital nerve

Пухлина головного мозку Brain tumor

Лицева симпаталгiя Facial Sympathy

Неврит лицевого нерву Neuritis of the facial nerve

73 / 200
У дитини 1,5 року з ознаками гострої респiраторної iнфекцiї ввечерi з’явилася iнспiраторна задишка, ”гавкаючий” кашель, захриплiсть голосу. При оглядi: дихання стенотичне, iнспiраторна задуха з участю допомiжної дихальної мускулатури, втягування поступливих дiлянок грудної клiтки. При аускультацiї дихання у легенях дифузно послаблене. Тахiкардiя, випадання пульсової хвилi на вдиху. Поставте дiагноз: A 1.5-year-old child with signs of an acute respiratory infection developed inspiratory dyspnea, a 'barking' cough, hoarseness in the evening. On examination: breathing is stenotic, inspiratory suffocation with the involvement of the auxiliary respiratory muscles, retraction of compliant areas of the chest. During auscultation, breathing in the lungs is diffusely weakened. Tachycardia, loss of the pulse wave during inhalation. Make a diagnosis:

Дифтерiйний круп Diphtheria

Вiрусний круп Viral Croup

Стороннєтiло дихальних шляхiв Foreign body of the respiratory tract

Обструктивний бронхiт Obstructive bronchitis

Пневмонiя Pneumonia

74 / 200
Хворий 48-ми рокiв, працiвник хiмiчної лабораторiї, доставлений в лiкарню зi скаргами на загальну слабкiсть, запаморочення, головний бiль, заплiтання язика при розмовi, блювання, кашель, дертя в горлi, кровотечу з носа, часте болюче сечовипускання. Об’єктивно: шкiра сiро-синього кольору, судоми м’язiв. Кров шоколадно-бура, вмiст MtHb у кровi - 60%, еритроцити з тiльцями ГейнцаЕрлiха - 25%. В сечi: протеїнурiя, макрогематурiя. Ваш попереднiй дiагноз: A 48-year-old patient, a chemical laboratory worker, was brought to the hospital with complaints of general weakness, dizziness, headache, entanglement of the tongue when speaking, vomiting, cough, scratchy throat, nosebleeds, frequent painful urination. Objectively: gray-blue skin, muscle spasms. Chocolate-brown blood, MtHb content in blood - 60%, erythrocytes with Heinz-Ehrlich bodies - 25%. In urine : proteinuria, macrohematuria. Your previous diagnosis:

Iнтоксикацiя марганцем Manganese poisoning

Iнтоксикацiя амiносполуками бензолу Intoxication with benzene amino compounds

Iнтоксикацiя ртуттю Mercury intoxication

Iнтоксикацiя тетраетилсвинцем Tetraethyl lead poisoning

Iнтоксикацiя сiрковуглецем Carbon disulfide intoxication

75 / 200
Хвора 36-ти рокiв працюєна хiмiчному заводi. Пiсля грубого порушення правил технiки безпеки та виникнення аварiйної ситуацiї, була госпiталiзована в стацiонар зi скаргами на пiдвищення температури тiла до 38oC, бiль у груднiй клiтцi, кашель з видiленням слизовогнiйного харкотиння з домiшками кровi, задишку. Об’єктивно: в нижнiх вiддiлах легень ослаблене везикулярне дихання, вислуховуються дрiбномiхурцевi вологi хрипи та крепiтацiя. Рентгенологiчно: вогнищево-iнфiльтративнi змiни в нижнiх вiддiлах легень. Поставте попереднiй дiагноз: The 36-year-old patient works at a chemical plant. After a gross violation of safety rules and an emergency situation, she was hospitalized with complaints of an increase in body temperature to 38oC, pain in the chest, cough with secretion of mucous sputum with blood impurities, shortness of breath. Objectively: vesicular breathing is weakened in the lower parts of the lungs, small alveolar moist rales and crepitations are heard. X-ray: focal infiltrative changes in the lower lobes of the lungs.

Гострий токсичний фарингiт Acute toxic pharyngitis

Гострий токсичний ларингiт Acute toxic laryngitis

Гострий токсичний бронхiт Acute toxic bronchitis

Гострий токсичний набряк легень Acute toxic pulmonary edema

Гострий токсичний пневмонiт Acute toxic pneumonitis

76 / 200
До iнфекцiйної лiкарнi лiкарем сiльської амбулаторiї направлений хворий 17-ти рокiв, який три днi тому був укушений коровою за кисть руки. При оглядi на мiсцi укусу визначається пухирець з кратероподiбним поглибленням. Навколо пустули - круглий вiнчик з пухирцiв iз серозно-кров’янистим вмiстом та регiональний лiмфаденiт. Що необхiдно зробити для уточнення дiагнозу? A 17-year-old patient who was bitten on the hand by a cow three days ago was sent to the infectious disease hospital by the doctor of the rural outpatient clinic. On examination, a blister with a crater-like deepening. Around the pustule there is a round crown of blisters with serous and bloody contents and regional lymphadenitis. What should be done to clarify the diagnosis?

Мiкроскопiя видiлень з рани Microscopy of secretions from the wound

Бактерiологiчне дослiдження вмiсту рани Bacteriological examination of wound contents

Загальний аналiз кровi General blood analysis

Рентгенолологiчне дослiдження кистi X-ray examination of the hand

Бактерiологiчне дослiдження кровi хворого Bacteriological examination of the patient's blood

77 / 200
У реанiмацiйне вiддiлення доставлений хлопчик 10-ти рокiв з гострим отруєнням свинцем. Млявий, шкiра блiда, артерiальний тиск - 110/55 мм рт.ст., ендогенний креатинiн - 1,098 ммоль/л, сечовина - 20,8 ммоль/л, калiй плазми - 7,9 ммоль/л, дiурез 200 мл/добу. Що слiд призначити першочергово? Крок 3 Загальна лiкарська пiдготовка ( україномовний варiант) 2018 рiк, весна 10 A 10-year-old boy with acute lead poisoning was brought to the intensive care unit. Lethargic, pale skin, blood pressure - 110/55 mm Hg, endogenous creatinine - 1.098 mmol/l, urea - 20.8 mmol/l, plasma potassium - 7.9 mmol/l, diuresis 200 ml/day. What should be prescribed as a priority? Step 3 General medical training (Ukrainian version) 2018, spring 10

Активоване вугiлля Activated carbon

Хелатоутворювачi Chelating Agents

Плазмаферез Plasmapheresis

Лазикс Лазикс

Ентеросгель Enterosgel

78 / 200
Мати залишила здорового сина 2,5 рокiв iз старшою дочкою, хворою на гостру респiраторну вiрусну iнфекцiю (ГРВI). Вона через 3 години повернулася додому i помiтила у сина частий кашель. На другий день стан його погiршився, температура тiла 38 oC, частота дихання - 42/хв., частота серцевих скорочень - 132/хв., цiаноз шкiри. Перкуторно: справа нижче кута лопатки вкорочення звуку, аускультативно в цьому мiсцi рiзке ослаблення дихання. На Rо ОГК ателектаз правих сегментiв S4 i S5, тiнь органiв середостiння змiщена вправо. Яка iмовiрна причина такого стану дитини? A mother left her healthy 2.5-year-old son with her eldest daughter, who is sick with an acute respiratory viral infection (ARVI). She returned home 3 hours later and noticed her son's frequent cough. On the second day, his condition worsened, body temperature 38 oC, respiratory rate - 42/min., heart rate - 132/min., cyanosis of the skin. Percussion: sound shortening on the right below the angle of the scapula, auscultation in this place sharp weakening of breathing . On Ro OGK, atelectasis of the right S4 and S5 segments, the shadow of the mediastinal organs is shifted to the right. What is the probable cause of this condition of the child?

Гостра респiраторна вiрусна iнфекцiя Acute respiratory viral infection

Стороннєтiло бронхiв Bronchi foreign body

Гострий обструктивний бронхiт Acute obstructive bronchitis

Гостра пневмонiя Acute pneumonia

Напружений пневмоторакс Tension pneumothorax

79 / 200
28- рiчна жiнка скаржиться на переймоподiбний бiль у нижнiх вiддiлах живота, кров’янистi видiлення з пiхви. Хвора блiда, пульс - 78/хв., артерiальний тиск - 110/70 мм рт.ст., температура - 36,6o. Пiд час бiмануального дослiдження: матка збiльшена до 6-7 тижнiв вагiтностi, болюча, плiдне яйце в цервiкальному каналi, видiлення з пiхви кров’янистi. Який найбiльш iмовiрний дiагноз? A 28-year-old woman complains of cramp-like pain in the lower abdomen, bloody discharge from the vagina. The patient is pale, pulse - 78/min, blood pressure - 110/70 mm Hg, temperature - 36.6 o. During bimanual examination: the uterus is enlarged to 6-7 weeks of pregnancy, painful, fertile egg in the cervical canal, vaginal discharge is bloody. What is the most likely diagnosis?

Апоплексiя яєчника Ovarian apoplexy

Позаматкова вагiтнiсть Ectopic pregnancy

Загроза викидня Threat of miscarriage

Аборт в ходу Abortion in progress

Неповний аборт Incomplete abortion

80 / 200
Вагiтна в термiнi 30 тижнiв надiйшла до пологового будинку у важкому станi. Скарги на головний бiль, блювання, мерехтiння мушок перед очима. Об’єктивно: анасарка, артерiальний тиск - 180/120 мм рт.ст., з’явилися фiбрилярнi посмикування м’язiв обличчя, тонiчнi судоми, дихання припинилося. Через 2 хвилини дихання вiдновилося. Амнезiя. Поставте дiагноз: A 30-week pregnant woman arrived at the maternity hospital in a critical condition. Complaints of headache, vomiting, flickering of flies in front of the eyes. Objectively: anasarca, blood pressure - 180/120 mm Hg, fibrillar twitching of facial muscles appeared, tonic convulsions, breathing stopped. Breathing resumed after 2 minutes. Amnesia. Make a diagnosis:

Гiпертонiчний криз Hypertensive crisis

Епiлепсiя Epilepsy

Еклампсiя Eclampsia

Пухлина мозку Brain tumor

Черепно-мозкова травма Tranio-cerebral injury

81 / 200
Хвора 62-х рокiв скаржиться на виражену слабкiсть, втомлюванiсть, набряки обличчя i нiг. Погiршення стану наростало протягом останнього року. У 55 рокiв перенесла субтотальну тиреоїдектомiю, 2 роки тому - iнфаркт мiокарда, з того часу зберiгається миготлива аритмiя. Змiна якого лабораторного показника вказуєна причину погiршення стану хворої? A 62-year-old patient complains of severe weakness, fatigue, swelling of the face and legs. The condition has worsened over the past year. At the age of 55, she underwent subtotal thyroidectomy, 2 years therefore - a myocardial infarction, since then the atrial fibrillation has persisted. A change in which laboratory indicator indicates the cause of the deterioration of the patient's condition?

КФК - 250 од/л CFC - 250 units/l

ТТГ - 12,0 мМЕ/л ТТГ - 12,0 мМЕ/л

Креатинiн - 95,3 мкмоль/л Creatinine - 95.3 µmol/L

Загальний холестерин - 8,7 ммоль/л Total cholesterol - 8.7 mmol/l

Протеїнурiя - 0,03 г/л Proteinuria - 0.03 g/l

82 / 200
До лiкаря психотерапевта звернулися батьки 14-рiчної дiвчинки у зв’язку з рiзкою змiною поведiнки доньки протягом останнiх 10-ти днiв. Вона стала замкненою, не хоче ходити до школи, увесь час сидить у своїй кiмнатi. Пiд час огляду дiвчинки лiкар звернув увагу на наявнiсть старих синцiв на руках, ногах, садна, що загоюються, на внутрiшнiй поверхнi стегон. Найбiльш можлива причина змiни поведiнки дiвчинки? The parents of a 14-year-old girl turned to a psychotherapist due to a sharp change in their daughter's behavior over the past 10 days. She has become withdrawn, does not want to go to of school, sits in her room all the time. During the examination of the girl, the doctor drew attention to the presence of old bruises on the hands, legs, and healing wounds on the inner surface of the thighs. What is the most possible reason for the change in the girl's behavior?

Нервова анорексiя Anorexia Nervosa

Конфлiкт з батьками Conflict with parents

Проблеми у школi School problems

Проблеми у особистих стосунках Problems in personal relationships

Зґвалтування Rape

83 / 200
Хвора 49-ти рокiв протягом останнього тижня пiсля стоматологiчної операцiї в зв’язку з остеомiєлiтом нижньої челюстi стала тривожною, плаксивою, порушився сон. Виказуєiдеї поганого до неї ставлення з боку близьких, мовляв, її засуджують, звинувачують у матерiальних негараздах сiм’ї. У вiддiленнi весь час ходить по палатi, заламує руки, голосить, ”краще б убили одразу”, ”так жити не хочу”. Який препарат слiд призначити в першу чергу хворiй? A 49-year-old patient during the last week after a dental operation in connection with osteomyelitis of the lower jaw became anxious, tearful, sleep was disturbed. She expresses ideas of bad attitude towards her from on the part of her relatives, they say that she is condemned, blamed for the family's financial troubles. In the ward, she walks around the ward all the time, wrings her hands, wails, 'I'd rather be killed immediately,' 'I don't want to live like this.' What drug should be prescribed first of all sick?

Галоперидол Haloperidol

Дiазепам Diazepam

Амiназин Aminazine

Амiтриптилiн Amitriptyline

Iмiзин Iмiзин

84 / 200
Хвора 16-ти рокiв скаржиться на iнтенсивний бiль у горлi злiва, що вiддаєу вухо, рiзке утруднення ковтання та вiдкриття рота, пiдвищення температури тiла до 39,2o. Хворiєтретю добу, за тиждень до початку скарг лiкувалася у дiльничного лiкаря з приводу ангiни. Вiдкриття рота значно утруднене (тризм жувальних м’язiв), асиметрiя зiву за рахунок випинання лiвого мигдалика, пiднебiнно-язикова дужка рiзко гiперемована, iнфiльтрована, набрякла. За кутом нижньої щелепи визначаються збiльшенi та болючi лiмфовузли. Який найбiльш iмовiрний дiагноз? A 16-year-old patient complains of an intense pain in the throat on the left side, radiating to the ear, severe difficulty in swallowing and opening the mouth, an increase in body temperature to 39.2o. The third patient day, a week before the onset of complaints, she was treated by a local doctor for angina. Opening the mouth is significantly difficult (trismus of the masticatory muscles), asymmetry of the throat due to the protrusion of the left tonsil, the palato-lingual arch is sharply hypererized, infiltrated, swollen. Behind the angle of the lower jaw enlarged and painful lymph nodes are detected. What is the most likely diagnosis?

Паратонзилярний абсцес Paratonsillar abscess

Ангiна Ангiна

Дифтерiя Diphtheria

Хронiчний тонзилiт Chronic tonsillitis

Фарингiт Faringit

85 / 200
Граючись на будiвельному майданчику, хлопчик 10-ти рокiв впав з висоти на бетонне покриття. Пiд час огляду дитина непритомна, дихальна екскурсiя грудної клiтки вiдсутня, пульс на соннiй артерiї не визначається, зiницi рiзко розширенi, у дитини наявний вiдкритий перелом кiсток гомiлки, численнi садна на видимих дiлянках шкiри, на губах свiжi слiди кровi. Яких заходiв невiдкладної Крок 3 Загальна лiкарська пiдготовка ( україномовний варiант) 2018 рiк, весна 11 допомоги потребуєпотерпiлий у першу чергу? While playing on a construction site, a 10-year-old boy fell from a height onto a concrete surface. During the examination, the child is unconscious, there is no respiratory excursion of the chest, the pulse is on the carotid artery cannot be determined, the pupils are sharply dilated, the child has an open fracture of the tibia, numerous sores on visible areas of the skin, fresh traces of blood on the lips. ?

Вiдновлення прохiдностi дихальних шляхiв та проведення серцево-легеневої реанiмацiї Restoration of airway patency and cardiopulmonary resuscitation

Накладання пов’язки та первинна iммобiлiзацiя нижньої кiнцiвки Applying a bandage and primary immobilization of the lower extremity

Загальний огляд потерпiлого з пальпацiєю з метою визначення провiдного ушкодження General examination of the victim with palpation to determine conductive damage

Транспортування потерпiлого до найближчого лiкувального закладу Transportation of the victim to the nearest medical facility

Введення протишокових засобiв Introduction of anti-shock drugs

86 / 200
В хiрургiчний стацiонар направлений хворий з явищами кишкової кровотечi та iмовiрною пухлиною селезiнкового кута ободової кишки. Який метод дослiдження матиме вирiшальне значення для встановлення i пiдтвердження дiагнозу? A patient with intestinal bleeding and a probable tumor of the splenic angle of the colon was sent to the surgical hospital. What research method will be of decisive importance for establishing and confirming the diagnosis?

Анамнез та фiзикальнi методи дослiдження Anamnesis and physical research methods

Оглядова рентгенограма черевної порожнини Overview x-ray of the abdominal cavity

Колоноскопiя з бiопсiєю Colonoscopy with biopsy

Iригоскопiя Irigoscopy

Ректороманоскопiя Sigmoidoscopy

87 / 200
Хвора 70-ти рокiв звернулася до лiкаря зi скаргами на блювання з домiшками жовчi, затримку випорожнень та газiв. П’ять рокiв тому оперована з приводу перфорацiйної виразки 12-палої кишки. Живiт здутий. Спостерiгаються шум плескоту та симптом Валя. Поставте дiагноз: A 70-year-old patient turned to the doctor with complaints of vomiting with bile impurities, delayed bowel movements and gases. Five years ago, she was operated on for a perforated ulcer 12- caecum. Abdomen is distended. A gurgling noise and Vala's symptom are observed. Diagnose:

Гострий панкреатит Hostry Pancreatitis

Гострий холецистит Hostry Cholecystitis

Загострення виразкової хвороби 12-палої кишки Exacerbation of duodenal ulcer

Гострий апендицит Acute appendicitis

Гостра непрохiднiсть кишечника Acute intestinal obstruction

88 / 200
У 70-рiчного пацiєнта скарги на бiль в правiй поперековiй дiлянцi з iррадiацiєю в статевi органи, часте болюче сечовипускання, пiдвищення температури тiла до 38,3oC, лихоманку, блювання. Аналiз сечi: реакцiя кисла, лейкоцитiв бiльше 110, еритроцитiв - 48 в полi зору, сеча каламутна, бурого кольору. Ультрасонографiчно виявлено нерiвномiрно вираженi змiни паренхiми, значне розширення миски правої нирки. Попереднiй дiагноз: A 70-year-old patient complains of pain in the right lumbar area with radiation to the genitals, frequent painful urination, an increase in body temperature up to 38.3oC, fever, vomiting . Urine analysis: acid reaction, leukocytes more than 110, erythrocytes - 48 in the field of vision, urine cloudy, brown in color. Ultrasonography revealed unevenly expressed changes in the parenchyma, significant expansion of the right kidney bowl. Preliminary diagnosis:

Загострення хронiчного пiєлонефриту Exacerbation of chronic pyelonephritis

Сечокам’яна хвороба Urolithiasis

Сосочковий некроз Papillary necrosis

Карбункул нирки. Гострий гнiйний паранефрит Kidney carbuncle. Acute purulent paranephritis

Сечо-сольовий дiатез. Абсцес нирки Urine-salt diathesis. Kidney abscess

89 / 200
У хлопчика 5-ти рокiв вночi стався напад ядухи, який характеризувався: сухим кашлем, експiраторною задишкою, вiдчуттям стиснення у грудях. В актi дихання бере участь допомiжна мускулатура. В анамнезi - напади ядухи 1-2 рази на мiсяць протягом 3-х рокiв, атопiчний дерматит з 2-х рокiв. Препаратом вибору для надання першої допомоги на догоспiтальному етапi є: A 5-year-old boy had a wheezing attack at night, which was characterized by: dry cough, expiratory shortness of breath, a feeling of tightness in the chest. Auxiliary muscles are involved in the act of breathing. In the anamnesis - attacks of hives 1-2 times a month for 3 years, atopic dermatitis since 2 years. The drug of choice for providing first aid at the pre-hospital stage is:

Iнгаляцiйнi холiнолiтики Inhalation cholinolytics

Бета-2-агонiсти короткої дiї Beta-2 agonists of short action

Iнгаляцiйнi глюкокортикостероїди Inhaled glucocorticosteroids

Пролонгованi бета-2-агонiсти Beta-2 agonist prolongation

Метилксантини Methylxantini

90 / 200
У дитини 10-ти мiсяцiв, яка хворiєна гостру респiраторну iнфекцiю, погiршився стан за рахунок пiдвищення температури до 39,8o. Дитина притомна. Шкiра блiда, акроцiаноз, кiнцiвки холоднi, позитивний симптом ”бiлої плями”. Тахiкардiя, задишка, судомна готовнiсть. Яку терапiю слiд призначити? The condition of a 10-month-old child with an acute respiratory infection worsened due to an increase in temperature to 39.8o. The child is unconscious. The skin is pale, acrocyanosis, extremities cold, positive 'white spot' symptom. Tachycardia, shortness of breath, convulsive readiness. What therapy should be prescribed?

Фiзичний метод охолодження Physical cooling method

Жарознижувальнi + протисудомнi препарати Antipyretics + anticonvulsants

Жарознижувальнi + судинорозширювальнi препарати Antipyretics + vasodilators

Жарознижувальнi + кардiотонiчнi препарати Antipyretic + cardiotonic drugs

Жарознижувальнi + преднiзолон Antipyretics + prednisolone

91 / 200
Хворий маєопiки обох гомiлок, 10% поверхнi тiла, стадiя 2-А, опiковий шок середньої важкостi. Яку невiдкладну допомогу необхiдно надати? The patient has burns on both legs, 10% of the body surface, stage 2-А, burn shock of medium severity. What emergency help should be provided?

Знеболювання та введення розчину Рiнгер-лактату Analgesia and injection of Ringer's lactate solution

Введення антибiотикiв та охолодження гомiлок Introduction of antibiotics and cooling of lower legs

Введення колоїдних розчинiв Introduction of colloidal solutions

Знеболювання та введення кортизону Analgesia and cortisone

Введення дiуретикiв та асептична пов’язка на раневу поверхню Introduction of diuretics and aseptic dressing on the wound surface

92 / 200
Хворий 63-х рокiв з трансмуральним задньодiафрагмальним iнфарктом мiокарда на 21 добу втратив свiдомiсть. Об’єктивно: пульс на периферичних судинах не визначається. На ЕКГ: частота серцевих скорочень 180/хв., зубцi Р вiдсутнi, шлуночковi комплекси розширенi, деформованi, ширина QRS бiльше 0,14 с; QS тип шлуночкового комплексу у V 6 . Яке ускладнення виникло у хворого? A 63-year-old patient with a transmural posterior diaphragmatic myocardial infarction lost consciousness for 21 days. Objectively: the pulse on the peripheral vessels is not determined. On the ECG: heart rate 180/min., P waves are absent, ventricular complexes are expanded, deformed, QRS width is more than 0.14 s; QS type of ventricular complex in V 6. What complication occurred in the patient?

Миготлива аритмiя Atrial fibrillation

Пароксизм суправентрикулярної тахiкардiї Paroxysm of supraventricular tachycardia

Трiпотiння шлуночкiв Ventricular flutter

Пароксизмальна шлуночкова тахiкардiя Paroxysmal ventricular tachycardia

Фiбриляцiя шлуночкiв Ventricular fibrillation

93 / 200
До гiнекологiчного вiддiлення надiйшла хвора зi значною кровотечею зi статевих шляхiв, болем внизу живота. В анамнезi: стоїть на облiку в ж/к з вагiтнiстю 12 тижнiв. Об’єктивно: хвора блiда, артерiальний тиск - 100/60 мм рт.ст., пульс - 90-100/хв. Зi статевих шляхiв значнi кров’янистi видiлення. Який найбiльш iмовiрний дiагноз? A patient came to the gynecological department with significant bleeding from the genital tract, pain in the lower abdomen. History: she is registered in the hospital with a pregnancy of 12 weeks. About' objectively: the patient is pale, blood pressure - 100/60 mm Hg, pulse - 90-100/min. There is significant bloody discharge from the genital tract. What is the most likely diagnosis?

Кровотеча з вагiни Bleeding from the vagina

Спонтанний аборт. Геморагiчний шок I ступеня Spontaneous abortion. I degree hemorrhagic shock

Неповний аборт Крок 3 Загальна лiкарська пiдготовка (україномовний варiант) 2018 рiк, весна 12 Incomplete abortion Step 3 General medical training (Ukrainian version) 2018, spring 12

Самовiльний аборт, що починається Spontaneous abortion starting

Загроза переривання вагiтностi Threat of abortion

94 / 200
У хворого 73-х рокiв у приймальному вiддiленнi раптово виникла асистолiя. Що з перерахованого, слiд враховувати лiкарю приймального вiддiлення, який проводить реанiмацiйнi заходи до прибуття бригади анестезiологiв, в зв’язку з вiком хворого? A 73-year-old patient suddenly developed asystole in the admission department. Which of the above should be taken into account by the doctor of the admission department, who carries out resuscitation measures until the arrival of the anesthesiologist team, in connection with the patient's age?

Зменшити частоту дихання при ШВЛ Reduce respiratory rate during ventilator

Вiдмовитися вiд використання дефiбрилятора Refuse to use a defibrillator

Зменшити ЧСС при непрямому масажi серця Reduce heart rate during indirect heart massage

Силу натискання при непрямому масажi серця Pressing force during indirect heart massage

Змiнити тривалiсть реанiмацiйних заходiв Change the duration of resuscitation measures

95 / 200
У хворого 30-ти рокiв пiд час сильного кашлю виникли рiзка задишка та гострий бiль в правiй половинi грудної клiтки, який посилюється при диханнi. Хворiє на бронхiт другу добу. Об’єктивно: артерiальний тиск 110/70 мм рт.ст., частота серцевих скорочень - 80/хв. Аускультативно: над легенями праворуч дихання не прослуховується. Який метод дослiдження найбiльш iнформативний в цьому випадку? A 30-year-old patient developed sharp shortness of breath and sharp pain in the right half of the chest during a strong cough, which worsens when breathing. He has bronchitis for the second day. Objectively: arterial pressure 110/70 mm Hg, heart rate - 80/min. Auscultatively: breathing is not heard above the lungs on the right. What research method is the most informative in this case?

УЗД серця Ultrasound of the heart

ЕКГ ЕКГ

УЗД органiв черевної порожнини Ultrasound of abdominal organs

Клiнiчний аналiз кровi Clinical blood analysis

Рентгеноскопiя ОГК Threat Fluoroscopy

96 / 200
Хворого 32-х рокiв, який страждаєна епiлепсiю з частими тонiко-клонiчними судомними нападами, пiсля самовiльного припинення вживання антиконвульсантiв, доставлено бригадою швидкої медичної допомоги у епiлептичному статусi. З яких препаратiв доцiльно починати невiдкладну допомогу? A 32-year-old patient suffering from epilepsy with frequent tonic-clonic convulsive attacks, after voluntarily stopping the use of anticonvulsants, was delivered by an ambulance in an epileptic state. which drugs should be used to start emergency care?

Лазикс 2,0 мл внутрiшньом’язово Lasix 2.0 ml intramuscularly

Кавiнтон 2,0 мл внутрiшньовенно на 250,0 мл фiзiологiчного розчину Cavinton 2.0 ml intravenously in 250.0 ml of physiological solution

Тiопентал-натрiю 1 мл 10% розчину на 10 кг маси хворого внутрiшньовенно Sodium thiopental 1 ml of 10% solution per 10 kg of the patient's weight intravenously

10,0 мл 25% розчину магнiю сульфату внутрiшньовенно 10.0 ml of 25% magnesium sulfate solution intravenously

Сибазон внутрiшньовенно 20 мг на 20 мл 40% розчину глюкози або фiзiологiчного розчину Sibazon intravenously 20 mg per 20 ml of 40 % glucose solution or saline

97 / 200
Хвора 52-х рокiв скаржиться на набряк та посинiння правої гомiлки, бiль, що розпирає та зменшується при пiднятому положеннi гомiлки. При пальпацiї гомiлки больова реакцiя посилюється. Яким буде дiагноз? A 52-year-old patient complains of swelling and bruising of the right shin, pain that spreads and decreases when the shin is raised. When palpating the shin, the pain reaction increases. What will it be like diagnosis?

Тромбофлебiт вен правого клубовостегнового сегменту Vein thrombophlebitis in the right clubfoot segment

Тромбофлебiт вен таза Pelvic Vein Thrombophlebitis

Тромбофлебiт нижньої порожнистої вени Thrombophlebitis of the inferior vena cava

Тромбофлебiт поверхневих вен в колiннiй дiлянцi правої нижньої кiнцiвки Thrombophlebitis of superficial veins in the knee region of the right lower limb

Тромбофлебiт глибоких вен правої гомiлки Thrombophlebitis of the deep veins of the right leg

98 / 200
На мiстi дорожньо-транспортної пригоди (ДТП) з пошкодженої автiвки вилучено дитину 10- ти рокiв з численними забоями та вiдкритим переломом гомiлки. При оглядi спостерiгаються задуха, цiаноз шкiри. Права половина грудної клiтини не бере участi у диханнi. Перкуторно над нею визначається високий тимпанiт, аускультативно дихання не прослуховується, межi серцевої тупостi змiщено влiво. Тони серця глухi, прискоренi. Ви дiагностували напружений правобiчний пневмоторакс. Якими будуть Вашi дiї? A 10-year-old child with numerous bruises and an open fracture of the lower leg was removed from the damaged car at the scene of the traffic accident. On examination, suffocation, cyanosis of the skin were observed . The right half of the thorax does not participate in breathing. High tympanitis is detected above it by percussion, breathing is not heard by auscultation, the limits of cardiac dullness are shifted to the left. Heart sounds are dull, accelerated. You have diagnosed a tense right-sided pneumothorax. What will be your actions?

Розпочати штучну вентиляцiю легень (ШВЛ) Start ventilator

Призначити кордiамiн пiдшкiрно Prescribe cordiamin subcutaneously

Призначити оксигенотерапiю Prescribe oxygen therapy

Негайна декомпресiя правої плевральної порожнини Immediate decompression of the right pleural cavity

Негайно транспортувати постраждалу дитину до лiкарнi Immediately transport the injured child to the hospital

99 / 200
Потерпiлий доставлений у МедР з вогнища через добу пiсля застосування хiмiчної зброї. Скаржиться на сильний бiль в очах, свiтлобоязнь, сльозотечу, запаморочення, слабкiсть, нудоту, блювання. Зi слiв постраждалого, краплi отруйної речовини (ОР) потрапили на шкiру обличчя й в очi. Через кiлька хвилин з’явилися вiдчуття пiску в очах, свiтлобоязнь, сльозотеча, якi неухильно наростали. При оглядi трохи загальмований, млявий, адинамiчний. Блефароспазм, виражений набряк i гiперемiя повiк, еритематознi вогнища на шкiрi обличчя. Пульс - 60/хв., ритмiчний, тони серця ослабленi, артерiальний тиск 90 /60 мм рт.ст. Дихання жорстке. Якими властивостями володiє отруйна речовина? The victim was brought to the MedR from the fire a day after the use of chemical weapons. He complains of severe pain in the eyes, photophobia, lacrimation, dizziness, weakness, nausea, vomiting. the words of the victim, drops of poisonous substance (OR) got on the skin of the face and in the eyes. After a few minutes, sensations of sand in the eyes, photophobia, lacrimation appeared, which steadily increased. On examination, he was slightly inhibited, lethargic, adynamic. Blepharospasm, marked swelling and hyperemia of the eyelids, erythematous foci on the skin of the face. Pulse - 60/min., rhythmic, heart tones are weakened, blood pressure 90/60 mm Hg. Breathing is hard. What properties does a poisonous substance possess?

Ураження ОР подразливої дiї Results of the OR of an irritating action

Ураження рiдкою ОР нервовопаралiтичної дiї Injury with a liquid OR of neuroparalytic action

Отруєння ароматичними вуглеводнями Poisoning with aromatic hydrocarbons

Ураження пароподiбним iпритом важкого ступеня Severe vaporous ipritis

Ураження рiдкою ОР шкiрно-наривної дiї Injuries with a string of OR of skin-absorptive action

100 / 200
Чоловiк 34-х рокiв, чабан, захворiв раптово. Об’єктивно: стан тяжкий, задуха, домiшки кровi в харкотиннi, t - 40oC, явища кон’юнктивiту, вираженi катаральнi явища, цiаноз, пульс - 140/хв., артерiальний тиск 80/40 мм рт.ст., над легенями притуплення перкуторного звуку, вислуховуються сухi та вологi хрипи. З епiданамнезу вiдомо про масову загибель овець. Поставте попереднiй дiагноз: A 34-year-old man, a shepherd, fell ill suddenly. Objectively: the condition is severe, suffocation, blood impurities in the sputum, t - 40oC, conjunctivitis symptoms , pronounced catarrhal phenomena, cyanosis, pulse - 140/min., arterial pressure 80/40 mm Hg, dullness of percussion sound above the lungs, dry and moist rales are heard. From the epidemiologic history, it is known about the mass death of sheep. Make a preliminary diagnosis:

Крупозна пневмонiя Croupe pneumonia

Сибiрка, легенева форма Anthrax, pulmonary form

Туляремiя, легенева форма Tularemia, Legendary form

Чума, легенева форма Plague, Legendary Form

Легiонельоз Legionellosis

101 / 200
У хворого з ангiною раптово на рiзних дiлянках шкiри з’явилися схильнi до злиття пухирi. Їх поява супроводжувалася нестерпним свербiнням, пiдвищенням температури тiла до 37,2o. У аналiзi кровi: еозiнофiлiя. Який найбiльш iмовiрний дiагноз? In a patient with angina, blisters prone to fusion suddenly appeared on different areas of the skin. Their appearance was accompanied by unbearable itching, an increase in body temperature up to 37.2o. In the analysis blood: eosinophilia. What is the most likely diagnosis?

Синдром Лайєлла Lyell's Syndrome

Кропивниця Hives

Пухирчатка Puffy

Синдром Стiвена-Джонса Steven Jones Syndrome

Вiтряна вiспа Wind Island

102 / 200
Пацiєнт 40-ка рокiв раптово, виходячи з тролейбуса, падаєна землю. У нього вiдсутнєсамостiйне дихання, та пульсацiя на магiстральних артерiях, розширенi зiницi. Ваш дiагноз: A 40-year-old patient suddenly falls to the ground while exiting a trolleybus. He is not breathing on his own, has pulsation on the main arteries, dilated pupils. Your diagnosis:

Запаморочення Dizziness

Ортостатичний колапс Orthostatic Collapse

Клiнiчна смерть Clinical death

Гострий iнфаркт мiокарда Acute myocardial infarction

Тромбоемболiя легеневої артерiї Thromboembolism of the pulmonary artery

103 / 200
Хворий 50-ти рокiв був збитий автомобiлем в результатi чого отримав вiдкритий перелом лiвого стегна в середнiй третинi з кутовим та поздовжнiм змiщенням уламкiв. Який оптимальний метод транспортної iммобiлiзацiї слiд застосувати? A 50-year-old patient was hit by a car as a result of which he received an open fracture of the left hip in the middle third with angular and longitudinal displacement of fragments. What is the optimal method of transport immobilization to be used ?

Три драбинчастi шини Three ladder tires

Шину Дiтерiхса Dieterichs Bus

Двi драбинчастi шини Two ladder tires

Шину Томаса Thomas' Tire

Шину Дiтерiхса та Крамера Dieterichs and Cramer's Tire

104 / 200
Чоловiк 72-х рокiв звернувся до лiкаря полiклiнiки зi скаргами на вiдсутнiсть видiлення сечi протягом 8 годин. Страждаєна аденому передмiхурової залози. При пальцевому ректальному дослiдженнi встановлено: передмiхурова залоза до 7 см в дiаметрi, безболiсна тугоеластична, мiжчасткова борозенка згладжена, слизова оболонка прямої кишки над нею рухома. Який метод дiагностики найбiльш iнформативний для визначення розмiрiв передмiхурової залози та об’єму залишкової сечi? A 72-year-old man turned to a polyclinic doctor with complaints about the absence of urine output for 8 hours. He suffered from adenoma of the prostate gland. During a digital rectal examination, it was found that the prostate gland 7 cm in diameter, painless, tight-elastic, the interstitial groove is smoothed, the mucous membrane of the rectum above it is mobile. What diagnostic method is the most informative for determining the dimensions of the prostate gland and the volume of residual urine?

Уретероцистоскопiя Ureterocystoscopy

Урофлоуметрiя Uroflowmetry

Екскреторна урографiя Excretory urography

Низхiдна цистографiя Low cystogram

Ультразвукова сонографiя Ultrasound sonography

105 / 200
Хворий 83-х рокiв надiйшов до приймально-дiагностичного вiддiлення з попереднiм дiагнозом: защемлена пупкова кила. Пiд час обстеження раптово зблiд, втратив свiдомiсть. Дихання вiдсутнє. Пульсацiя на магiстральних артерiях припинилася 30 секунд тому. Зiницi розширенi, на свiтло не реагують. З чого треба розпочати реанiмацiйнi заходи? The 83-year-old patient came to the reception and diagnostic department with a preliminary diagnosis: a pinched umbilical hernia. During the examination, he suddenly turned pale, lost consciousness. There is no breathing. Pulsation on in the main arteries stopped 30 seconds ago. The pupils are dilated, they do not react to light. Where should resuscitation measures be started?

Закритий масаж серця Closed heart massage

Прекардiальний удар Precordial Stroke

Штучне дихання Artificial respiration

Медикаментозна терапiя Medication

Електродефiбриляцiя Electrodefibrillation

106 / 200
У новонародженої дитини вiд матерiносiя поверхневого антигену гепатиту В (НВsAg), стан задовiльний. Активна, дихальних розладiв нема. Печiнка на 2 см нижче реберної дуги. Сеча свiтла, меконiй вiдходить. Якi профiлактичнi заходи треба призначити дитинi? A newborn child from a mother carrying hepatitis B surface antigen (HBsAg) is in satisfactory condition. She is active, there are no respiratory disorders. The liver is 2 cm below the costal arch. Urine is light, meconium comes out. What preventive measures should be prescribed to the child?

Скринiнг матерi на антиген гепатиту В Maternal screening for hepatitis B antigen

Очiкувальна тактика Fine tactics

Скринiнг дитини на НВsAg Screening of a child for HBsAg

Iзоляцiя дитини щоб уникнути поширення iнфекцiї Isolation of the child to avoid the spread of infection

Вакцинацiя проти гепатиту В на I добi життя Vaccination against hepatitis B on the 1st day of life

107 / 200
7- рiчному хлопчику з дифузним гнiйним перитонiтом в пiсляоперацiйному перiодi для компенсацiї iснуючого дефiциту калiю протягом доби потрiбно ввести 25 мл 7,5% розчину калiю хлориду. Яким чином потрiбно вводити калiю хлорид? A 7-year-old boy with diffuse purulent peritonitis in the postoperative period needs to be injected with 25 ml of 7.5% potassium chloride solution during the day to compensate for the existing potassium deficit. How should enter potassium chloride?

Внутрiшньовенно, струменево, повiльно Intravenous, jet, slow

Внутрiшньовенно, струменево, швидко Intravenous, jet, fast

Внутрiшньовенно, крапельно, повiльно Intravenous, drip, slow

Внутрiшньом’язово Intramuscular

Внутрiшньовенно, крапельно, швидко Intravenous, drip, fast

108 / 200
Потерпiлий 32-х рокiв був водiєм легкового автомобiля, що перевернувся i зайнявся. Спостерiгалася короткочасна втрата свiдомостi. Об’єктивно: анiзокорiя. Тахiпное до 30 /хв. Крепiтацiя 6-10 ребер злiва по переднiй пахвовiй лiнiї. Артерiальний тиск - 90/60 мм рт.ст. Деформацiя правого передплiччя в нижнiй третинi за типом ”багнета”, гематома. Маємiсце опiкова рана грудей, тулуба та стегон, площею 36% з наявнiстю великої кiлькостi епiдермальних пухирiв що прорвалися. Який вид травми у хворого? The 32-year-old victim was the driver of a passenger car that overturned and engaged. Short-term loss of consciousness was observed. Objectively: anisocoria. Tachypnea up to 30/min. Crepitation 6-10 ribs on the left along the front axillary line. Blood pressure - 90/60 mm Hg. Deformation of the right forearm in the lower third according to the 'bayonet' type, hematoma. There is a burn wound of the chest, trunk and thighs, the area of ​​36% with the presence of a large the number of epidermal blisters that have broken through. What type of injury does the patient have?

Поєднана Combined

Полiфокальна Polyfocal

Множинна Multiple

Монолокальна Monolocal

Комбiнована Combined

109 / 200
Хвора 32-х рокiв страждаєна виразкову хворобу дванадцятипалої кишки протягом 6-ти рокiв. Годину назад пiсля їжi з’явився раптово рiзкий бiль в епiгастральнiй областi. Загальний стан важкий, стогне вiд болю в животi. Живiт рiзко напружений, болiсний у всiх вiддiлах, черевна стiнка вiдстає в актi дихання. При пальпацiї печiнка на 1 см нижче реберної дуги. При перкусiї визначається вiдсутнiсть печiнкової тупостi. Який найбiльш iмовiрний дiагноз? A 32-year-old patient has been suffering from duodenal ulcer disease for 6 years. An hour ago, after eating, a sudden sharp pain appeared in the epigastric region. The general condition is serious , moans from abdominal pain. The abdomen is sharply tense, painful in all areas, the abdominal wall protrudes during the act of breathing. On palpation, the liver is 1 cm below the costal arch. On percussion, the absence of hepatic dullness is determined. What is the most likely diagnosis?

Гострий холецистит Hostry Cholecystitis

Гостра кишкова непрохiднiсть Acute intestinal obstruction

Перфоративна виразка Perforating ulcer

Гострий панкреатит Hostry Pancreatitis

Цироз печiнки Liver cirrhosis

110 / 200
Хворий 63-х рокiв надiйшов до хiрургiчного вiддiлення з кровотечею з хронiчної виразки правої гомiлки. Туге бинтування виразки неефективне. Якою буде тактика лiкування? Крок 3 Загальна лiкарська пiдготовка (україномовний варiант) 2018 рiк, весна 14 A 63-year-old patient came to the surgical department with bleeding from a chronic right shin ulcer. Tight bandaging of the ulcer is ineffective. What will be the treatment tactics? Step 3 General medical training ( Ukrainian version) 2018, spring 14

Венектомiя в термiновому порядку Urgent Venectomy

Склерозувальна терапiя Sclerosis therapy

Венектомiя в плановому порядку Venectomy as planned

Прошивання судини, що кровоточить Suturing a bleeding vessel

Тампонування виразки Ulcer tamponade

111 / 200
Пiсля падiння з опором на праве плече, потерпiлий скаржиться на бiль, обмеження рухiв, деформацiю в дiлянцi плечового суглоба. Якi заходи на даному етапi надання допомоги необхiдно провести? After falling with support on the right shoulder, the victim complains of pain, restriction of movement, deformation in the area of ​​the shoulder joint. What measures should be taken at this stage of providing assistance?

Транспортна iммобiлiзацiя, знеболення Transport immobilization, anesthesia

Блокада мiсця перелому Blockade of the fracture site

Накладення шини Крамера вiд пальцiв до плечового суглоба Applying a Kramer splint from the fingers to the shoulder joint

Накладення шини Дiтерiхса Dieterichs bus overlay

Накладення гiпсової пов’язки Applying a plaster cast

112 / 200
До вiддiлення травматологiї надiйшов чоловiк 25-ти рокiв iз переломом кiсток тазу (ДТП). Через кiлька годин почав скаржитися на вiдсутнiсть сечовипускання при наявностi бажання це зробити та бiль у надлобковiй зонi. При оглядi хворого привернув до себе увагу повний сечовий мiхур, при ректальному обстеженнi у хворого виявилася змiщена вверх, високо ”посаджена” простата. Який метод обстеження необхiдний для остаточного дiагнозу? A 25-year-old man came to the trauma department with a fracture of the pelvic bones (traffic accident). After a few hours, he began to complain about the lack of urination despite the desire to do so and pain in in the suprapubic area. During the examination of the patient, a full bladder attracted attention, during the rectal examination, the patient had an upwardly displaced, high 'planted' prostate. What examination method is necessary for the final diagnosis?

Уретроскопiя Ureteroscopy

УЗД органiв малого тазу Ultrasound of pelvic organs

Внутрiшньовенна урографiя Intravenous urography

Екскреторна урографiя Excretory urography

Ретроградна уретрографiя Retrograde urethrography

113 / 200
Народилася недоношена дитина, гестацiйний вiк 32 тижнi, з вагою 1700 г, рiвень глюкози кровi становить 2,1 ммоль/л. У дитини спостерiгається порушення стану, судомна готовнiсть. Оберiть першочергову допомогу для дитини: A premature baby was born, gestational age 32 weeks, weight 1700 g, blood glucose level is 2.1 mmol/l. The child is observed to have a condition disorder, convulsive readiness Choose first aid for the child:

0,9% розчин натрiю хлориду, 10 мл/кг 0.9% sodium chloride solution, 10 ml/kg

25% розчин MgSO4, 1 мл/кг 25% MgSO4 solution, 1 ml/kg

10% розчин глюкози, 2 мл/кг 10% glucose solution, 2 ml/kg

Фенобарбiтал, 20 мг/кг Phenobarbital 20mg/kg

10% розчин натрiю хлориду, 4 мл/кг 10% sodium chloride solution, 4 ml/kg

114 / 200
У вагiтної в 34 тижнi пiд час доплерографiчного ультразвукового обстеження виявили стiйкий реверсивний дiастолiчний кровоплин у судинах плода й провели кесарiв розтин. У новонародженого вiдсутнi дихання i м’язовий тонус. Вентиляцiя мiшком i маскою не забезпечуєадекватнi рухи грудної клiтки, i пiсля наступних 30 секунд вентиляцiї частота серцевих скорочень не збiльшується. Який наступний етап реанiмацiї? In a 34-week pregnant woman, persistent reversible diastolic blood flow in the fetal vessels was detected during a Doppler ultrasound examination, and a cesarean section was performed. The newborn has no breathing and muscle tone. Bag and mask ventilation does not provide adequate movements of the chest, and after the next 30 seconds of ventilation, the heart rate does not increase. What is the next stage of resuscitation?

Припинити надавати допомогу Stop helping

Iнтубацiя трахеї немовляти Intubation of the baby's trachea

Введення атропiну Introduction of atropine

Введення адреналiну Injection of adrenaline

Уведення натрiю бiкарбонату Introduction of sodium bicarbonate

115 / 200
Дiвчинка 8-ми рокiв вже другий тиждень хворiєна кашлюк. Вранцi дитина самостiйно випила увесь флакон сиропу вiд кашлю, в якому мiстився кодеїн. Спочатку дiвчинка була неспокiйною, згодом стала сонливою, мати двiчi спостерiгала судоми. Який антидот необхiдно ввести при цьому отруєннi? The 8-year-old girl has been sick with whooping cough for the second week. In the morning, the child drank the entire bottle of cough syrup containing codeine on her own. At first, the girl was restless, later she became drowsy, the mother observed convulsions twice. What antidote should be administered for this poisoning?

Налоксону гiдрохлорид Naloxone hydrochloride

Тарен Taren

Амiтриптилiн Amitriptyline

Атропiн Atropine

Цистамiн Cystamine

116 / 200
Жiнка 46-ти рокiв виведена з вогнища пожежi. Бригада екстреної медичної допомоги дiагностувала термiчнi опiки верхнiх дихальних шляхiв. З чого необхiдно розпочати невiдкладну допомогу? A 46-year-old woman was taken out of the fire. The emergency medical team diagnosed thermal burns of the upper respiratory tract. What should be done to start emergency treatment?

Iнгаляцiї сальбутамолу (вентолiну) Salbutamol (Ventolin) inhalation

Iнгаляцiї 100% киснем через лицьову маску 100% oxygen inhalation through a face mask

Аналгезiї наркотичними аналгетиками Analgesia with narcotic analgesics

Серцево-легеневої реанiмацiї Cardiopulmonary resuscitation

Довенного введення 2,4% розчину еуфiлiну Annual administration of 2.4% euphilin solution

117 / 200
Дитину 5-ти рокiв на п’ятий день гострої кишкової iнфекцiї доставлено в стацiонар у зв’язку з погiршенням загального стану. При надходженнi свiдомiсть вiдсутня, виражений ексикоз, гiпертермiя, напади судом, афазiя. Виявленi гiперглiкемiя - 50 ммоль/л, осмолярнiсть плазми (360 мОсм/л), нормальний рiвень кетонових тiл в кровi, вiдсутнiсть ацетону в сечi. Вiдомо, що з трьох рокiв хворiє на цукровий дiабет 1-го типу. З якого препарату треба починати невiдкладну iнфузiйну терапiю? On the fifth day of an acute intestinal infection, a 5-year-old child was taken to the hospital due to the deterioration of his general condition. On admission, he was unconscious, had pronounced exicosis, hyperthermia, convulsions, aphasia. Hyperglycemia was detected - 50 mmol/l, plasma osmolality (360 mOsm/l), normal level of ketone bodies in the blood, absence of acetone in the urine. It is known that he has been suffering from type 1 diabetes for the past three years What drug should be used for emergency infusion therapy?

5% розчин глюкози 5% glucose solution

10% розчин глюкози 10% glucose solution

0,45% розчин натрiю хлориду 0.45% sodium chloride solution

4% розчин натрiю гiдрокарбонату 4% sodium bicarbonate solution

10% розчин натрiю гiдрокарбонату 10% sodium bicarbonate solution

118 / 200
У постраждалого в дорожньотранспортнiй пригодi (ДТП) чоловiка 31-го року лiкар швидкої медичної допомоги дiагностував закритий перелом правого стегна, травматичний шок 2 ступеня. В комплексi протишокової терапiї почав швидку в/в iнфузiю кристалоїдiв. Через 10 хвилин систолiчний артерiальний тиск стабiлiзовано на 90 мм рт.ст. Яка подальша iнфузiйна терапiя потрiбна потерпiлому? In a 31-year-old man injured in a road traffic accident (accident), an emergency medical doctor diagnosed a closed fracture of the right hip, traumatic shock of the 2nd degree. In the complex of anti-shock therapy, he started rapid IV infusion of crystalloids. After 10 minutes, systolic blood pressure is stabilized at 90 mm Hg. What further infusion therapy does the victim need?

Повiльне крапельне введення кристалоїдiв Slow drip introduction of crystalloids

Швидке введення альбумiну Rapid albumin injection

Крапельне введення розчину крохмалю Drip introduction of starch solution

Крапельне введення еритроцитарної маси Drip administration of erythrocyte mass

Швидке введення розчину низькомолекулярних декстранiв Quick introduction of a solution of low molecular weight dextrans

119 / 200
Хворого 39-ти рокiв госпiталiзовано пiсля дорожньо-транспортної пригоди (ДТП) з дiагнозом ”Забiй грудної клiтки”. Скаржиться на бiль у лiвiй половинi грудної клiтки, задишку. Об’єктивно: блiдий, акроцiаноз, артерiальний тиск - 100/60 мм рт.ст., пульс 110 /хв. При глибокому диханнi вiдзначається значне вiдставання лiвої половини грудної клiтки, аускультативно - вiдсутнiсть дихальКрок 3 Загальна лiкарська пiдготовка ( україномовний варiант) 2018 рiк, весна 15 них шумiв злiва. Поставте попереднiй дiагноз : A 39-year-old patient was hospitalized after a traffic accident (accident) with a diagnosis of 'Chest Bruise'. He complains of pain in the left side of the chest, shortness of breath. Objectively: pale, acrocyanosis, blood pressure - 100/60 mmHg, pulse 110/min. During deep breathing, significant protrusion of the left half of the chest is noted, auscultation - absence of breathing Step 3 General medical training (Ukrainian version) 2018 , spring 15 of them noises from the left. Make a preliminary diagnosis :

Вiдкритий пневмоторакс Open pneumothorax

Закрита травма живота Tummy injury closed

Закритий пневмоторакс злiва Closed left pneumothorax

Перелом ребер Fractured Ribs

Закритий пневмоторакс справа Closed pneumothorax on the right

120 / 200
У родiллi 36-ти рокiв, яка перебуваєв кiнцi I перiоду пологiв з великим плодом, з’явилися потуги при високому розташуваннi голiвки плода, вiдсутнiсть поступального руху голiвки, позитивний симптом ГенкеляВастена, симптоми вдавлення сечового мiхура. Який найбiльш iмовiрний дiагноз? In a 36-year-old woman in labor who was at the end of the first stage of labor with a large fetus, there were efforts at a high position of the fetal head, the absence of forward movement of the head, positive HenkelVasten's symptom, bladder compression symptoms. What is the most likely diagnosis?

Дискоординована пологова дiяльнiсть Discoordinated birth activity

Слабкiсть пологової дiяльностi Weakness of labor activity

Анатомiчно вузький таз Anatomically narrow pelvis

Фiзiологiчний I перiод пологiв Physiological I period of childbirth

Клiнiчно вузький таз Clinically narrow pelvis

121 / 200
Хворий 32-х рокiв перебуваєв збудженому станi, напружений, вiд чогось вiдмахується. Показує на вiкно i говорить, що йому хтось погрожує, викликає його. Стверджує, що в кiмнатi повно щурiв, ногою на них замахується. Рiк тому отримав ЧМТз втратою свiдомостi. З’ясувалось, що хворий зловживаєалкоголем, останнiй тиждень п’єзапоєм. Третю нiч не спить, боїться виходити з дому. Не хоче їхати в лiкарню. Яким буде дiагноз та якої допомоги потребуєхворий? The 32-year-old patient was in an excited state, tense, waving away something. He points to the window and says that someone is threatening him, calling him. He claims that in the room is full of rats, he swings at them with his foot. A year ago he received a TBI with loss of consciousness. It turned out that the patient abuses alcohol, has been drinking for the last week. He has not slept for the third night, he is afraid to leave the house. He does not want to go to the hospital. What will be the diagnosis and what kind of help does the patient need?

Сутiнковий стан свiдомостi. Слiд надати медичну допомогу на дому Twilight state of consciousness. Medical assistance should be provided at home

Соматогенний психоз. Лiкування в терапевтичному вiддiленнi Somatogenic psychosis. Treatment in the therapeutic department

Алкогольний делiрiй. Пiдлягаєгоспiталiзацiї в психiатричну лiкарню Alcoholic delirium. Subject to hospitalization in a psychiatric hospital

Патологiчне сп’янiння. Дати снодiйне Pathological intoxication. Give sleeping pills

Посттравматичний психоз. Лiкування в неврологiчному вiддiленнi Post-traumatic psychosis. Treatment in the neurological department

122 / 200
На дорожньо-транспортну пригоду була викликана бригада швидкої медичної допомоги (ШМД). Постраждалий непритомний, у нього дiагностовано вiдкритий перелом правої стегнової кiстки, артерiальний тиск 80/40 мм рт.ст. Якими будуть дiї лiкаря швидкої допомоги? The emergency medical team was called to the traffic accident. The victim is unconscious, he was diagnosed with an open fracture of the right femur, blood pressure 80/40 mm What will be the actions of the emergency doctor?

Iммобiлiзацiя шинами Крамера, введення вазопресорiв, блокада мiсця перелому Immobilization with Kramer splints, administration of vasopressors, blockade of the fracture site

Iммобiлiзацiя, транспортування в лiкарню Immobilization, transportation to hospital

Iммобiлiзацiя, крапельне введення плазмозамiнних розчинiв, асептична пов’язка Immobilization, drip administration of plasma exchange solutions, aseptic dressing

Негайно транспортувати в спецiалiзоване вiддiлення Immediately transport to a specialized department

Знеболювання, асептична пов’язка, iммобiлiзацiя шиною Дiтерiхса, iнфузiйна терапiя Anesthesia, aseptic dressing, immobilization with a Dietrich splint, infusion therapy

123 / 200
Потерпiлого було витягнуто з автомобiля пiсля дорожньо-транспортної пригоди (ДТП). Скаржиться на бiль в шийному вiддiлi хребта, який посилюється при рухах головою. Вкажiть, який метод транспортної iммобiлiзацiї потрiбен при даному пошкодженнi: The victim was pulled out of the car after a traffic accident (traffic accident). He complains of pain in the cervical spine, which worsens when he moves his head. Indicate what method of transport immobilization is required for this damage:

Положення хворого на жорстких ношах з валиком пiд шиєю Position of the patient on a rigid stretcher with a roller under the neck

Iммобiлiзацiя шиною Крамера вiд потилицi до поперекового вiддiлу хребта Immobilization with a Kramer splint from the back of the head to the lumbar spine

Iммобiлiзацiя жорстким головоутримувачем Immobilization with a rigid head restraint

Положення хворого на спинi з валиком пiд головою Position of the patient on his back with a pillow under his head

Положення хворого на животi на жорстких ношах Position of a patient on his stomach on a hard stretcher

124 / 200
Хворого 72-х рокiв доставлено до лiкарнi машиною швидкої допомоги у станi непритомностi. При оглядi: шкiра блiда, суха зi зниженим тургором, тонус очних яблук знижений, арефлексiя, пульс частий, м’який, артерiальний тиск - 70/40 мм рт.ст. У додаткових дослiдженнях: виявлена глюкоза в сечi, цукор кровi - 28,5 ммоль/л, гiпернатрiємiя, осмолярнiсть плазми - 430 мосм/л. Розвиток якого стану можна припустити? A 72-year-old patient was brought to the hospital by ambulance in a state of unconsciousness. On examination: the skin is pale, dry with reduced turgor, the tone of the eyeballs is reduced, areflexia, the pulse is frequent, soft, blood pressure - 70/40 mm Hg. In additional studies: glucose was detected in the urine, blood sugar - 28.5 mmol/l, hypernatremia, plasma osmolarity - 430 mosm/l. Development of which condition can we assume?

Гiперлактацидемiчної коми Hyperlactacidemic Komi

Гiперосмолярної коми Hyperosmolar coma

Кетоацидотичної коми Ketoacidotic Komi

Гiпотиреоїдної коми Hypothyroid coma

Гострої надниркової недостатностi Acute adrenal insufficiency

125 / 200
Хворий 18-ти рокiв пiд час пiрнання потонув. Пiсля проведення заходiв реанiмацiї близькими потерпiлого було вiдновлено самостiйне дихання. На час прибуття бригади швидкої допомоги на мiсце подiї хворий самостiйно дихає, свiдомiсть не порушена. Спостерiгається легкий стан алкогольного сп’янiння. Частота дихання - 20/хв., артерiальний тиск - 140/70 мм рт.ст., пульс - 100/хв. Скаржиться на слабкiсть, при частому кашлi вiдхаркуєбiлувату пiну. Яка подальша тактика ведення такого потерпiлого? An 18-year-old patient drowned while diving. After resuscitation measures by relatives of the victim, independent breathing was restored. By the time the ambulance crew arrived at the scene, the patient was breathing on his own , consciousness is not disturbed. A mild state of alcohol intoxication is observed. Breathing rate - 20/min., blood pressure - 140/70 mm Hg, pulse - 100/min. He complains of weakness, when he coughs frequently, he coughs up whitish foam. What further tactics of managing such a victim?

Спостереження за хворим протягом 30 хвилин, за вiдсутностi погiршення стану амбулаторний нагляд Observation of the patient for 30 minutes, in the absence of deterioration of the condition, outpatient supervision

Вiдвiдування районної лiкарнi пiсля зникнення ознак сп’янiння Visit to the district hospital after the disappearance of signs of intoxication

Госпiталiзацiя для подальшого спостереження та обстеження Hospitalization for follow-up and examination

Хворий не потребуєподальшого спостереження та лiкування The patient does not need further observation and treatment

Хворому мають бути призначенi заспокiйливi засоби (настойки валерiани чи пiвонiї), рекомендовано вiдмовитися вiд вживання алкогольних напоїв The patient should be prescribed sedatives (tinctures of valerian or peony), it is recommended to refrain from drinking alcoholic beverages

126 / 200
Хворий 56-ти рокiв звернувся зi скаргами на бiль та набряк лiвої гомiлки. Об’єктивно: лiва гомiлка збiльшена в дiаметрi (+5 см у порiвняннi з правою), тепла на дотик. Болючiсть та напружений набряк гомiлки при пальпацiї. Для уточнення дiагнозу першочергово необхiдно виконати: A 56-year-old patient complained of pain and swelling of the left shin. Objectively: the left shin has increased in diameter (+5 cm compared to the right) , warm to the touch. Pain and intense swelling of the lower leg during palpation. To clarify the diagnosis, it is necessary to perform:

Коагулограму Coagulogram

Плетизмографiю нижнiх кiнцiвок Plethysmography of the lower extremities

Флебографiю нижнiх кiнцiвок Phlebography of the lower extremities

Рентгенографiю нижнiх кiнцiвок X-ray of the lower extremities

Доплерографiю вен нижнiх кiнцiвок Dopplerography of the veins of the lower extremities

127 / 200
Хворий 24-х рокiв лiкується у вiддiленнi iнтенсивної терапiї з приводу синдроКрок 3 Загальна лiкарська пiдготовка (україномовний варiант) 2018 рiк, весна 16 му тривалого стиснення, гострої ниркової недостатностi. При лабораторному обстеженнi: сечовина 28,4 ммоль/л; креатинiн 820 мкмоль/л; К - 8,4 ммоль/л; гемоглобiн 76 г/л; гематокрит 0,26 л/л; еритроцити 2, 4 • 1012/л. Яке ускладнення може безпосередньо загрожувати життю хворого? A 24-year-old patient is being treated in the intensive care unit for syndroStep 3 General medical training (Ukrainian-language version) 2018, spring 16 of prolonged compression, acute renal failure During laboratory examination: urea 28.4 mmol/l, creatinine 820 μmol/l, K - 8.4 mmol/l, hemoglobin 76 g/l, hematocrit 0.26 l/l, erythrocytes 2.4 • 1012/l . What complication can directly threaten the patient's life?

ДВЗ-синдром DVZ-syndrome

Гемiчна гiпоксiя Hemic hypoxia

Респiраторний дистрес-синдром Respiratory Distress Syndrome

Анемiя Анемiя

Гiперкалiємiчна зупинка серця Hyperkalemic cardiac arrest

128 / 200
Дитинi 10 мiсяцiв. Скарги на to - 38oC, повторне блювання, пронос, вiдсутнiсть апетиту. Дитина млява, виражена спрага. Шкiра блiда, еластичнiсть знижена. Слизова рота волога. Пiднебiннi дужки гiперемованi. Носове дихання утруднене, мiзернi слизовi видiлення. Живiт здутий, помiрна болючiсть, бурчання. Випорожнення за добу 10 разiв, ряснi, водянистi, жовто-зеленкуватi, без патологiчних домiшок. Незначна олiгоурiя. Поставте дiагноз: The child is 10 months old. Complaints about temperature - 38oC, repeated vomiting, diarrhea, lack of appetite. The child is lethargic, very thirsty. The skin is pale, elasticity is reduced. The mucous membrane of the mouth is moist Palate arches are hyperpronounced. Nasal breathing is difficult, scanty mucous secretions. Abdomen is distended, moderate pain, grumbling. Defecation 10 times a day, abundant, watery, yellow-greenish, without pathological impurities. Slight oliguria. Make a diagnosis:

Сальмонельоз, гастроентероколiтична середньотяжка форма Salmonellosis, gastroenterocolitic moderate form

Ешерихiоз ентеропатогенний, типова середньотяжка форма Escherichia enteropathogenic, typical moderate form

Шигельоз, типова середньотяжка форма Shigelosis, typical moderate form

Холера Cholera

Ротавiрусна iнфекцiя, типова середньотяжка форма Rotavirus infection, typical moderate form

129 / 200
Дитина народилася вiд 2 вагiтностi, 2 пологiв в термiнi 34-35 тижнiв, з оцiнкою за шкалою Апгар 1/3 балiв, на початку другої доби спостерiгається зригування кров’янистим вмiстом, мелена. Нb- 165 г/л, еритроцити - 4, 8 • 1012/л, тромбоцити - 192 • 109/л. Назвiть основнi принципи лiкування: The child was born from 2 pregnancies, 2 deliveries at 34-35 weeks, with an Apgar score of 1/3 points, at the beginning of the second day, vomiting of blood is observed' with yellow content, ground. Hb - 165 g/l, erythrocytes - 4.8 • 1012/l, platelets - 192 • 109/l. Name the main principles of treatment:

Внутрiшньовенне болюсне введення 10% розчину альбумiну Intravenous bolus administration of 10% albumin solution

Iнфузiя 10% глюконату кальцiю Infusion of 10% calcium gluconate

Термiнове переливання одногрупної кровi Urgent blood transfusion of the same group

Введення вiтамiну K1, плазмотрансфузiя Introduction of vitamin K1, plasma transfusion

Iнфузiя 5% розчину амiнокапронової кислоти Infusion of 5% aminocaproic acid solution

130 / 200
У жiнки 35-ти рокiв через вживання забруднених харчових продуктiв в органiзм потрапила значна кiлькiсть радiоактивного йоду-131. Якi заходи необхiднi в першу чергу? A 35-year-old woman ingested a significant amount of radioactive iodine-131 due to the consumption of contaminated food products. What measures are necessary first of all?

Негайно провести евакуацiю жiнки Evacuate the woman immediately

Якнайшвидше розпочати йодну профiлактику Start iodine prophylaxis as soon as possible

Негайно промити шлунок Immediately flush the stomach

Провести антибiотикотерапiю для профiлактики вторинної iнфекцiї на тлi пригнiчення iмунiтету Conduct antibiotic therapy to prevent secondary infection against the background of immunosuppression

Ретельно зiбрати харчовi продукти в закриту тару для подальшого аналiзу Carefully collect food products in a closed container for further analysis

131 / 200
Пацiєнта 19-ти рокiв доставлено з мiсця дорожньо-транспортної пригоди (ДТП), лiва нижня кiнцiвка iммобiлiзована шиною Дiтерiхса. Свiдомiсть потьмарена, шкiрнi покриви блiдi, з синюшним вiдтiнком. Пульс 130/хв., артерiальний тиск - 70/0 мм рт.ст. Дихання поверхневе, часте. Спостерiгаються гiпорефлексiя, м’язова гiпотонiя. Який патофiзiологiчний стан супроводжуєдане пошкодження? A 19-year-old patient was brought from the scene of a traffic accident (traffic accident), the left lower extremity was immobilized with a Dieterichs splint. Consciousness is dimmed, the skin is pale, with a bluish tinge . Pulse 130/min., arterial pressure - 70/0 mm Hg. Breathing is shallow, frequent. Hyporeflexia, muscle hypotonia are observed. What pathophysiological condition accompanies this damage?

Травматичний шок II ступеня Traumatic shock of the II degree

Кома глибока Comma deep

Травматичний шок III ступеня Traumatic shock III degree

Травматичний шок I ступеня Traumatic shock of the first degree

Кома помiрна Coma is peaceful

132 / 200
Чоловiк 26-ти рокiв неадекватно себе поводив, звернувся до вiддiлку мiлiцiї з вимогами, щоб його передали до суду, оскiльки вiн вiдчуваєсебе винним за розповсюдження зла та насильства у свiтi. Спiвробiтникам мiлiцiї вдалося встановити особу чоловiка, зв’язатися з його родичами, якi повiдомили, що за останнi 5 рокiв хворий неодноразово лiкувався у психiатричних закладах. При оглядi збуджений, легко стаєрозлюченим, пiдозрiлим. Стверджує, що голос всерединi голови говорить: ”я заслуговую на смерть”. Який тип психомоторного збудження у хворого? A 26-year-old man behaved inappropriately, turned to the police department with demands that he be brought to court, as he feels guilty for spreading evil and violence in the world . The police officers managed to establish the identity of the man, to contact his relatives, who reported that over the past 5 years the patient had been treated repeatedly in psychiatric institutions. When examined, he is excited, easily becomes angry, suspicious. He claims that a voice inside his head says: 'I deserve to death'. What type of psychomotor excitement does the patient have?

Кататонiчне Catatonic

Депресивне Depressed

Манiакальне Manic

Галюцинаторно-параноїдне Hallucinatory-paranoid

Психопатичне Psychopathic

133 / 200
У вагiтної пiд час стрiмких пологiв з’явилися вiдчуття задухи, цiаноз, падiння артерiального тиску. Через деякий час кров, що надходила з пiхви, перестала згортатися. Який найбiльш iмовiрний дiагноз? During rapid labor, a pregnant woman felt suffocation, cyanosis, and a drop in blood pressure. After some time, the blood coming from the vagina stopped clotting. What is the biggest probable diagnosis?

Серцева недостатнiсть Heart failure

Бронхiальна астма Bronchial asthma

Емболiя навколоплiдними водами Amniotic fluid embolism

Гiпотонiчна маткова кровотеча Hypotonic uterine bleeding

Анафiлактичний шок Anaphylactic Shock

134 / 200
У дитини вiком 2 тижнi спостерiгається раптове блювання фонтаном пiсля їжi. Пiсля блювання стан дещо покращується, але наступний прийом їжi закiнчується так само. Вiд призначення спазмолiтикiв ефекту немає. Поставте попереднiй дiагноз: A 2-week-old child suddenly vomits with a fountain after eating. After vomiting, the condition improves somewhat, but the next meal ends in the same way. There is no effect from the appointment of antispasmodics. Put preliminary diagnosis:

Iнвагiнацiя Intussusception

Пiлоростеноз Pylorostenosis

Пiлороспазм Pylorospasm

Гастроентерит Gastroenteritis

Кишкова непрохiднiсть Intestinal obstruction

135 / 200
Солдат перебуваєу вiйськовому польовому пересувному госпiталi з кульовим пораненням середньої третини правого стегна. Стан важкий, шкiра блiда, температура тiла - 37,8oC, пульс - 106/хв., артерiальний тиск 90/70 мм рт.ст. Останнi 3 днi турбуєкашель з харкотинням. При аускультацiї у нижнiй частцi лiвої легенi вологi хрипи. З чим пов’язане погiршення стану? Крок 3 Загальна лiкарська пiдготовка (україномовний варiант) 2018 рiк, весна 17 The soldier is in a military field mobile hospital with a bullet wound in the middle third of the right thigh. The condition is serious, the skin is pale, the body temperature is 37.8oC, the pulse is 106/min. , blood pressure 90/70 mm Hg. For the last 3 days, a hacking cough with sputum. During auscultation in the lower lobe of the left lung, there is a wet wheeze. What is the reason for the worsening of the condition? Step 3 General medical training (Ukrainian-language version) 2018, spring 17

Плеврит Pleuri

Гостра респiраторна вiрусна iнфекцiя (ГРВI) Acute respiratory viral infection (ARVI)

Пневмоторакс Pneumothorax

Приєднання пневмонiї Pneumonia accession

Набряк легень Pulmonary edema

136 / 200
Вiйськовослужбовець опинився в осередку хiмiчного ураження. За даними хiмiчної розвiдки супротивник застосував зарин. У потерпiлого швидко погiршився зiр, з’явилися слинотеча, вiдчуття стиснення за грудниною. Який антидот маєввести собi потерпiлий? The serviceman found himself in the center of a chemical attack. According to chemical intelligence, the enemy used sarin. The victim's eyesight rapidly deteriorated, drooling, a feeling of compression behind the sternum appeared. What an antidote should the victim introduce himself?

Амiлнiтрит Amyl nitrite

Будаксим Будаксим

Антидоту немає There is no antidote

Унiтiол Унiтiол

Дипiроксим Dipoxime

137 / 200
Пiсля опромiнення до госпiталю доставлений юнак 26-ти рокiв. Показник iндивiдуального дозиметра 5 Гр. Скарги на рiзку загальну слабкiсть, головний бiль, нудоту, повторне блювання. Об’єктивно: на шкiрi обличчя еритема, пульс - 100/хв., артерiальний тиск 90 /60 мм рт.ст., блювання продовжується. В якому клiнiчному перiодi гострої променевої хвороби перебуваєхворий? After radiation, a 26-year-old man was taken to the hospital. The individual dosimeter reading was 5 Gy. Complaints of severe general weakness, headache, nausea, repeated vomiting. Ob' objectively: erythema on the skin of the face, pulse - 100/min., blood pressure 90/60 mm Hg, vomiting continues. In what clinical period of acute radiation sickness is the patient?

Перiод первинної реакцiї Primary reaction period

Перiод вторинної реакцiї Secondary reaction period

Перiод виражених клiнiчних проявiв Period of pronounced clinical manifestations

Перiод запалення Inflammation period

Перiод розпалу Heating period

138 / 200
В результатi проведення антитерористичної операцiї боєць на полi бою внаслiдок дiї ударної хвилi отримав тупу травму живота. При оглядi, через 30 хвилин, солдат блiдий, артерiальний тиск - 80/50 мм рт.ст., пульс - 110/хв., частота дихальних рухiв - 18/хв., живiт помiрно болючий у всiх вiддiлах. Яке iмовiрне ускладнення виникло у хворого? As a result of an anti-terrorist operation, a soldier on the battlefield received a blunt trauma to the abdomen as a result of the impact of the shock wave. On examination, 30 minutes later, the soldier is pale, blood pressure - 80/ 50 mm Hg, pulse - 110/min., frequency of respiratory movements - 18/min., abdomen is moderately painful in all departments. What possible complication occurred in the patient?

Пневмоторакс Pneumothorax

Пневмоперитонеум Pneumoperitoneum

Гемопневмоторакс Hemopneumothorax

Гемоторакс Hemothorax

Гемоперитонеум Hemoperitoneum

139 / 200
Хворого 47-ми рокiв госпiталiзовано у терапевтичне вiддiлення. Скарги на зниження апетиту, багаторазовий пронос, кровоточивiсть ясен, бiль при ковтаннi, пiдвищену пiтливiсть, якi виникли пiсля опромiнювання. Об’єктивно: хворий загальмований, крововиливи у пахвиннiй дiлянцi. Температура тiла - 38,7o. Пульс - 90/хв., артерiальний тиск - 140/95 мм рт.ст. Вiдзначається значне розширення площi вiдносної серцевої тупостi вправо. Тони серця приглушенi, ритмiчнi, систолiчний шум над верхiвкою. Встановлено дiагноз гострої променевої хвороби. Який перiод гострої променевої хвороби найбiльш iмовiрний у потерпiлого? A 47-year-old patient was hospitalized in the therapeutic department. Complaints of decreased appetite, frequent diarrhea, bleeding gums, pain when swallowing, increased sweating, which occurred after radiation. Objectively: the patient is retarded, hemorrhages in the inguinal region. Body temperature - 38.7o. Pulse - 90/min., arterial pressure - 140/95 mm Hg. Significant expansion of the area of ​​relative heart dullness to the right is noted. Heart sounds are muffled, rhythmic, systolic murmur over the apex. A diagnosis of acute radiation sickness has been established. What period of acute radiation sickness is most likely in the victim?

Перiод виходiв Period of outputs

Перiод розпалу Heating period

Перiод вiддалених наслiдкiв Period of remote consequences

Початковий перiод Initial period

Латентний перiод Latent period

140 / 200
Через годину пiсля аварiї на пiдприємствi з виробництва пластмас спiвробiтник скаржиться на головний бiль, шум у вухах, слинотечу, пронос, пiдвищення температури тiла до 38o. Дiагностовано отруєння похiдними фенолу. Вмiст якої речовини у кровi пiдвищується при отруєннi фенолом? An hour after the accident at the plastics production enterprise, the employee complains of a headache, tinnitus, drooling, diarrhea, an increase in body temperature up to 38o. Poisoning by phenol derivatives is diagnosed The content of which substance in the blood increases during phenol poisoning?

Карбгемоглобiн Carbhemoglobin

Креатинiн та сечовина Creatinine and urea

Карбоксигемоглобiн Carboxyhemoglobin

Калiй Калiй

Метгемоглобiн Метгемоглобiн

141 / 200
Потерпiлого 36-ти рокiв переведено до хiрургiчного вiддiлення вiйськового госпiталю через 4 доби лiкування в районнiй лiкарнi. Дiагноз: уламкове слiпе множинне поранення лiвої кiнцiвки. У пацiєнта пiдвищилася температура тiла, рана почервонiла, набрякла. Який вид хiрургiчної допомоги слiд надати потерпiлому? The 36-year-old victim was transferred to the surgical department of the military hospital after 4 days of treatment in the district hospital. Diagnosis: multiple blind fragmentary wound of the left extremity. The patient's body temperature increased , the wound is red, swollen. What kind of surgical care should be given to the victim?

Вторинна хiрургiчна обробка рани Secondary surgical treatment of the wound

Рання первинна хiрургiчна обробка рани Early primary surgical treatment of the wound

Пiзня первинна хiрургiчна обробка рани Late primary surgical treatment of the wound

Ампутацiя кiнцiвки Kintz Amputation

Вiдкладена первинна хiрургiчна обробка рани Postponed initial surgical treatment of the wound

142 / 200
До медичного пункту окремої вiйськової частини доставлено вiйськовослужбовця з вогнепальним пораненням живота та явними ознаками внутрiшньої кровотечi. Загальний стан важкий. Блiдiсть шкiри та слизових оболонок. Пульс - 130/хв. Систолiчний артерiальний тиск - 70/40 мм рт.ст. Язик сухий. Бiль при пальпацiї живота, симптоми подразнення очеревини позитивнi. Притуплення перкуторного звуку в пологих мiсцях черевної порожнини. В якому положеннi необхiдно евакуювати пораненого на наступний етап медичної евакуацiї? A military serviceman with a gunshot wound to the abdomen and obvious signs of internal bleeding was brought to the medical center of a separate military unit. The general condition is serious. Paleness of the skin and mucous membranes. Pulse - 130/min . Systolic blood pressure - 70/40 mm Hg. Dry tongue. Pain when palpating the abdomen, symptoms of peritoneal irritation are positive. Dull percussion sound in soft areas of the abdominal cavity. In what position should the wounded be evacuated to the next stage of medical evacuation?

У положеннi лежачи на щитi Lying on the shield

У положеннi сидячи Seated

У положеннi лежачи на спинi з пiднятим головним кiнцем носилок In the supine position with the head end of the stretcher raised

У положеннi стоячи Standing

У положеннi лежачи In the lying position

143 / 200
Рядовий був притиснутий уламками зруйнованого будинку протягом 7 годин. Хворий збуджений, погано орiєнтується у навколишньому середовищi. Пульс - 92/хв., слабкого наповнення i напруження, набряк всiєї правої нижньої кiнцiвки, на зовнiшнiй поверхнi стегна та гомiлки шкiра маєсиньо-багряний колiр. Рухи в колiнному та гомiлковоступневому суглобах вiдсутнi. Протягом доби поранений не мочився. Поставте дiагноз: Крок 3 Загальна лiкарська пiдготовка (україномовний варiант) 2018 рiк, весна 18 The private was crushed by the debris of a destroyed building for 7 hours. The patient is excited, poorly oriented in the surrounding environment. Pulse - 92/min., weak filling and tension, swelling of the entire of the right lower extremity, on the outer surface of the thigh and lower leg, the skin has a blue-purple color. There are no movements in the knee and ankle joints. The injured person did not urinate during the day. Make a diagnosis: Step 3 General medical training (Ukrainian version) 2018, spring 18

Синдром тривалого розчавлення легкого ступеня Prolonged crushing syndrome of mild degree

Синдром тривалого розчавлення тяжкого ступеня, II стадiя Prolonged severe crushing syndrome, stage II

Травматичний шок середнього ступеня, еректильна фаза Traumatic shock of moderate degree, erectile phase

Травматичний шок тяжкого ступеня, торпiдна фаза Severe traumatic shock, torpid phase

Синдром тривалого розчавлення Prolonged crushing syndrome

144 / 200
У хворого 57-ми рокiв, пiд час трансфузiї еритроцитарної маси виникли занепокоєння, задишка. При оглядi: почервонiння обличчя, цiаноз, пульс - 110/хв., артерiальний тиск 70 /40 мм рт.ст., сеча червонуватого кольору. Який дiагноз найбiльш iмовiрний? A 57-year-old patient developed anxiety and shortness of breath during transfusion of erythrocyte mass. On examination: facial redness, cyanosis, pulse - 110/min., arterial blood pressure is 70/40 mm Hg, urine is reddish in color. What is the most likely diagnosis?

Анафiлактичний шок Anaphylactic Shock

Гостра ниркова недостатнiсть Acute renal failure

Гостра серцева недостатнiсть Acute heart failure

Переливання несумiсної еритромаси Transfusion of incompatible erythromass

Переливання свiжоцитратної кровi Transfusion of fresh citrated blood

145 / 200
Хворий 28-ми рокiв скаржиться на слабкiсть, мерехтiння в очах, запаморочення, випорожнення темного кольору. Протягом 10ти рокiв хворiєна виразку. При оглядi: шкiра та слизовi оболонки блiдi, пульс - 112/хв., артерiальний тиск - 90/60 мм рт.ст. Hb- 86 г/л. До якого вiддiлення слiд направити хворого? A 28-year-old patient complains of weakness, twinkling in the eyes, dizziness, dark stools. She has had an ulcer for 10 years. On examination: the skin and mucous membranes are pale , pulse - 112/min., arterial pressure - 90/60 mm Hg, Hb - 86 g/l. To which department should the patient be sent?

До вiддiлення iнтенсивної терапiї Before isolation of intensive therapy

До гастроентерологiчного вiддiлення To the gastroenterology department

До проктологiчного вiддiлення Before proctological separation

До терапевтичного вiддiлення To the therapeutic department

До хiрургiчного вiддiлення Before surgical separation

146 / 200
Пацiєнт 74-х рокiв протягом останнiх 5ти рокiв страждаєна доброякiсну гiперплазiю простати. 4 доби тому, пiсля вживання алкоголю, виникла гостра затримка сечовипускання. На догоспiтальному етапi двiчi на добу катетеризували сечовий мiхур металевим катетером. Пiд час огляду: придаток правого яєчка збiльшений, ущiльнений, болючий, єгнiйнi видiлення з уретри. Який вид невiдкладної допомоги слiд обрати? A 74-year-old patient has been suffering from benign prostatic hyperplasia for the past 5 years. 4 days ago, after drinking alcohol, there was an acute urinary retention. At the pre-hospital stage, twice a day the urinary bladder was catheterized with a metal catheter. During the examination: the right epididymis is enlarged, compacted, painful, purulent discharge from the urethra. What type of emergency care should be chosen?

Встановлення iнтрапростатичного стенту Intraprostatic stent installation

Встановлення постiйного уретрального катетера Installation of permanent urethral catheter

Трансуретральна резекцiя Transurethral Resection

Епiцистостомiя в ургентному порядку Urgent epicystostomy

Мiкрохвильова термотерапiя простати Microwave thermotherapy of the prostate

147 / 200
У хворої 52-х рокiв з наявним трансфузiологiчним анамнезом одразу пiсля початку внутрiшньовенного введення свiжозамороженої плазми виникли непродуктивний кашель, бронхоспазм, задуха, нудота, бiль у животi, гiпотонiя, пiсля чого наступила втрата свiдомостi. Якi найбiльш правильнi лiкувальнi заходи? A 52-year-old patient with a history of transfusions developed a non-productive cough, bronchospasm, suffocation, nausea, abdominal pain, hypotension, after which came the loss of consciousness. What are the most correct treatment measures?

Джгут вище мiсця введення, обколювання розчином адреналiну, гiдрокортизон в/в Tourniquet above the injection site, injection with adrenaline solution, IV hydrocortisone

Продовження iнфузiї, введення адреностимуляторiв, глюкокортикостероїдiв Continuation of infusion, introduction of adrenostimulators, glucocorticosteroids

В/в введення еуфiлiну, гiдрокортизону, iнфузiйна терапiя, iнгаляцiя кисню IV administration of euphylin, hydrocortisone, infusion therapy, oxygen inhalation

Негайна зупинка iнфузiї, iнтубацiя трахеї, штучна вентиляцiя легень (ШВЛ) Immediate stop of infusion, tracheal intubation, artificial ventilation of lungs (VHL)

Негайна зупинка iнфузiї, в/в введення адреналiну, преднiзолону, еуфiлiну, iнгаляцiя кисню Immediate stop of infusion, intravenous administration of adrenaline, prednisolone, euphylline, oxygen inhalation

148 / 200
Хворий 70-ти рокiв надiйшов зi скаргами на лихоманку до 39oC, тупий, ниючий бiль в поперековiй дiлянцi справа. В анамнезi: гематурiя з черв’якоподiбними згустками кровi. Об’єктивно: в поперековiй дiлянцi справа пальпується утворення еластичної консистенцiї розмiром з кулак, малорухливе, дещо болiсне. Припущено новоутворення нирки. Яка перша ознака злоякiсних новоутворень нирок? A 70-year-old patient came in with complaints of fever up to 39oC, dull, aching pain in the lumbar region on the right. History: hematuria with worm-like blood clots. Objectively: in the lumbar area on the right, a formation of elastic consistency the size of a fist is palpable, immobile, somewhat painful. A kidney neoplasm is suspected. What is the first sign of malignant neoplasms of the kidneys?

Втрата маси тiла Loss of body mass

Гарячка Fever

Болiсне утворення у фланках Painful formation in the flanks

Протеїнурiя Proteinuria

Гематурiя Hematuria

149 / 200
Хвора 39-ти рокiв скаржиться на раптову появу задишки, болю в груднiй клiтцi, серцебиття пiсля пробiжки в парку. В анамнезi: тривалий прийом оральних контрацептивiв, палiння. Об’єктивно: стан важкий, дифузний цiаноз. Частота дихання 36 /хв., пульс - 124/хв., малого наповнення, ритмiчний. Артерiальний тиск - 80/40 мм рт.ст. ЕКГ: блокада правої нiжки пучка Гiса, глибокий зубець QIII, глибокi зубцi S I, aVL, високий загострений зубець РII. Визначте серологiчний маркер даного невiдкладного стану: A 39-year-old patient complains of sudden onset of shortness of breath, chest pain, palpitations after jogging in the park. History: long-term use of oral contraceptives, smoking. About objectively: the condition is severe, diffuse cyanosis. Respiration rate 36/min., pulse - 124/min., low filling, rhythmic. Arterial pressure - 80/40 mm Hg. ECG: blockade of the right bundle branch of His, deep wave QIII , deep waves SI, aVL, high pointed wave PII. Determine the serological marker of this emergency:

Аспартатамiнотрансфераза Aspartate aminotransferase

Д-димер в кровi D-dimer in blood

Креатинфосфокiназа Creatine Phosphokinase

Тропонiн Tropon

Антинуклеарнi антитiла Antinuclear antibodies

150 / 200
Хворий 36-ти рокiв госпiталiзований у коматозному станi. Пiсля грипу з’явилися спрага, головний бiль. Напередоднi виник бiль у животi, блювання та втрата свiдомостi. Об’єктивно: запах ацетону з рота, шкiра та язик сухi, дихання глибоке, шумне, 23/хв. Артерiальний тиск - 90/60 мм рт.ст. Пульс 110/хв., малого наповнення. Живiт м’який. Печiнка збiльшена. Глiкемiя - 25,6 ммоль/л. В аналiзi сечi: глюкоза та ацетон. Поставте дiагноз: A 36-year-old patient is hospitalized in a comatose state. After the flu, thirst and headache appeared. The day before, he had abdominal pain, vomiting, and loss of consciousness. About' objectively: the smell of acetone from the mouth, dry skin and tongue, deep, noisy breathing, 23/min. Blood pressure - 90/60 mm Hg. Pulse 110/min., small filling. Abdomen is soft. Liver is enlarged. Glycemia - 25.6 mmol/l. In the urine analysis: glucose and acetone. Make a diagnosis:

Гiперосмолярна кома Hyperosmolar coma

Гiпоглiкемiчна кома Hypoglycemic Coma

Кетоацидотична кома Ketoacidotic Coma

Молочнокисла кома Lactic Coma

Харчова токсикоiнфекцiя Food poisoning

151 / 200
Бригада екстреної медичної допомоги (ЕМД) викликана у дитячий садок, де хлопчик 4-х рокiв випадково проковтнув iграшкову кульку. При оглядi: iнспiраторна задишка, судоми, мiдрiаз. Якi невiдкладнi заходи Крок 3 Загальна лiкарська пiдготовка (україномовний варiант) 2018 рiк, весна 19 необхiдно негайно провести в даному випадку? Emergency medical assistance team (EMS) was called to a kindergarten, where a 4-year-old boy accidentally swallowed a toy ball. On examination: inspiratory shortness of breath, convulsions, mydriasis. What urgent measures Step 3 General medical training (Ukrainian-language version) 2018, spring 19 should be carried out immediately in this case?

ШВЛ ”рот до рота” SHVL ”mouth to mouth”

Потрiйний прийом Сафара Triple reception of Safar

Прийом Селiка Selik Reception

Прекардiальний удар Precordial Stroke

Прийом Хеймлiха (Геймлiха) Reception of Heimlich (Heimlich)

152 / 200
У хворого 54-х рокiв з приводу вiдсутностi пульсу на сонних артерiях i дихання вже протягом 10 хвилин бригадою екстреної медичної допомоги (ЕМД) проводяться реанiмацiйнi заходи: зовнiшнiй масаж серця, штучне дихання мiшком Амбу, введення адреналiну, дефiбриляцiя. Стан залишається без змiн. З якою частотою i на яку глибину маєздiйснюватися компресiя грудної клiтки? In a 54-year-old patient, due to the absence of a pulse on the carotid arteries and breathing, within 10 minutes, the emergency medical assistance (EMD) team performed resuscitation measures: external heart massage , artificial respiration with an Ambu bag, administration of adrenaline, defibrillation. The condition remains unchanged. At what frequency and to what depth should chest compression be performed?

60-80 /хв. та 4-5 см 60-80/min and 4-5 cm

90-100 /хв. та 4-5 см 90-100/min and 4-5 cm

100-120 /хв. та 3-4 см 100-120 /min and 3-4 cm

100-120/хв. та 5-6 см 100-120/min and 5-6 cm

140 /хв. та 4-5 см 140 /min and 4-5 cm

153 / 200
Жiнка 52-х рокiв впала на вулицi. Свiдомiсть та серцева дiяльнiсть вiдсутнi. Реанiмацiю почали з непрямого масажу серця. Яка ознака свiдчить про його ефективнiсть? A 52-year-old woman collapsed on the street. Consciousness and cardiac activity are absent. Resuscitation was started with indirect heart massage. What sign testifies to its effectiveness?

Роздування живота Abdominal bloating

Поява пульсацiї на сонних артерiях Appearance of pulsation on the carotid arteries

Пiдняття грудної клiтки Chest lift

Посмикування кiнцiвок Twitching limbs

Розширення зiниць Pupil Dilation

154 / 200
Жiнка 45-ти рокiв пiсля конфлiктної ситуацiї на роботi вiдчула iнтенсивний головний бiль, була нудота та неодноразове блювання. До цього вважала себе здоровою. Об’єктивно: свiдомiсть ясна, артерiальний тиск - 160/100 мм рт.ст., частота серцевих скорочень 70 /хв., серцева дiяльнiсть ритмiчна, загальна гiперестезiя, фото-фонофобiя, ригiднiсть м’язiв потилицi. Парезiв та iнших неврологiчних розладiв не виявлено. Найбiльш iмовiрний дiагноз: After a conflict situation at work, a 45-year-old woman experienced an intense headache, nausea and repeated vomiting. Before that, she considered herself healthy. Objectively: consciousness is clear , arterial pressure - 160/100 mm Hg, heart rate 70/min, heart activity rhythmic, general hyperesthesia, photo-phonophobia, rigidity of the muscles of the back of the head. No paresis and other neurological disorders were detected. The most probable diagnosis:

Iстеричний невроз Hysterical Neurosis

Менiнгiт Менiнгiт

Гостра гiпертонiчна енцефалопатiя Acute hypertensive encephalopathy

Iшемiчний iнсульт Ischemic stroke

Субарахноїдальний крововилив Subarachnoid hemorrhage

155 / 200
До шпиталю доставлений вiйськовий, який був в 1 км вiд епiцентру ядерного вибуху. З анамнезу вiдомо, що пацiєнт пив воду з рiчки, через 6 годин пiсля чого з’явилися нудота, болi за ходом кишкiвника, блювання, рiдкi випорожнення. Об’єктивно: шкiра звичайного забарвлення, язик сухий, обкладений бiлим нальотом. Живiт м’який, болiсний в епiгастрiї. Артерiальний тиск - 110/60 мм рт.ст. Пульс ритмiчний, 84/хв. Поставте дiагноз: A military man who was 1 km from the epicenter of the nuclear explosion was brought to the hospital. It is known from the anamnesis that the patient drank water from the river, 6 hours later and then nausea, abdominal pain, vomiting, liquid stools appeared. Objectively: skin of normal color, tongue dry, coated with white plaque. Abdomen soft, painful in epigastrium. Blood pressure - 110/60 mm Hg. Pulse rhythmic , 84/min. Make a diagnosis:

Гостре респiраторне захворювання Acute respiratory disease

Радiацiйне ураження вiд внутрiшнього зараження радiонуклiдами Radiation damage from internal contamination with radionuclides

Гостра кишкова непрохiднiсть Acute intestinal obstruction

Гострий гастроентероколiт Hostry Gastroenterocolitis

Гострий геморагiчний гастрит Acute hemorrhagic gastritis

156 / 200
Хворий з цирозом печiнки, портальною гiпертензiєю III ступеня скаржиться на виражену слабкiсть, запаморочення, втрату свiдомостi, серцебиття, нудоту, блювання кров’ю, чорний рiдкий кал. Означенi симптоми наростають протягом доби. Шкiра i слизовi блiдi, iктеричнi, холоднi, липкi. Пульс - 100/хв., артерiальний тиск - 90/60 мм рт.ст. Живiт м’який, безболiсний. Аналiз кровi: еритроцити - 1, 8 • 1012/л, гемоглобiн - 60 г/л, КП- 1,0, лейкоцити - 8 • 109/л. Яке ускладнення виникло у хворого? A patient with cirrhosis of the liver, portal hypertension of the III degree complains of pronounced weakness, dizziness, loss of consciousness, palpitations, nausea, vomiting blood, black liquid stool. Marked symptoms increase during the day. Skin and mucous membranes are pale, icteric, cold, sticky. Pulse - 100/min., blood pressure - 90/60 mm Hg. Abdomen is soft, painless. Blood analysis: erythrocytes - 1, 8 • 1012/l, hemoglobin - 60 g/l, KP - 1.0, leukocytes - 8 • 109/l. What complication did the patient have?

Iнфаркт кишки Intestinal infarction

Печiнкова кома Pechin's Coma

Гемолiтичний криз Hemolytic crisis

Гостра кровотеча Acute bleeding

Перфорацiя виразки Perforation of ulcer

157 / 200
Хворий 42-х рокiв доставлений в iнфекцiйне вiддiлення на третiй день хвороби зi скаргами на головний бiль, загальну слабкiсть, сухiсть у ротi, двоїння в очах, утруднення при ковтаннi, вiдсутнiсть випорожнень. Напередоднi захворювання їв в’ялену рибу. Об’єктивно: гугнявий голос, птоз, мiдрiаз, анiзокорiя. Язик сухий, живiт здутий. Найефективнiшим лiкуванням хворого є: A 42-year-old patient was brought to the infectious disease department on the third day of illness with complaints of headache, general weakness, dry mouth, double vision, difficulty swallowing , absence of bowel movements. On the eve of the disease, he ate dried fish. Objectively: hoarse voice, ptosis, mydriasis, anisocoria. Dry tongue, swollen stomach. The most effective treatment of the patient is:

Сольовi розчини Saline solutions

Гiпербарична оксигенацiя Hyperbaric oxygenation

Протиботулiнiчна антитоксична сироватка Anti-botulinum anti-toxic serum

Антибiотики Antibiotics

Колоїднi розчини Colloidal solutions

158 / 200
Хворий 43-х рокiв, будiвельник, скаржиться на напруження i болючiсть жувальних м’язiв, утруднення при вiдкриваннi рота. Хвороба розпочалась 4 днi тому з появи тягнучого болю в дiлянцi рани на лiвiй кистi, поранення сталось 2 тижнi тому пiд час роботи. При оглядi вiдкривання рота утруднене через тонiчне напруження м’язiв. М’язи живота напруженi, болiснi. Температура 38,8o Вкажiть iмовiрний дiагноз: A 43-year-old patient, a construction worker, complains of tension and pain in the chewing muscles, difficulty opening the mouth. The disease began 4 days ago with the appearance of a pulling pain in areas of the wound on the left hand, the injury occurred 2 weeks ago during work. On examination, opening the mouth is difficult due to tonic tension of the muscles. The abdominal muscles are tense, painful. Temperature 38.8o Indicate the probable diagnosis:

Сказ Сказ

Правець Tetanus

Паратонзилярний абсцес Paratonsillar abscess

Полiомiєлiт Polio

Енцефалiт Encephalitis

159 / 200
Хворий 18-ти рокiв захворiв гостро з пiдвищенням температури до 39oC, катаральними явищами, головним болем, свiтлобоязню. На 5-й день з’явився рясний плямистопапульозний висип на обличчi та тулубi. Об’єктивно: iн’єкцiя судин склер, гiперемiя обличчя, слизова зiву гiперемована, енантема. На шкiрi обличчя та тулуба яскравий плямисто-папульозний висип. Над легенями жорстке дихання. Назвiть найбiльш iмовiрний дiагноз: Крок 3 Загальна лiкарська пiдготовка (україномовний варiант) 2018 рiк, весна 20 An 18-year-old patient became acutely ill with an increase in temperature to 39oC, catarrhal symptoms, headache, photophobia. On the 5th day, a profuse maculopapular rash appeared on face and trunk. Objectively: injection of scleral vessels, facial hyperemia, hyperemic pharyngeal mucosa, enanthema. A bright spotted-papular rash on the skin of the face and trunk. Hard breathing over the lungs. Name the most likely diagnosis: Step 3 General medical training ( Ukrainian version) 2018, spring 20

Кiр Кiр

Скарлатина Scarlet fever

Кропив’янка Hives

Вiтряна вiспа Wind Island

Iнфекцiйний мононуклеоз Infectious mononucleosis

160 / 200
Жiнка 25-ти рокiв скаржиться на плаксивiсть, рiзкi змiни настрою, прискорене серцебиття, що пов’язуєз втомою - мiсяць тому народила близнюкiв. З ранку вiдчула ”зупинки” серцевої дiяльностi. При оглядi з боку внутрiшнiх органiв патологiї не встановлено. Артерiальний тиск - 130/70 мм рт.ст., частота серцевих скорочень - 115/хв., частi екстрасистоли, частота дихання - 17/хв. Щитоподiбна залоза збiльшена до другого ступеня, безболiсна. Який найбiльш iмовiрний попереднiй дiагноз? A 25-year-old woman complains of tearfulness, sudden changes in mood, rapid heartbeat, which is associated with fatigue - she gave birth to twins a month ago. From the morning she felt 'stops' cardiac activity. When examining the internal organs, no pathology was found. Blood pressure - 130/70 mm Hg, heart rate - 115/min, frequent extrasystoles, respiratory rate - 17/min. The thyroid gland was enlarged to the second degree , painless. What is the most probable preliminary diagnosis?

Гострий вiрусний тиреоїдит, гiпертиреоз Acute viral thyroiditis, hyperthyroidism

Пiсляпологовий тиреоїдит, гiпертиреоз Postpartum thyroiditis, hyperthyroidism

Гострий вiрусний тиреоїдит, гiпотиреоз Acute viral thyroiditis, hypothyroidism

Пiсляпологовий тиреоїдит, гiпотиреоз Postpartum thyroiditis, hypothyroidism

Пiдгострий вiрусний тиреоїдит, гiпертиреоз Subacute viral thyroiditis, hyperthyroidism

161 / 200
Дитина 1 року 6 мiсяцiв доставлена у лiкарню швидкою допомогою зi скаргами батькiв на часте дихання. Пiд час бесiди стало вiдомо, що дитина була на кухнi, де мати розбирала рiзнi крупи та залишилася на деякий час без нагляду. При оглядi: задишка мiшаного характеру за участю допомiжної мускулатури. Частота дихання - 60/хв. Частота серцевих скорочень - 120/хв. Який з iнструментальних методiв треба застосувати? A 1-year-old 6-month-old child was taken to the hospital by ambulance with the parents' complaints of frequent breathing. During the conversation, it became known that the child was in the kitchen, where the mother was taking apart various croup and remained unattended for some time. On examination: dyspnea of ​​a mixed nature with the involvement of accessory muscles. Breathing rate - 60/min. Heart rate - 120/min. Which of the instrumental methods should be used?

Рентгеноскопiю Fluoroscopy

МРТ МРТ

Доплерехокардiографiю Doplerechocardiography

Бронхоскопiю Bronchoscopy

Спiрометрiю Spirometry

162 / 200
У пiдлiтка 15-ти рокiв пiд час забору кровi спостерiгалася раптова короткочасна втрата свiдомостi зi спонтанним вiдновленням стану. Пiд час огляду помiрна блiдiсть шкiрних покривiв, виражений гiпергiдроз кiнцiвок, частота дихання - 20/хв., тони серця звучнi, ритмiчнi, частота серцевих скорочень - 100/хв., артерiальний тиск - 90/60 мм рт.ст. Який невiдкладний стан розвинувся у дитини? A 15-year-old teenager experienced a sudden short-term loss of consciousness during blood sampling with spontaneous recovery. During the examination, moderate pallor of the skin, pronounced hyperhidrosis of the extremities, frequency breathing - 20/min., heart sounds are sonorous, rhythmic, heart rate - 100/min., blood pressure - 90/60 mm Hg. What emergency condition has developed in the child?

Психогенна атака Psychogenic Attack

Синкопе Syncope

Повна AV-блокада Full AV block

Гiпоглiкемiчний стан Hypoglycemic state

Епiлептичний напад Epileptic seizure

163 / 200
У дитини 3-х рокiв, яка перебуваєу стацiонарi, на тлi гострої респiраторної iнфекцiї 3 рази було блювання, непокоїть гострий бiль у животi. Температура тiла - 38,5oC. Шкiра блiда, суха. Дихання глибоке, частота серцевих скорочень - 130/хв. Цукор кровi 4,0 ммоль/л. Який з дiагнозiв найбiльш iмовiрний? A 3-year-old child, who is in a hospital, has vomited 3 times due to an acute respiratory infection, is worried about acute abdominal pain. Body temperature - 38.5oC The skin is pale, dry. Breathing is deep, heart rate is 130/min. Blood sugar is 4.0 mmol/l. Which of the diagnoses is the most probable?

Гострий апендицит Acute appendicitis

Ацетонемiчний стан Acetonemic state

Цукровий дiабет Diabetes

Гострий гастроентерит Acute gastroenteritis

Дiабетична кома Diabetic Coma

164 / 200
У жiнки 29-ти рокiв вдома вiдбулися стрiмкi II пологи у термiнi 36 тижнiв вагiтностi. Народився живий хлопчик масою 2300,0 г. Лiкар швидкої допомоги, який був викликаний, при оглядi породiллi вiдмiтив, що послiд не видiлився (пiсля народження дитини пройшло 40 хвилин). Ознак кровотечi немає. Яку допомогу повинен надати лiкар швидкої допомоги? A 29-year-old woman had a rapid II birth at home at 36 weeks of pregnancy. A live boy was born weighing 2300.0 g. The emergency physician who was called , during the examination of the woman in labor, he noted that the afterbirth was not separated (40 minutes had passed after the birth of the child). There are no signs of bleeding. What help should the emergency doctor provide?

Провести ручне вiддiлення послiду Manual separation of the trail

Ввести в/в 10 ОД окситоцину i дочекатися видiлення послiду Inject 10 units of oxytocin intravenously and wait for the discharge to occur

Негайно транспортувати жiнку до пологового вiддiлення III рiвня Immediately transport the woman to the maternity ward of level III

Негайно транспортувати жiнку i дитину до найближчого пологового вiддiлення Immediately transport the woman and child to the nearest maternity ward

Негайно транспортувати жiнку до пологового вiддiлення II рiвня Immediately transport the woman to the maternity ward of level II

165 / 200
Дитина 1,5 рокiв хворiєтретю добу: пiдвищення температури тiла, частi рiдкi випорожнення з зеленим вiдтiнком. Тургор тканин дещо знижений. Бiохiмiчнi показники: Ht- 0,52; K- 5,2 ммоль/л, N a- 154 ммоль/л. Призначте стартовий розчин для iнфузiйної терапiї: A 1.5-year-old child has been ill for the third day: increased body temperature, frequent liquid stools with a green tint. Tissue turgor is slightly reduced. Biochemical indicators: Ht- 0.52; K- 5.2 mmol/l, N a- 154 mmol/l. Prescribe the starting solution for infusion therapy:

Реосорбiлакт Rheosorbilact

Розчин Рiнгера Ringer's solution

Неогемодез Неогемодез

Реополiглюкiн Reopoliglyukin

5% розчин глюкози 5% glucose solution

166 / 200
До iнфекцiйного вiддiлення надiйшов хлопчик 4-х мiсяцiв у важкому станi. Дитина млява, гiподинамiчна. Вогнищевої симптоматики не виявлено. Велике тiм’ячко запале, 1,5х1,5 см. Шкiра суха, блiда. Тургор тканин знижений. Тахiпное - 60/хв., тахiкардiя 170/хв. З анамнезу: хворiв протягом тижня, коли виникло блювання пiсля кожного годування. Лабораторно: гемоглобiн - 146 г/л, гематокрит - 0,58, калiй плазми - 3,6 ммоль/л, натрiй плазми - 148 ммоль/л, глюкоза кровi - 3,6 ммоль/, сечовина - 13 ммоль/л. Який стартовий iнфузiйний розчин необхiдно призначити? A 4-month-old boy was admitted to the infectious department in serious condition. The child is lethargic, hypodynamic. No focal symptoms were detected. A large inflamed tetanus, 1.5x1, 5 cm. The skin is dry, pale. Tissue turgor is reduced. Tachypnea - 60/min., tachycardia 170/min. From the anamnesis: he was sick for a week, when vomiting occurred after each feeding. Laboratory: hemoglobin - 146 g/l, hematocrit - 0 .58, plasma potassium - 3.6 mmol/l, plasma sodium - 148 mmol/l, blood glucose - 3.6 mmol/l, urea - 13 mmol/l. What starting infusion solution should be prescribed?

Реосорбiлакт Rheosorbilact

5% розчин альбумiну 5% albumin solution

Розчин Рiнгер-лактату Ringer lactate solution

Iнфузiйна терапiя не показана Infusion therapy not indicated

10% розчин глюкози 10% glucose solution

167 / 200
Пiд час роботи в шахтi чоловiк 37-ми рокiв пошкодив кабель високої напруги, та був уражений струмом. Об’єктивно: стан вкрай тяжкий, дихання - 9/хв., частота серцевих скорочень - 56/хв. Яких лiкувальних заходiв треба негайно вжити? Крок 3 Загальна лiкарська пiдготовка (україномовний варiант) 2018 рiк, весна 21 While working in a mine, a 37-year-old man damaged a high-voltage cable and was electrocuted. Objectively: the condition is extremely serious, breathing - 9/min ., heart rate - 56/min. What medical measures should be taken immediately? Step 3 General medical training (Ukrainian-language version) 2018, spring 21

Введення атропiну сульфату Introduction of atropine sulfate

Електродефiбриляцiя Electrodefibrillation

Введення адреналiну гiдрохлориду Injection of adrenaline hydrochloride

Введення дихальних аналептикiв Introduction of respiratory analeptics

Штучна вентиляцiя легенiв Artificial lung ventilation

168 / 200
Чоловiк 54-х рокiв пiд час фiзичного навантаження три доби тому вiдчув гострий бiль у лiвiй нозi, швидко з’явилися набряк та посинiння ноги. Встановлено дiагноз: ”Гострий iлiофеморальний флеботромбоз злiва”. На УЗД вен нижнiх кiнцiвок виявлено оклюзiйний тромбоз загальної клубової вени. Якою буде лiкувальна тактика? A 54-year-old man felt a sharp pain in his left leg during physical activity three days ago, swelling and bruising of the leg quickly appeared. The diagnosis was established: 'Acute left iliofemoral phlebothrombosis'. Ultrasound of the veins of the lower extremities revealed occlusive thrombosis of the common iliac vein. What will be the treatment tactics?

Хiрургiчна венозна тромбектомiя Surgical venous thrombectomy

Тромболiтична терапiя Thrombolytic therapy

Системна антикоагуляцiя та дiуретики Systemic anticoagulation and diuretics

Системна антикоагуляцiя та протизапальнi засоби Systemic anticoagulation and anti-inflammatory drugs

Системна антикоагуляцiя Systemic anticoagulation

169 / 200
Хворий 62-х рокiв, що перебував на стацiонарному лiкуваннi з приводу мiокардиту раптово знепритомнiв, спостерiгались короткотривалi тонiчнi судоми скелетних м’язiв. Пiд час огляду: свiдомiсть вiдсутня, дихання поверхневе, рiдке, пульс на сонних артерiях вiдсутнiй, артерiальний тиск не визначається. При надходженнi на ЕКГ були перiодичнi випадiння комплексiв QRST без наростання iнтервалу Q-T. Яке ускладнення виникло у хворого? A 62-year-old patient undergoing inpatient treatment for myocarditis suddenly fainted, short-term tonic convulsions of skeletal muscles were observed. During the examination: consciousness is absent, breathing is shallow, liquid, there is no pulse on the carotid arteries, arterial pressure is not determined. During the ECG, there were periodic drops of QRST complexes without an increase in the QT interval. What complication occurred in the patient?

Гострий коронарний синдром Host Acute Coronary Syndrome

Гiпоглiкемiчна кома Hypoglycemic Coma

Синдром Морганьї-Адамса-Стокса Morgani-Adams-Stokes Syndrome

Напад епiлепсiї Attack of epilepsy

Динамiчне порушення мозкового кровообiгу Dynamic violation of cerebral circulation

170 / 200
Дiвчинка 10-ти рокiв протягом 3-х рокiв хворiєна цукровий дiабет. Отримуєiнсулiнотерапiю. Пiд час тривалої прогулянки пiшки дiвчинка раптово втратила свiдомiсть. Шкiрнi покриви вкрилися рясним потом, спостерiгається гiпертонус очних яблук, судоми. Яке ускладнення виникло у дитини? A 10-year-old girl has been suffering from diabetes for 3 years. She is receiving insulin therapy. During a long walk, the girl suddenly lost consciousness. Her skin was covered with profuse sweat, it is observed hypertonus of the eyeballs, convulsions. What complication did the child have?

Синдром Морганьї-Адамса-Стокса Morgani-Adams-Stokes Syndrome

Дiабетична нейропатiя Diabetic neuropathy

Молочнокисла кома Lactic Coma

Кетоацидотична кома Ketoacidotic Coma

Гiпоглiкемiчна кома Hypoglycemic coma

171 / 200
В жiночу консультацiю звернулася вагiтна в термiнi 37 тижнiв, яка протягом останнiх 3-х дiб не вiдчуваєворушiння плода. Якi дослiдження необхiдно провести для встановлення дiагнозу? A 37-week pregnant woman who has not felt the fetus move for the past 3 days came to the antenatal clinic. What tests should be performed to establish a diagnosis?

Клiнiчний аналiз сечi Clinical cross-sectional analysis

Клiнiчний аналiз кровi Clinical blood analysis

УЗД УЗД

Кардiотокографiя Cardiotocography

Амнiоскопiя Amnioscopy

172 / 200
У пацiєнта 35-ти рокiв пiсля надмiрного вживання алкоголю i калорiйної їжi з’явилися гострий бiль у верхнiй частинi живота, блювання без домiшок жовчi, здуття, температура - 38oC. Поступово наростав метеоризм, розвинулася затримка газiв i випорожнень. При аускультацiї кишковi шуми не вислуховуються. Поставте дiагноз: A 35-year-old patient developed acute pain in the upper part of the abdomen, vomiting without bile impurities, bloating, temperature - 38oC after excessive consumption of alcohol and high-calorie food . Flatulence gradually increased, gas and stool retention developed. Intestinal sounds are not heard during auscultation. Make a diagnosis:

Гостра кишкова непрохiднiсть Acute intestinal obstruction

Перфорацiя виразки Perforation of ulcer

Гострий панкреатит з явищами парезу кишечника Acute pancreatitis with intestinal paresis phenomena

Панкреонекроз Pancreonecrosis

Гострий холецистит Hostry Cholecystitis

173 / 200
Хвора 30-ти рокiв скаржиться на головний бiль, марення. Об’єктивно: температура тiла 39 oC, збуджена, обличчя гiперемоване, позитивний симптом Говорова-Годелье, петехiальний висип на тулубi, кiнцiвках, частота серцевих скорочень - 92/хв., артерiальний тиск - 90/60 мм рт.ст., гепатоспленомегалiя. Вживаєалкоголь, iн’єкцiйнi наркотики, живе в примiщеннi пiдвального типу. Поставте попереднiй дiагноз: A 30-year-old patient complains of headache, delirium. Objectively: body temperature 39 oC, excited, face hyperactive, positive Govorov-Godelier symptom, petechial a rash on the trunk, extremities, heart rate - 92/min, blood pressure - 90/60 mm Hg, hepatosplenomegaly. He consumes alcohol, injecting drugs, lives in a basement-type room. Make a preliminary diagnosis:

BIЛ-iнфекцiя HIV infection

Лептоспiроз Leptospirosis

Грип Грип

Алкогольний психоз Alcoholic psychosis

Висипний тиф Typhoid

174 / 200
У хворого 45-ти рокiв на 1 добу пiсля субтотальної резекцiї щитоподiбної залози з приводу дифузного токсичного зоба виникли психомоторне збудження, профузна пiтливiсть, тремор рук, гiперемiя обличчя, гiпертермiя - 40oC, частота серцевих скорочень - 140/хв., артерiальний тиск - 90/60 мм рт.ст. Яке ускладнення виникло у даного пацiєнта? A 45-year-old patient developed psychomotor agitation, profuse sweating, hand tremors, facial hyperemia, and hyperthermia 1 day after subtotal resection of the thyroid gland due to diffuse toxic goiter - 40oC, heart rate - 140/min., arterial pressure - 90/60 mm Hg. What complication occurred in this patient?

Пневмоторакс Pneumothorax

Гострий гiпопаратиреоз Acute hypoparathyroidism

Тиреотоксичний криз тяжкого ступеня Severe thyrotoxic crisis

Гiпотиреоз Гiпотиреоз

Гострий iнфаркт мiокарда Acute myocardial infarction

175 / 200
Хвора 27-ми рокiв звернулася до лiкаря зi скаргами на бiль внизу живота, пiдвищення температури до 38,7oC, слабкiсть. Об’єктивно: артерiальний тиск - 120/80 мм рт.ст., пульс - 100/хв. Симптом ЩоткiнаБлюмберга позитивний в гiпогастральнiй областi. Вагiнальне дослiдження: матка та додатки не визначаються через напруження передньої очеревинної стiнки. Заднєсклепiння нависає, рiзко болюче. Яким буде дiагноз? A 27-year-old patient turned to the doctor with complaints of lower abdominal pain, temperature rise to 38.7oC, weakness. Objectively: blood pressure - 120/ 80 mm Hg, pulse - 100/min. Shtokkin-Blumberg's symptom is positive in the hypogastric region. Vaginal examination: the uterus and appendages are not defined due to the tension of the anterior peritoneal wall. The posterior vault is overhanging, sharply painful. What will be the diagnosis?

Апоплексiя яєчника Ovarian apoplexy

Гострий сальпiнгоофорит Acute salpingo-oophoritis

Позаматкова вагiтнiсть Ectopic pregnancy

Гострий ендометрит Acute endometritis

Пельвiоперитонiт Pelvioperitonitis

176 / 200
У хворого через 48 годин пiсля застiлля, де вiн їв в’ялену рибу, з’явилися слабкiсть у м’язах, порушення зору, сухiсть у ротi, порушення дихання. Куди треба госпiталiзувати хворого? Крок 3 Загальна лiкарська пiдготовка (україномовний варiант) 2018 рiк, весна 22 48 hours after a meal where he ate dried fish, the patient developed muscle weakness, visual impairment, dry mouth, breathing. Where should the patient be hospitalized? Step 3 General medical training (Ukrainian-language version) 2018, spring 22

Неврологiчне вiддiлення Neurological Department

Терапевтичне вiддiлення Therapeutic department

ВРIТ iнфекцiйної лiкарнi ICU of infectious disease hospital

Очне вiддiлення Eye separation

Ревматологiчне вiддiлення Rheumatology Department

177 / 200
При оглядi кардiологом хворий 11-ти рокiв, який страждаєна вроджену ваду серця, раптово знепритомнiв. Шкiра та слизовi оболонки блiдо-цiанотичнi. Дихання агональне. Пульс на магiстральних артерiях не визначається. Який найбiльш iмовiрний дiагноз? During examination by a cardiologist, an 11-year-old patient suffering from a congenital heart defect suddenly fainted. The skin and mucous membranes are pale cyanotic. Breathing is agonal. Pulse on the main arteries not determined. What is the most likely diagnosis?

Зупинка серцевої дiяльностi Cardiac arrest

Простий синкопальний стан Simple syncopal state

Запаморочення Dizziness

Колапс Collapse

Кома Coma

178 / 200
До Вас звернулися батьки з дитиною 8-ми рокiв, покусаною вуличним собакою, який пiсля нападу зник. Рани розташованi у дiлянцi передплiччя та кистi. Оберiть оптимальний комплекс лiкувальних заходiв: You were contacted by the parents of an 8-year-old child bitten by a street dog, who disappeared after the attack. The wounds are located in the area of ​​the forearm and hand. Choose the optimal set of medical measures :

Промити рану мильним розчином, обробити шкiру антисептиком Wash the wound with a soap solution, treat the skin with an antiseptic

Якнайшвидше провести антирабiчне щеплення Get vaccinated against rabies as soon as possible

Призначити антибiотики та якнайшвидше провести антирабiчне щеплення Prescribe antibiotics and carry out anti-rabies vaccination as soon as possible

Промити рану мильним розчином, обробити шкiру антисептиком, та якнайшвидше провести антирабiчне щеплення Wash the wound with a soapy solution, treat the skin with an antiseptic, and administer an anti-rabies vaccination as soon as possible

Промити рану мильним розчином, обробити шкiру антисептиком, призначити антибiотики та якнайшвидше провести антирабiчне щеплення Wash the wound with a soapy solution, treat the skin with an antiseptic, prescribe antibiotics and administer an anti-rabies vaccination as soon as possible

179 / 200
Хвора 53-х рокiв, яка протягом 2-х мiсяцiв перебувала в зонi бойових дiй, скаржиться на бiль в дiлянцi серця, напади серцебиття, диcфункцiю кишково-шлункового тракту, безсоння, нiчнi кошмари, тривогу, вiдчуття страху, яке посилюється, коли хвора чуєзвуки, якi нагадують пострiли чи вибухи. Артерiальний тиск - 130/80 мм рт.ст., тахiкардiя. Куди треба госпiталiзувати хвору? A 53-year-old patient, who was in the war zone for 2 months, complains of pain in the area of ​​the heart, palpitations, dysfunction of the gastrointestinal tract , insomnia, night nightmares, anxiety, a feeling of fear that intensifies when the patient hears sounds reminiscent of gunshots or explosions. Blood pressure - 130/80 mm Hg, tachycardia. Where should the patient be hospitalized?

В гастроентерологiчне вiддiлення In the gastroenterology department

В кардiологiчне вiддiлення In the cardiology department

В психiатричне вiддiлення In a psychiatric ward

В хiрургiчне вiддiлення In the surgical department

В iнфекцiйне вiддiлення In infectious department

180 / 200
Розривом мiни бiйця вiдкинуло лiвим боком на кам’яну брилу, пiсля чого вiн тимчасово втратив свiдомiсть. Пiд час огляду блiдий, постiйно поривається сiсти. Порушення цiлiсностi ребер та кiсток не виявлено. Живiт болючий в лiвому пiдребер’ї, притуплення перкуторного звуку в бокових вiддiлах. Артерiальний тиск - 80/60 мм рт.ст. Пульс 120/хв. Який попереднiй дiагноз? The explosion of a mine threw the fighter on his left side onto a stone block, after which he temporarily lost consciousness. During the examination, he is pale, constantly tries to sit down. Violation of the integrity of the ribs and bones not detected. The abdomen is painful in the left hypochondrium, dulling of the percussion sound in the lateral areas. Blood pressure - 80/60 mm Hg. Pulse 120/min. What is the preliminary diagnosis?

Тупа травма живота, розрив печiнки, травматичний шок Blunt abdominal injury, liver laceration, traumatic shock

Тупа травма живота без пошкодження внутрiшнiх органiв Blunt abdominal injury without damage to internal organs

Закрита травма живота, розрив селезiнки, геморагiчний шок Closed abdominal injury, spleen rupture, hemorrhagic shock

Тупа травма живота, розрив аневризми черевної частини аорти, геморагiчний шок Blunt trauma to the abdomen, rupture of an aneurysm of the abdominal part of the aorta, hemorrhagic shock

Закрита травма живота, розрив тонкої кишки, геморагiчний шок Closed abdominal injury, rupture of the small intestine, hemorrhagic shock

181 / 200
Чоловiк 54-х рокiв на вулицi раптово втратив свiдомiсть. По прибуттi лiкарем бригади швидкої медичної допомоги встановлено ознаки клiнiчної смертi. Проводились реанiмацiйнi заходи. На ЕКГ визначена фiбриляцiя шлуночкiв. Тричi проведено дефiбриляцiю. Медикаментозне лiкування слiд розпочати з: A 54-year-old man suddenly lost consciousness on the street. Upon arrival, the doctor of the emergency medical team found signs of clinical death. Resuscitation measures were carried out. Ventricular fibrillation was detected on the ECG. Defibrillation was performed three times. Medical treatment should be started with:

Магнiю сульфату Magnesium sulfate

Новокаїнамiду Novokanamidu

Амiодарону Амiодарону

Анальгiну Analgin

Натрiю гiдрокарбонату Sodium Bicarbonate

182 / 200
Хворий 49-ти рокiв надiйшов зi скаргами на бiль за грудниною стискального характеру, який з’явився пiсля фiзичного навантаження, iррадiюєв лiве плече. Пацiєнт самостiйно приймав нiтроглiцерин, проте стан суттєво не змiнився (за 30 хвилин). Об’єктивно: частота серцевих скорочень - 94/хв., пульс ритмiчний, артерiальний тиск - 115/70 мм рт.ст. На ЕКГ: пiдйом сегменту ST у лiвих вiдведеннях. Який найбiльш iмовiрний дiагноз? A 49-year-old patient came in with complaints of chest pain of a compressive nature, which appeared after physical exertion, radiating to the left shoulder. The patient took nitroglycerin on his own, but the condition did not change significantly (in 30 minutes). Objectively: heart rate - 94/min., pulse rhythmic, arterial pressure - 115/70 mm Hg. On the ECG: elevation of the ST segment in the left leads. Which is the most likely diagnosis?

Спонтанний пневмоторакс Spontaneous pneumothorax

Гострий мiокардит Acute myocarditis

Розшарувальна аневризма аорти Dissecting aortic aneurysm

Гострий коронарний синдром Acute coronary syndrome

Тромбоемболiя легеневої артерiї Thromboembolism of the pulmonary artery

183 / 200
Жiнка 38-ми рокiв скаржиться на напади головного болю одностороннього характеру. Головний бiль iнтенсивний, локалiзується в лобно-скроневiй областi, починається раптово, триваєвiд 3 до 10 годин. Перед нападом вiдзначається ”затуманення” зору. Мiж нападами головного болю - стан задовiльний. Неврологiчний статус без особливостей. Якi з наведених продуктiв можуть спровокувати напад? A 38-year-old woman complains of unilateral headache attacks. The headache is intense, localized in the frontotemporal region, begins suddenly, lasts from 3 to 10 hours. 'Blurred' vision is noted before the attack. Between headache attacks, the condition is satisfactory. The neurological status is unremarkable. Which of the following products can provoke an attack?

Смажена картопля Fried potatoes

Молочнокислi продукти Lactic Products

Бiлий хлiб White bread

Ананас Pineapple

Шоколад Шоколад

184 / 200
Юнак надiйшов до стацiонару на 7-й день хвороби, що розпочалася гостро з остуди, пiдвищення температури тiла до 39,5oC, болю у м’язах, головного болю. Тиждень тому повернувся з Пiвденної Америки. Свiдомiсть на рiвнi сопору, шкiра та склери жовтяничнi. На тулубi та кiнцiвках рясний геморагiчний висип. Температура тiла - 38,8o. Пульс 110/хв., артерiальний тиск - 90/60 мм рт.ст. Гепатоспленомегалiя. Олiгурiя. В кровi: проКрок 3 Загальна лiкарська пiдготовка (україномовний варiант) 2018 рiк, весна 23 тромбiновий iндекс - 45%, сечовина - 24,8 ммоль/л. Дiагностовано жовту гарячку. Чим зумовлена тяжкiсть стану хворого? The young man entered the hospital on the 7th day of the illness, which began acutely with a cold, an increase in body temperature to 39.5oC, muscle pain, headache A week ago, he returned from South America. Consciousness at the level of sopor, the skin and sclera are jaundiced. A profuse hemorrhagic rash on the trunk and extremities. Body temperature - 38.8o. Pulse 110/min., blood pressure - 90/60 mm Hg. Hepatosplenomegaly. Oliguria. In the blood: proStep 3 General medical training (Ukrainian-language version) 2018, spring 23, thrombin index - 45%, urea - 24.8 mmol/l. Yellow fever is diagnosed. What is the cause of the severity of the patient's condition?

Гострою печiнково-нирковою недостатнiстю Acute liver and kidney failure

Гострою затримкою сечi Acute urinary retention

Гострою асфiксiєю Acute asphyxia

Iнфекцiйно-токсичним шоком Infectious Toxic Shock

Гiповолемiчним шоком Hypovolemic Shock

185 / 200
До сiмейного лiкаря звернулася хвора 54-х рокiв, зi скаргами на ”стартовий”, ”блокадний” бiль в колiнних та кульшових суглобах, рухи обмеженi, болючi. Хвора працює кухарем, пiдвищеного живлення. З анамнезу вiдомо, що мати хворої теж мала подiбнi скарги. Який метод дiагностики євизначальним для встановлення дiагнозу? A 54-year-old patient came to the family doctor with complaints of 'starting', 'blocking' pain in the knee and hip joints, limited, painful movements. The patient works as a cook, with increased nutrition. It is known from the anamnesis that the patient's mother also had similar complaints. What diagnostic method is decisive for establishing the diagnosis?

Загальний аналiз кровi General blood analysis

Рентгенографiя X-ray

Бiохiмiчне визначення рiвня сечовини Biochemical determination of urea level

Артроскопiя Arthroscopy

Загальний аналiз сечi General urine analysis

186 / 200
Хворий 33-х рокiв, приїхав з Ефiопiї, де перебував у туристичнiй поїздцi. На другу добу виникла профузна дiарея, блювання фонтаном до 7-8 разiв на добу, сухiсть в ротi, загальна слабкiсть. Об’єктивно: хворий блiдий, акроцiаноз. Шкiра суха, холодна на дотик. Пiд очима темнi кола, живiт запалий, температура - 35,8oC, артерiальний тиск 80/40 мм рт.ст., пульс - 105/хв., слабкого наповнення. Судоми литкових м’язiв. Яке невiдкладне лiкування необхiдно призначити хворому? The patient is 33 years old, came from Ethiopia, where he was on a tourist trip. On the second day, he developed profuse diarrhea, vomiting in a fountain up to 7-8 times a day, dry mouth, general weakness. Objectively: the patient is pale, acrocyanosis. The skin is dry, cold to the touch. Dark circles under the eyes, the abdomen is inflamed, temperature - 35.8oC, blood pressure 80/40 mm Hg, pulse - 105/min., weak filling. Cramps of the calf muscles. What emergency treatment should be prescribed to the patient?

Кортикостероїди Corticosteroids

Глюкоза з iнсулiном Glucose with insulin

Реополiглюкiн Reopoliglyukin

Кристалоїднi сольовi розчини Crystalloid salt solutions

Парентеральне харчування Parenteral nutrition

187 / 200
У дитини 3,5 рокiв iз тетрадою Фалло раптово виникли задуха, збудження, дифузний цiаноз i тахiкардiя. Пiсля призначення пропранололу стан дитини нормалiзувався. Яка причина цього епiзоду задухи? A 3.5-year-old child with tetrad of Fallot suddenly developed suffocation, agitation, diffuse cyanosis, and tachycardia. After prescribing propranolol, the child's condition normalized. What is the cause of this episode of suffocation?'

Гостра лiвошлуночкова недостатнiсть Acute left ventricular failure

Стенозувальний ларингiт Stenosis laryngitis

Задишково-цiанотичний криз Dyspnea-cyanotic crisis

Тромбоемболiя легеневої артерiї Thromboembolism of the pulmonary artery

Гостра дихальна недостатнiсть Acute respiratory failure

188 / 200
Дiвчинка 15-ти рокiв госпiталiзована зi скаргами на блювання ”кавовою гущею”, жовтушнiсть шкiри, носовi кровотечi, лихоманку. Об’єктивно: дихання уповiльнене, шкiра iктерична, частота серцевих скорочень - 110/хв., артерiальний тиск - 80/50 мм рт.ст. В бiохiмiчному аналiзi кровi: АлАт220 ОД•л; АсАт- 300 ОД•л; загальний бiлiрубiн - 160 мкмоль/л, протромбiновий iндекс 30%, фiбриноген - 1,1 г/л. З анамнезу: дитина страждаєна хворобу Вiльсона-Коновалова. Яке ускладнення виникло у дитини? A 15-year-old girl was hospitalized with complaints of vomiting 'coffee grounds', yellow skin, nosebleeds, fever. Objectively: breathing is slowed down, skin is icteric, heart rate - 110/min., arterial pressure - 80/50 mm Hg. In the biochemical analysis of blood: AlAt220 U·l; AsAt - 300 U·l; total bilirubin - 160 μmol/l, prothrombin index 30%, fibrinogen - 1.1 g/l. From the anamnesis: the child suffers from Wilson-Konovalov disease. What complications did the child have?

Гемолiтичний криз Hemolytic crisis

Гостра ниркова недостатнiсть Acute renal failure

Гостра надниркова недостатнiсть Acute adrenal insufficiency

Гостра печiнкова недостатнiсть Acute liver failure

Iнфекцiйно-токсичний шок Infectious-toxic shock

189 / 200
Поранений з мiнно-вибуховою травмою доставлений з поля бою в мобiльний госпiталь у станi шоку. Пульс на променевiй артерiї не визначається. Артерiальний тиск - 50/0 мм рт.ст. Якi першочерговi заходи мають бути здiйсненi на цьому етапi? The wounded man with a mine-explosive injury was brought from the battlefield to a mobile hospital in a state of shock. The pulse on the radial artery is not determined. Blood pressure - 50/0 mm Hg. Art. What priority measures should be taken at this stage?

Крапельна iнфузiя 0,9% розчину N aCl у межах подвоєної добової потреби в рiдинi Drip infusion of 0.9% NaCl solution within twice the daily fluid requirement

Швидке внутрiшньовенне введення колоїдно-кристалоїдних розчинiв Rapid intravenous administration of colloid-crystalloid solutions

Внутрiшньовенна iнфузiя мезатону до вiдновлення пульсу на променевiй артерiї Intravenous infusion of mesaton until recovery of the pulse on the radial artery

Ультразвукове дослiдження черевної та плевральних порожнин для виключення кровотечi Ultrasound examination of the abdominal and pleural cavities to rule out bleeding

Внутрiшньом’язово ввести 1 мл адреналiну. Повторити через 5 хвилин при неефективностi Intramuscularly inject 1 ml of adrenaline. Repeat after 5 minutes if ineffective

190 / 200
Поранений з травматичною ампутацiєю правої верхньої кiнцiвки доставлений з поля бою в мобiльний госпiталь у станi шоку. Артерiальний тиск - 50/0 мм рт.ст. На травмованiй кiнцiвцi - турнiкет. Якi першочерговi заходи мають бути здiйсненi на цьому етапi? The wounded with traumatic amputation of the right upper extremity was brought from the battlefield to the mobile hospital in a state of shock. Blood pressure - 50/0 mm Hg. On the injured extremity - turnstile. What priority measures should be taken at this stage?

Розпочати внутрiшньовенну iнфузiю мезатону i норадреналiну Start intravenous infusion of mesatone and norepinephrine

Швидке вiдновлення ОЦК, адекватне знеболювання Quick recovery of BCC, adequate analgesia

Внутрiшньовенно ввести 1 мл адреналiну. Повторювати кожнi 5 хвилин для пiдтримки адекватного артерiального тиску Inject 1 ml of adrenaline intravenously. Repeat every 5 minutes to maintain adequate blood pressure

Розпочати краплинну iнфузiю 0,9% розчину натрiю хлориду у межах подвоєної добової потреби в рiдинi Start a drip infusion of 0.9% sodium chloride solution within twice the daily fluid requirement

Послабити турнiкет. Якщо вiдновилась кровотеча, то ввести гемостатичнi препарати i розпочати iнфузiю кристалоїдiв Relax the tourniquet. If bleeding resumes, administer hemostatic drugs and start crystalloid infusion

191 / 200
У пораненого в дiлянку черепа спостерiгається бiль у грудях, кровохаркання, кашель, задишка. Температура тiла - 37,0o. При обстеженнi: дихання в легенях ослаблено, хрипiв немає. При рентгенологiчному дослiдженнi виявляються множиннi дрiбнi тiнi в обох легенях. Яка причина виявлених змiн? The injured person in the area of ​​the skull has chest pain, hemoptysis, cough, shortness of breath. Body temperature - 37.0o. On examination: breathing in the lungs is weakened, there is no wheezing . During an X-ray examination, multiple small shadows are revealed in both lungs. What is the cause of the detected changes?

Гемоторакс Hemothorax

Крововилив у легенi Hemorrhage in the lungs

Гемоперикард Hemopericardium

Аспiрацiйна пневмонiя Aspiration pneumonia

Пневмоторакс Pneumothorax

192 / 200
В хiрургiчне вiддiлення вiйськового шпиталю доставлений поранений iз зони бойових дiй, з проникним вогнепальним ураженням черевної порожнини. При обстеженнi встановлено наявнiсть пошкодження 0,5 см нижньої третини лiвого сечоводу на вiдстанi 3 см вiд сечового мiхура, що поєднане з пошкодженням прямої кишки. Якою буде тактика лiкування? Крок 3 Загальна лiкарська пiдготовка (україномовний варiант) 2018 рiк, весна 24 A wounded man from the combat zone was brought to the surgical department of a military hospital with a penetrating gunshot wound to the abdominal cavity. During the examination, the presence of a 0.5 cm damage to the lower third of the left ureter on a distance of 3 cm from the urinary bladder, which is combined with damage to the rectum. What will be the treatment tactics? Step 3 General medical training (Ukrainian-language version) 2018, spring 24

Ушивання прямої кишки, уретерокутанеостомiя Suturing of the rectum, ureterocutaneous ostomy

Iмплантацiя сечоводу в пряму кишку Ureter implantation into the rectum

Колостомiя, уретерокутанеостомiя Colostomy, ureterocutaneous stomy

Ушивання дефектiв прямої кишки, сечоводу Suturing defects of the rectum, ureter

Колостомiя, первинний анастомоз сечоводу Colostomy, primary ureteral anastomosis

193 / 200
Вiйськовослужбовець потрапив пiд бомбардування, отримав закриту черепномозкову травму; при оглядi спостерiгаються змiни в неврологiчному статусi та психiцi. До якого госпiталю направити хворого? The military serviceman was hit by a bombardment, received a closed craniocerebral injury; during the examination, changes in the neurological status and psyche are observed. To which hospital should the patient be sent?

Вiйськово-польового терапевтичного госпiталю Military Field Therapeutic Hospital

Вiйськово-польового неврологiчного госпiталю Military Field Neurological Hospital

Вiйськово-польового травматологiчного госпiталю Military field trauma hospital

Вiйськово-польового iнфекцiйного госпiталю Military field infectious hospital

Вiйськово-польового хiрургiчного госпiталю Military Field Surgical Hospital

194 / 200
Хворий 44-х рокiв звернувся зi скаргами на зниження гостроти зору, нежить з рясними рiдкими видiленнями, слинотечу, утруднення дихання, мимовiльнi посмикування м’язiв, тахiкардiю, пiдвищення артерiального тиску. Об’єктивно: хворий збуджений, температура тiла - 37,5o. Пульс - 126/хв., артерiальний тиск - 180/90 мм рт.ст., зiницi рiзко звуженi. При аускультацiї в легенях поодинокi хрипи. Отруєння речовиною якої дiї найбiльш iмовiрно у даному випадку? A 44-year-old patient complained of decreased visual acuity, runny nose with abundant liquid secretions, drooling, difficulty breathing, involuntary muscle twitches, tachycardia, increased of arterial pressure. Objectively: the patient is excited, body temperature - 37.5o. Pulse - 126/min., arterial pressure - 180/90 mm Hg, pupils are sharply narrowed. When auscultating the lungs, isolated wheezing. Poisoning by which substance action most likely in this case?

Шкiрно-наривної Skin-abscess

Сльозогiнної Slosohinnoy

Задушливої Suffocating

Нервово-паралiтичної Nerve-paralytic

Загальноотруйної Generally poisonous

195 / 200
До медичної роти бригади доставлено вiйськовослужбовцiв з ознаками iнфекцiйного кишкового захворювання (гiпертермiя, дiарея). До якої сортувальної групи необхiдно зарахувати цих хворих при проведеннi медичного сортування? Servicemen with signs of an infectious intestinal disease (hyperthermia, diarrhea) were delivered to the medical company of the brigade. To which triage group should these patients be assigned during medical triage?

Тi, що потребують надання медичної допомоги на даному етапi медичної евакуацiї в другу чергу Those who need medical assistance at this stage of medical evacuation in the second place

Тi, що пiдлягають подальшiй евакуацiї без надання їм медичної допомоги Those subject to further evacuation without medical assistance

Тi, що пiдлягають поверненню до своїх пiдроздiлiв пiсля надання їм медичної допомоги Those subject to return to their units after providing them with medical assistance

Тi, що потребують надання медичної допомоги на даному етапi медичної евакуацiї в першу чергу Those who need medical assistance at this stage of medical evacuation in the first place

Небезпечнi для оточуючих Dangerous for others

196 / 200
Пiд час патрулювання вiйськовослужбовець отримав мiнно-вибухову травму. Об’єктивно: лiва стопа вiдсутня, iз рани гомiлки спостерiгається виражена кровотеча. Шкiра блiда, пульс на променевiй артерiї вiдсутнiй, на соннiй артерiї - 112/хв. В чому полягаєалгоритм надання домедичної допомоги? During patrolling, the serviceman received a mine-explosive injury. Objectively: the left foot is missing, severe bleeding is observed from the leg wound. The skin is pale, there is no pulse on the radial artery , on the carotid artery - 112/min. What is the algorithm for providing pre-medical care?

Накладання асептичної пов’язки, у разi неможливостi зупинки кровотечi накладання кровоспинного джгута Applying an aseptic bandage, if it is impossible to stop the bleeding, applying a hemostatic tourniquet

Накладання джгута та асептичної пов’язки, транспортна iммобiлiзацiя, знеболення Applying a tourniquet and aseptic bandage, transport immobilization, anesthesia

Знеболення, накладання кровоспинного джгута та негайна евакуацiя на наступний етап Anesthesia, application of a hemostatic tourniquet and immediate evacuation to the next stage

Знеболення, накладання кровоспинного джгута, накладання асептичної пов’язки Anesthesia, applying a hemostatic tourniquet, applying an aseptic bandage

Знеболення та негайна евакуацiя на наступний етап Anesthesia and immediate evacuation to the next stage

197 / 200
Яких заходiв домедичної допомоги необхiдно вжити при клапанному пневмотораксi? What pre-medical measures should be taken for valvular pneumothorax?

Плевральна пункцiя Pleural Puncture

Торакотомiя та бронхотомiя Thoracotomy and Bronchotomy

Герметична пов’язка на рану Seamless wound dressing

Штучна вентиляцiя легенiв Artificial lung ventilation

Дренування плевральної порожнини за Бюлау Drainage of the pleural cavity according to Bülau

198 / 200
Полковника 39-ти рокiв поранено у лiве стегно. Спостерiгається пульсуюча кровотеча з рани, кров яскраво-червоного кольору. Скарги пораненого на слабкiсть, серцебиття, запаморочення, бiль в ураженiй кiнцiвцi. Тони серця ритмiчнi. Пульс - 104/хв. Артерiальний тиск - 110/65 мм рт.ст. Живiт м’який, безболiсний. Що дозволить попередити розвиток геморагiчного шоку? A 39-year-old colonel was wounded in the left thigh. Pulsating bleeding from the wound is observed, the blood is bright red. The wounded complains of weakness, palpitations, dizziness, pain in affected extremities. Heart sounds are rhythmic. Pulse - 104/min. Blood pressure - 110/65 mm Hg. Abdomen is soft, painless. What will prevent the development of hemorrhagic shock?

Накладання джгута вище мiсця поранення Applying a tourniquet above the wound

Накладання шини Дiтерiхса Dieterichs bus overlay

Накладання пов’язки типу ”Дезо” Deso dressing

Накладання джгута нижче мiсця поранення Applying a tourniquet below the wound

Накладання пов’язки типу ”Дамуазо” Damoisot dressing

199 / 200
Пацiєнт 25-ти рокiв демобiлiзований та госпiталiзований на 3-ю добу пiсля сильного обстрiлу пiд час бою. Скаржиться на тривогу, вiдстороненiсть, вiдсутнiсть апетиту, слабкiсть, швидку змiну настрою. Симптоми швидко припинилися пiсля дiї стресового фактору через 48 годин. Поставте дiагноз: A 25-year-old patient was demobilized and hospitalized on the 3rd day after heavy shelling during the battle. He complains of anxiety, detachment, lack of appetite, weakness, rapid change mood. The symptoms quickly stopped after the action of the stress factor after 48 hours. Diagnose:

Гостра реакцiя на стрес Acute reaction to stress

Депресивний синдром Depressive syndrome

Тривожний синдром Anxiety syndrome

Дисоцiативний розлад особистостi Dissociative personality disorder

Розлад свiдомостi Disordered consciousness

200 / 200
Солдат активний, рухливий, метушливий. Чинить опiр обстеженню. Говорить швидко, голосно. Вислови спонтаннi, непослiдовнi. Поставте дiагноз: The soldier is active, mobile, fussy. Resists examination. Speaks quickly, loudly. Expressions are spontaneous, incoherent. Make a diagnosis:

Кататонiчне збудження Catatonic excitement

Аменцiя Amentia

Ейфорiя Euphoria

Делiрiй Delirium

Психомоторне збудження Psychomotor excitement